Anda di halaman 1dari 71

www.onlinelesson.

org    336 
Simulasi Soal Snmptn 2011 
 
Mata Pelajaran  : Matematika Dasar 
Kode Soal    : 336 
 
 
Gunakan  PETUNJUK  A  untuk  menjawab  soal  4. Fungsi  f (x) = x 2 + ax   mempunyai  grafik 
nomor 1 sampai dengan nomor 15!  berikut. 
 
1. Pernyataan  yang  mempunyai  nilai 
kebenaran  sama  dengan  pernyataan:  “Jika 
bilangan  ganjil  sama  dengan  bilangan 
genap, maka 1 + 2 bilangan ganjil” adalah ... 
(A) “Jika  bilangan  ganjil  tidak  sama   
dengan  bilangan  genap,  maka  1  +  2  Grafik fungsi  g(x) = x 2 − ax + 5  adalah ... 
bilangan genap”  (A)  
(B) “Bilangan ganjil sama dengan bilangan   
genap dan 1 + 2 bilangan ganjil”    
(C)  “Jika  1  +  2  bilangan  ganjil,  maka   
bilangan  ganjil  sama  dengan  bilangan   
genap”    
(D) “Jika  bilangan  ganjil  sama  dengan   
bilangan  genap,  maka  1  +  2  bilangan   
genap”  (B)  
(E) “Bilangan ganjil sama dengan bilangan   
genap dan 1 + 2 bilangan genap”    
   
2. Jika n memenuhi   
25
144
0 , 25
×4 0 , 25
254 4 ×2
4254 4×4
0 , 25
L4×4
2543 = 125  
0 , 25
 
n faktor  
Maka (n − 3)(n + 2) = ...  (C)  
(A) 24   
(B) 26   
(C) 28   
(D) 32   
(E) 36   
   
3. Persamaan  x 2 − ax − (a + 1) = 0   mempunyai  (D)  
akar‐akar  x1 > 1  dan  x 2 < 1  untuk …   
 
(A) a < 0  
 
(B) a > 0  
 
(C) a > −2    
(D) − 2 < a < 0    
(E) a ≠ −2   (E)  
   
   
   
   
   
   

an_ay
 
 

Halaman 1 
www.onlinelesson.org    336 
5. Nilai  x  yang  memenuhi  pertidaksamaan  10. Balok  ABCD.EFGH   mempunyai  panjang 
x +1 x rusuk  AB = 4cm ,  BC = 3cm , dan  AE = 3cm . 
>  adalah ....  Bidang  AFH  memotong  balok  menjadi  2 
x +1 x −1
(A) −1 ≤ x < 1  bagian  dengan  perbandingan  volumenya 
(B) x > −1  adalah .... 
(C) x < 1  (A) 1 : 3 
(B) 2 : 3 
(D) x < −1 atau x > 1 
(C) 3 : 5 
(E) x < −1 atau −1 < x < 1 
(D) 1 : 5 
 
(E) 1 : 6 
6. Jika  −6, a , b, c , d , e , f , g ,18   merupakan 
 
barisan aritmatika, maka  a + d + g = .... 
11. Jika  fungsi  f ( x, y ) = 5000 − x − y   dengan 
(A) 12 
syarat  x ≥ 0 ,  y ≥ 0 ,  x − 2 y + 2 ≥ 0 ,  dan 
(B) 18 
(C) 24  2 x + y − 6 ≥ 0 , maka .... 
(D) 30  (A) fungsi  f   mempunyai  nilai  minimum 
(E) 36  dan nilai maksimum 
  (B) fungsi  f   tidak  mempunyai  nilai 
7. Jika M adalah matriks sehingga   minimum maupun nilai maksimum 
⎛a b⎞ ⎛ a b ⎞ (C) fungsi  f   mempunyai  nilai  minimum 
M⎜⎜ ⎟⎟ = ⎜⎜ ⎟⎟  
⎝c d⎠ ⎝− a + c − b + d⎠ dan tidak mempunyai nilai maksimum 
maka determinan matriks M adalah ....  (D) fungsi  f  mempunyai nilai maksimum 
(A) 1  dan tidak mempunyai nilai minimum 
(B) −1  (E) nilai  minimum  dan  nilai  maksimum 
(C) 0  fungsi  f  tidak dapat ditentukan 
(D) −2   
(E) 2  12. Jika  p < −3  dan  q > 5  , maka nilai  q − p  .... 
  (A) lebih besar daripada 9 
8. Jika penyelesaian sistem persamaan  (B) lebih besar daripada 7 
⎧(a − 2) x + y = 0 (C) lebih kecil daripada 8 
⎨  
⎩ x + (a − 2) y = 0 (D) lebih kecil daripada 2 
Tidak  hanya  ( x, y ) = (0,0)   saja,  maka  nilai  (E) lebih kecil daripada −2 
 
a 2 − 4a + 3 = ....  13. Distribusi  frekuensi  usia  pekerja  pada 
(A)   0  perusahaan  A  dan  perusahaan  B  diberikan 
(B)   1  pada tabel berikut. 
(C)   4   

(D)   9  Usia  Banyak Pekerja 


(E) 16  (tahun)  Perusahaan A  Perusahaan B
  20 – 29  7  1 
9. Jika  g(x − 2) = 2 x − 3   dan  30 – 39  26  8 
( fog)(x − 2) = 4 x − 8 x + 3 , maka  f (−3) = .... 
2 40 – 49  15  1 
50 – 59  2  32 
(A) 15 
60 – 69  0  8 
(B) 12 
(C)   3  Total  50  50 
 

(D)   0  Berdasarkan  data  pada  tabel  tersebut, 


(E) −3  kesimpulan yang tidak benar adalah 
  (A) rata‐rata,  median  dan  modus  usia 
  pekerja  perusahaan  A  masing‐masing 
  lebih  rendah  daripada  rata‐rata, 

an_ay
  median,  dan  modus  usia  pekerja 
  perusahaan B. 
 

Halaman 2 
www.onlinelesson.org    336 
(B) rata‐rata  usia  pekerja  perusahaan  A 
lebih  kecil  daripada  median  usia 
pekerja perusahaan B. 
(C) modus  usia  pekerja  perusahaan  A 
lebih  kecil  daripada  median  usia 
pekerja perusahaan B. 
(D) median  usia  pekerja  perusahaan  A 
lebih  kecil  daripada  rata‐rata  usia 
pekerja perusahaan B. 
(E) rata‐rata,  median,  dan  modus  usia 
pekerja  kedua  perusahaan  terletak 
pada kelas interval yang sama. 
 
14. Jika  0 ≤ x ≤ 2π  dan  0 ≤ y ≤ 2π  memenuhi 
persamaan  sin( x + y) = sin y cos x ,  maka 
cos y sin x = ... 
(A) −1 
(B) −1 
2
(C) 0 
(D) 1 
2
(E) 1 
 
15. Andri  pergi  ke  tempat  kerja  pukul  7.00 
setiap  pagi.  Jika  menggunakan  mobil 
dengan  kecepatan  40  km/jam,  maka  dia 
tiba  di  tempat  kerja  terlambat  10  menit. 
Jika menggunakan mobil dengan kecepatan 
60  km/jam,  maka  dia  tiba  di  tempat  kerja 
20  menit  sebelum  jam  kerja  dimulai.  Jadi, 
jarak antara rumah Andri  dan tempat  kerja 
adalah .... 
(A) 120 km 
(B)   90 km 
(C)   80 km 
(D)   70 km 
(E)   60 km 

an_ay Halaman 3 
www.onlinelesson.org    336 
Simulasi Soal Snmptn 2011 
 
Mata Pelajaran  : Bahasa Indonesia 
Kode Soal    : 336 
 
 
Gunakan  PETUNJUK  A  untuk  menjawab  soal  Pemerintah  secara  simbolis,  telah  menerima 
nomor 16 sampai dengan nomor 30!  tiga  sertifikat  dari  United  Nations 
  Educational,  Scientific,  and  Cultural 
16. Fisika  adalah  sains  atau  ilmu  tentang  alam  Organization  (UNESCO).  Sertifikat  itu 
dalam  arti  luas.  Fisika  ...  gejala  alam  yang  merupakan  simbol  pengakuan  warisan 
tidak  hidup  atau  materi  dalam  lingkup  ruang  budaya  Indonesia  dalam  daftar  The 
dan  waktu.  Fisikawan  mempelajari  ...  dan  Representative  List  of  the  Intangible  Culture 
sifat  materi  dalam  bidang  yang  sangat  Heritage  of  Humanity.  “Sertifikat  itu 
beragam,  mulai  dari  partikel  submikroskopis  mengakui wayang Indonesia, batik Indonesia, 
yang  ...  segala  materi  hingga  perilaku  materi  dan  keris  Indonesia  sebagai  warisan  budaya 
alam semesta sebagai satu kesatuan kosmos.  dunia dan kita harus bangga karenanya,” ujar 
Beberapa  sifat  yang  ...  dalam  fisika  Menko  Kesra  Agung  Laksono  dalam  serah 
merupakan  sifat  yang  ada  dalam  semua  terima sertifikat yang dilakukan di kantornya, 
sistem  materi  yang  ada,  seperti  hukum  Jalan  Merdeka  Barat  Jakarta,  tanggal  5 
kekekalan energi. Sifat semacam ini ... hukum  Februari 2010.  
fisika.    
  Pernyataan  berikut  yang  sesuai  dengan  isi 
Urutan  kata  yang  tepat  untuk  melengkapi  teks di atas adalah ... 
teks di atas adalah ...  (A) Wayang,  batik,  dan  keris  yang  telah 
(A) mempelajari,  perilaku,  membentuk,  diakui  UNESCO  sebagai  warisan 
dipelajari, disebut.   kebudayaan  dunia  hanya  ada  di 
(B) menguraikan,  kegiatan,  membentuk,  Indonesia.  
dinyatakan, diulas.   (B) Sertifikat  UNESCO  atas  warisan 
(C) mempelajari,  aktivitas,  menjadikan,  kebudayaan  Indonesia  merupakan 
diulas, dipelajari.   pengakuan  atas  peradaban  bangsa 
(D) membahas,  sikap,  membuat,  dibahas,  Indonesia  dalam  kancah  kebudayaan 
disebut.   dunia.  
(E) menjelaskan,  perilaku,  mengubah,  (C) Sertifikat  UNESCO  tentang  pengakuan 
dikatakan, dipelajari.   warisan  budaya  Indonesia  sebagai 
  warisan  kebudayaan  dunia  diserahkan 
  oleh Menko Kesra.  
  (D) Masyarakat  Indonesia  bangga  pada 
  wayang,  batik,  dan  keris  sebagai 
  warisan kebudayaannya.  
  (E) Sertifikat  UNESCO  diharapkan  menjadi 
  simbol  pengakuan  warisan  budaya 
  Indonesia  sebagai  bagian  warisan 
  kebudayaan Indonesia.  
   
   
   
   
   
   
  18. Komite  Darurat  Organisasi  Kesehatan  Dunia 

an_ay
17. Pengakuan  dunia  internasional  terhadap  akan meninjau kembali status pandemi H1N1 
warisan  budaya  nasional  semakin  mantap.  untuk  memutuskan  apakah  dunia  telah 

Halaman 1 
www.onlinelesson.org    336 
berpindah  ke  fase  pascapuncak  atau  belum.  Kalimat utama paragraf di atas adalah ... 
Hilangnya H1N1 di Kanada disebabkan sekitar  (A) kalimat 1.  
45%  warga  Kanada  telah  divaksinasi,  30%  (B) kalimat 2.  
didiagnosis  kebal,  dan  hanya  sedikit  orang  (C) kalimat 3.  
kemungkinan  tertulari.  Hal  tersebut  (D) kalimat 4.  
membuat  penyebaran  virus  semakin  sulit  (E) kalimat 6.  
terjadi. Akan tetapi, virus masih menyebar di   
bebera[a  bagian  Eropa  Timur  dan  Asia  20. Belakangan  ini  berbagai  situs  jejaring  sosial, 
Tengah.  WHO  mengatakan  bahwa  ada  misalnya  facebook,  twitter,  my  space, 
aktivitas virus di negara‐negara Afrika, seperti  multiply,  plurk,  Hi5,  dan  tagged  mewabah  di 
Senegal  dan  Mauritania.  Untuk  membantu  Indonesia.  Keberadaan  situs‐situs  jejaring 
badan  internasional  yang  berbasis  di  Jenewa  sosial  itu  sempat  menuai  kritik.  Situs‐situs 
itu,  Kanada  menyumbangkan  lima  juta  dosis  jejaring  sosial  tersebut  dianggap  kurang 
vaksin untuk didistribusikan ke negara‐negara  mendidik  dan  menyebabkan  kecanduan 
berkembang  yang  tidak  mampu  penggunanya. Seringkali karena berbagai hal, 
menyediakannya.   orang menjadi lupa waktu setelah mengakses 
  situs jejaring sosial. Hal negatif lainnya adalah 
Pernyataan berikut yang tidak sesuai dengan  pengguna situs jejaring sosial itu dinilai dapat 
isi paragraf di atas adalah ...  terasing dari kehidupan nyata kesehariannya. 
(A) Penyebaran  virus  H1N1  menjadi  sulit  Oleh  sebab  itu,  diperlukan  kesadaran  dan 
terjadi di Kanada antara lain disebabkan  kontrol  diri  yang  baik  sehingga  pengguna 
30%  penduduk  diduga  kebar  virus  dapat  terhindar  dari  berbagai  efek  negatif 
tersebut.   situs‐situs jejaring sosial tersebut.  
(B) Hilangnya  virus  H1N1  di  Kanada  salah   
satunya  disebabkan  vaksinasi  yang  Ide pokok paragraf di atas adalah ... 
dilakukan kepada sekitar 45% warga.   (A) Situs  jejaring  sosial  yang  sedang 
(C) Kanada  menyumbang  lima  juta  dosis  mewabah  di  Indonesia  sempat  menuai 
vaksin  kepada  WHO  untuk  membantu  banyak kritik.  
negara‐negara berkembang.   (B) Situs jejaring sosial dapat menyebabkan 
(D) Penyebaran  virus  H1N1  saat  ini  masih  kecanduan bagi para penggunanya.  
terjadi  di  seluruh  negara  bagian  Eropa  (C) Facebook,  Twitter,  My  Space,  Multiply, 
Timur dan Asia Tengah.   Plurk,  dan  lainnya  mewabah  di 
(E) Di negara‐negara Afrika, seperti Senegal  Indonesia.  
dan  Mauritania,  masih  ada  aktivitas  (D) Para  pengguna  situs  jejaring  sosial 
virus H1N1.   dapat  terasing  dari  kehidupan  nyata 
  kesehariannya.  
19. (1) Salah satu akibat aturan pasar bebas yang  (E) Situs  jejaring  sosial  berdampak  negatif 
paling  mencolok  adalah  dibatasinya  jumlah  sehingga  diperlukan  kontrol  diri  yang 
order (pesanan) oleh grosir dan pelanggan di  baik.  
beberapa  daerah,  seperti  Surabaya,  Malang,   
Bali,  Kalimantan,  Jawa  Tengah,  dan  Jakarta.   
(2)  “Pembatasan  order  itu  kini  mencapai  50   
persen.  (3)  Mereka  (pelanggan  dan  grosir)   
menyatakan  tidak  ingin  gegabah  memesan   
barang  dalam  jumlah  besar  seperti  dulu,”   
ungkap  Emru,  perajin  sepatu  fashion  dan   
mayoret.  (4)  Menurut  Amru,  pembatasan   
jumlah  order  tersebut,  sebelumnya,  tidak   
diketahui alasan pastinya. (5) Sebab, sebagian   
perajin  banyak  yang  belum  mengetahui   
adanya  perdagangan  bebas  itu.  (6)  Jadi,  21. Sebagai  negara  berkembang,  Indonesia 

an_ay
produksi perajin sepatu menurun drastis, dari  sangat  rentan  terhadap  impor  limbah  B‐3 
50 kodi menjadi hanya 25 kodi.   (bahan  berbahaya  dan  beracun),  khususnya 
  e‐waste  (e;ectronic  waste),  seperti  telepon 

Halaman 2 
www.onlinelesson.org    336 
seluler  dan  komputer  dari  negara‐negara  2008  285.000  286.000  305.000  260.000 
 
maju. Minimnya pengawasan di pintu  masuk  Pernyataan  berikut  yang  sesuai  dengan  isi 
dan  belum  adanya  regulasi  khusus  tentang  tabel di atas adalah ... 
penangan  e‐waste  ini  menjadi  kendala  bagi  (A) poduksi  tebu  setiap  daerah  selalu 
pemerintah  dalam  menanggulangi  masalah  meningkat setiap tahun.  
ini.  Selama  ini,  negara‐negara  berkembang,  (B) persentase  kenaikan  produksi  tebu 
termasuk  Indonesia,  menjadi  salah  satu  daerah B paling rendah.  
importir  terbesar  e‐waste  karena  tidak  (C) persentase  kenaikan  produksi  tebu 
mampu  mengatur  regulasi  tersebut.  Deputi  daerah D paling tinggi.  
Kementrian  Lingkungan  Hidup  Bidang  (D) persentase  kenaikan  produksi  tebu 
Pengelolaan  Bahan  Berbahaya  Beracun,  daerah C paling tinggi.  
Imam  Hendargo  Abu  Ismoyo  mengatakan  (E) persentase  kenaikan  produksi  tebu 
bahwa  permasalahan  e‐waste  semakin  lama  daerah D paling rendah.  
semakin  rumit.  Oleh  sebab  itu,  kita  hrus   
memiliki regulasi khusus soal itu.   23. Menurut  para  ahli  mengatakan  pengambilan 
  data  penelitian  yang  dilakukan  berulang‐
Rangkuman yang tepat untuk paragraf di atas  ulang  kali  dapat  mempermudah  peneliti 
adalah ...  dalam  melakukan  analisis  data  pasca 
(A) Permasalahan  e‐waste  ini  mendapat  pengumpulan data.  
perhatian serius dari Deputi Kementrian   
Lingkungan  Hidup  Bidang  Pengelolaan  Kalimat  tidak  efektif  di  atas  dapat  diperbaiki 
Bahan  Berbahaya  Beracun  dan  Limbah  dengan cara berikut, kecuali ... 
Bahan Berbahaya dan Beracun.   (A) kata menurut dihilangkan.  
(B) Indonesia  sebagai  negara  berkembang  (B) kata  berulang  kali  diganti  dengan 
sangat rentan terhadap impor limbah B‐ berulang‐ulang. 
3  (bahan  berbahaya  dan  beracun),  (C) setelah  kata  ahli  ditambahkan  tanda 
khususnya  e‐waste  (electronic  waste)  koma.  
dari negara‐negara maju.   (D) kata  bahwa  ditambahkan  setelah  kata 
(C) Negara‐negara  berkembang,  termasuk  mengatakan.  
Indonesia  menjadi  salah  satu  importir  (E) pasca  pengumpulan  dituliskan 
terbesar  e‐waste  karena  tidak  mampu  serangkai.  
mengatur  regulasi  penanganan   
pengelolaan limbah B‐3.    
(D) Indonesia  sangat  rentan  terhadap   
limbah  B‐3  karena  masih  minimnya   
pengawasan  serta  belum  adanya   
regulasi  khusus  terhadap  penanganan   
e‐waste sehingga diperlukan regulasi.    
(E) Permasalahan  pengelolaan  bahan   
berbahaya  beracun  dan  limbah  bahan   
berbahaya  dan  beracun  di  Indonesia   
semakin  berkembang  sehingga  harus   
ada regulasi khusus soal itu.    
   
   
   
   
   
  24. Berikut  adalah  identitas  pustaka  yang  dapat 
  digunakan  sebagai  sumber  untuk  menulis 
22. Bacalah tabel berikut dengan cermat!  karangan.  

an_ay
 
Tahun  Daerah A  Daerah B  Daerah C  Daerah D   
Judul  Penulis  Kota  Penerbit  Tahun 
2006  275.000  275.000  285.000  240.000 
Pendidikan  Rahmini P.  Surabaya   Bintang   2008 
2007  280.000  275.000  295.000  249.000  Anak Jalanan 

Halaman 3 
www.onlinelesson.org    336 
Memanusiakan  P. Puspitasari  Medan   Cahaya   2007  menunjukkan  bahwa  semua  pihak  berharap 
Anak Manusia 
Perdagangan  Ahmad  Ambon  Cemerlang   2008 
agar  pelaksanaan  ujian  nasional  tahun  ini 
Anak   Sugentar  berkualitas.  Orang  tua  berharap  agar 
Bimbing  Anak  Dr.  Siana  Surabaya   Gemerlap   2009  pelaksanaan ujian nasional tahun ini berjalan 
Berakhlak Mulia  Rohe 
  lancar  sehingga  anak‐anak  mereka  dapat 
Azhari  menulis  daftar  pustaka  untuk  lulus.  Pemerintah  menghendaki  agar 
karangannya  yang  berjudul  Pembinaan  Anak  pelaksanaan  ujian  nasional  tahun  ini 
Jalanan.  Berdasarkan  data  pustaka  di  atas,  didukung penuh oleh semua pihak.  
penulisan  daftar  pustaka  yang  relevan  dan   
tepat untuk karangan Azhari tersebut adalah  Agar  menjadi  paragraf  uang  baik,  kalimat 
...  penutup yang sesuai adalah ... 
(A) Ahmad  Sugentar.  2008.  Perdagangan  (A) Dengan demikian, pemerintah berharap 
Anak. Ambon: Cemerlang.  agar  ujian  nasional  tahun  ini  jauh  lebih 
P.Puspitasari.  2007.  Memanusiakan  berkualitas  dibandingkan  dengan  ujian 
Anak Manusia. Medan: Cahaya.   nasional  tahun  lalu  yang  penuh 
Rahmini  P.  2008.  Pendidikan  Anak  kecurangan.  
Jalanan. Surabaya: Bintang.   (B) Oleh  karena  itu,  pemerintah  juga 
Siana  Rohe.  2009.  Bimbing  Anak  berharap  agar  ujian  nasional  tahun  ini 
Berakhlak  Mulia.  Surabaya:  jauh  lebih  berkualitas  dibandingkan 
Gemerlap.  dengan  ujian  nasional  tahun  lalu  yang 
(B) Puspitasari,  P.  2007.  Memanusiakan  penuh kecurangan.  
Anak Manusia. Medan: Cahaya.   (C) Oleh  pemerintah  diharapkan  agar  ujian 
Rahmini  P.  2008.  Pendidikan  Anak  nasional tahun ini jauh lebih berkualitas 
Jalanan. Surabaya: Bintang.   dibandingkan  dengan  ujian  nasional 
Rohe, S. 2009. Bimbing Anak Berakhlak  tahun lalu yang penuh kecurangan.  
Mulia. Surabaya: Gemerlap.  (D) Pemerintah  berharap  agar  ujian 
Sugentar,  Ahmad.  2008.  Perdagangan  nasional tahun ini jauh lebih berkualitas 
Anak. Ambon: Cemerlang.  dibandingkan  dengan  ujian  nasional 
(C) Puspitasari,  P.  2007.  Memanusiakan  tahun lalu karena penuh kecurangan.  
Anak Manusia. Medan: Cahaya.   (E) Pemerintah  juga  berharap  agar  ujian 
Rahmini  P.  2008.  Pendidikan  Anak  nasional tahun ini jauh lebih berkualitas 
Jalanan. Surabaya: Bintang.   dibandingkan  dengan  ujian  nasional 
Rohe,  Siana.  2009.  Bimbing  Anak  tahun lalu yang penuh kecurangan.  
Berakhlak  Mulia.  Surabaya:   
Gemerlap.   
(D) Puspitasari,  P.  2007.  Memanusiakan   
Anak Manusia. Medan: Cahaya.    
Rahmini  P.  2008.  Pendidikan  Anak   
Jalanan. Surabaya: Bintang.   
Rohe, S. 2009. Bimbing Anak Berakhlak   
Mulia. Surabaya: Gemerlap.   
(E) Rahmini  P.  2008.  Pendidikan  Anak   
Jalanan. Surabaya: Bintang.   
Puspitasari,  P.  2007.  Memanusiakan   
Anak Manusia. Medan: Cahaya.   
Sugentar,  Ahmad.  2008.  Perdagangan   
Anak. Ambon: Cemerlang.  26. Untuk  mengembangkan  topik  pembinaan 
Rohe,  Siana.  2009.  Bimbing  Anak  keamanan  lingkungan  RT  menjadi  tulisan 
Berakhlak  Mulia.  Surabaya:  yang  runtut,  kerangka  karangan  yang  baik 
Gemerlap.   adalah ... 
  (A) sasaran  pembinaan  keamanan 

an_ay
25. Ujian  nasional  menjadi  perhatian  bnayak  lingkungan  RT,  pentingnya  pembinaan 
pihak,  baik  pemerintah,  orang  tua,  maupun  keamanan  lingkungan  RT,  langkah 
masyarakat.  Tingginya  perhatian  tersebut  pembinaan  keamanan  lingkungan  RT, 

Halaman 4 
www.onlinelesson.org    336 
bentuk  pembinaan  keamanan 
lingkungan RT.  
(B) pentingnya  pembinaan  keamanan 
lingkungan  RT,  sasaran  pembinaan 
keamanan  lingkungan  RT,  bentuk 
pembinaan  keamanan  lingkungan  RT, 
langkah  pembinaan  keamanan 
lingkungan RT.  
(C) pentingnya  pembinaan  keamanan   
lingkungan  RT,  langkah  pembinaan  Pernyataan  berikut  yang  sesuai  dengan  isi 
keamanan  lingkungan    RT,  bentuk  diagram di atas adalah dari tahun ke tahun ... 
pembinaan  keamanan  lingkungan  RT,  (A) perlu  peningkatan  penyuluhan 
sasaran  pembinaan  keamanan  pertanian  untuk  petani  semangka 
lingkungan RT.   karena  jumlah  produksi  semangka 
(D) bentuk  pembinaan  keamanan  merosot.  
lingkungan  RT,  sasaran  pembinaan  (B) perlu  pembinaan  kepada  para  petani 
keamanan  lingkungan  RT,  pentingnya  mangga  karena  jumlah  produksi 
pembinaan  keamanan  lingkungan  RT,  mangga semakin merosot.  
langkah  pembinaan  keamanan  (C) petani  anggur  hendaknya  dibina  lebih 
lingkungan RT.   intensif  karena  jumlah  produksi  anggur 
(E) Pentingnya  pembinaan  keamanan  paling rendah.  
lingkungan  RT,  langkah  pembinaan  (D) petani  anggur  hendaknya  dibina  lebih 
keamanan  lingkungan  RT,  sasaran  intensif  karena  jumlah  produksi  anggur 
pembinaan  keamanan  lingkungan  RT,  cenderung menurun.  
bentuk  pembinaan  keamanan  (E) perlu  pengintensifan  kembali  program 
lingkungan RT.   pertanian karena jumlah produksi buah‐
  buahan  terbanyak  hanya  terjadi  pada 
27. Baru‐baru  ini,  tim  peneliti  dari  University  2006. 
School  of  Medicine,  Washington,  berhasil   
mengidentifikasi  protein  plasmodium  sp,   
yaitu  parasit  nyamuk  yang  menyebabkan   
penyakit  malaria.  Penemuan  ini  penting   
karena  malaria  kini  sebagai  pembunuh   
nomor  tiga  di  dunia.  Penyakit  ini  tercatat   
membunuh satu hingga tiga juta orang setiap   
tahun.    
Paragraf di atas dapat diperbaiki dengan cara   
berikut, kecuali ...   
(A) kata ini diganti itu.   
(B) kata tim ditulis Tim.    
(C) kata dari dihilangkan.    
(D) kata plasmodium sp dicetak miring.   
(E) tanda  koma  setelah  Washington   
dihilangkan.   
   
   
28. Bacalah diagram berikut dengan teliti!   
 
 
Diagram  Perkembangan  Jumlah  Produksi   
Buah‐buahan  di  Kecamatan  Panca  Indah  29. Dalam  buku  Jangan  Serahkan  anak  kepada 
(dalam Ton)  Pembantu  halaman  11  karya  Imron  Zamani 
yang  terbit  tahun  2008  terdapat  kutipan 

an_ay
berikut.  
 

Halaman 5 
www.onlinelesson.org    336 
“Anak,  pada  hakikatnya,  titipan  Tuhan.  kerja,  sikap,  dan  perilaku  seseorang 
Sebagai  pemegang  titipan,  tidak  selayaknya  dalam kehidupan sehari‐hari.  
orang tua menitipkan lagi sang anak keapda  (D) Banyak  orang  setuju  motifasi  itu  bagai 
pembantu”  misteri,  karena  motifasi  itu  tidak 
(Nurudin, 2007)  tampak  tetapi  berpengaruh  pad 
Jika  Nunung  mengutip  pendapat  Nurudin  performance  kerja,  sikap  dan  perilaku 
yang  dikutip  dalam  buku  Imron  Zamani,  seseorang dalam kehidupan sehari‐hari.  
penulisan kutipan yang benar adalah ...  (E) Banyak  orang  setuju  motivasi  itu  bagai 
(A) Nurudin  (dalam  Zamani,  2008:11)  misteri  karena  motivasi  itu  tidak 
menyatakan  bahwa  sebagai  pemegang  tampak,  tetapi  berpengaruh  pad 
titipan,  tidak  selayaknya  orang  tua  performance  kerja,  sikap,  dan  perilaku 
menitipkan  lagi  sang  anak  kepada  seseorang dalam kehidupan sehari‐hari.  
pembantu.    
(B) Nurudin  (Imron  Zamani,  2008:11)   
menyatakan  bahwa  sebagai  pemegang   
titipan,  tidak  selayaknya  orang  tua 
menitipkan  lagi  sang  anak  kepada 
pembantu.  
(C) Nurudin  (2007)  menyatakan  bahwa 
sebagai  pemegang  titipan,  tidak 
selayaknya  orang  tua  menitipkan  lagi 
sang  anak  kepada  pembantu  (dalam 
Zamani, 2008) 
(D) Sebagai  pemegang  titipan,  tidak 
selayaknya  orang  tua  menitipkan  lagi 
sang  anak  kepada  pembantu  (Nurudin, 
2007). 
(E) Menurut  Nurudin  dalam  bukunya 
Zamani  (2008)  menyatakan  bahwa 
sebagai  pemegang  titipan,  tidak 
selayaknya  orang  tua  menitipkan  lagi 
sang anak kepada pembantu.  
 
30. Banyak orang setuju motifasi itu bagai misteri 
karena  motifasi  itu  tidak  tampak  tetapi 
berpengaruh  pada  performance  kerja,  sikap, 
dan  perilaku  seseorang  dalam  kehidupan 
sehari‐hari.  
 
Perbaikan  ejaan  kalimat  di  atas  yang  tepat 
adalah ... 
(A) Banyak  orang  setuju  motifasi  itu  bagai 
misteri,  karena  motifasi  itu  tidak 
tampak  tetapi  berpengaruh  pad 
performance  kerja,  sikap  dan  perilaku 
seseorang dalam kehidupan sehari‐hari.  
(B) Banyak  orang  setuju  motifasi  itu  bagai 
misteri karena motifasi itu tidak tampak 
tetapi  berpengaruh  pad  performance 
kerja,  sikap  dan  perilaku  seseorang 
dalam kehidupan sehari‐hari.  

an_ay
(C) Banyak  orang  setuju  motivasi  itu  bagai 
misteri karena motivasi itu tidak tampak 
tetapi  berpengaruh  pada  performance 

Halaman 6 
www.onlinelesson.org     336 
Simulasi Soal Snmptn 2011 
 
Mata Pelajaran  : Bahasa Inggris 
Kode Soal    : 336 
 
 
Gunakan PETUNJUK A untuk menjawab soal nomor 31 sampai dengan nomor 45! 
   
Text 1 
 
The first ancient DNA sequences to be gathered ‐ 3400 base pairs from a 2400‐year‐old Egyptian mummy    − 
were a proof of principle. A full genome sequence would be far more informative  − perhaps explaining what 
killed King Tut, for instance. At present, Inuk's is the only published ancient human genome. However, 
a  team  led  by  Svante  Paabo  and  Ed  Green  at  the  Max  Planck  Institute  for  Evolutionary  Anthropology  in 
5  Leipzig,  Germany,  will  soon  publish  the  complete  genome  sequence  combined  together  from  several 
Neanderthals, from between 38,000 and 70,000 years ago. 
Neanderthals  are  not  the  only  hominids  whose  genomes  could  be  sequenced,  says  Willerslev.  Homo 
erectus,  a  species  that  emerged  in  Africa  about  2  million  years  ago,  survived  in  east  Asia  until  less  than 
100,000  years  ago.  If  well‐preserved  bones  can  be  found,  a  genome  might  be  possible,  Willerslev  says. 
10 Willerslev's  laboratory  has  just  received  bones  from  Spain  belonging  to  Homo  heidelbergensis,  the 
predecessor  to  Neanderthals.  "We  are  basically  starting  on  it  right  now,"  he  says.  If  these  genomes  ever 
materialize ‐ and that's a big if ‐ they could lead to a better understanding of how different hominid species  
are related, and when and where they branched off. If the genetic information is good enough, it may tell us 
something about the nature of past peoples − possibly even what they looked like. Ancient human genomes 
15  could give us insights into the evolution of our own species, explaining when genes involved in disease and 
higher cognitive skills emerged. 
But  DNA  is  not  forever.  As  it  ages,  its  long  strands  shred  into  ever  smaller  pieces.  Eventually  they 
become too small to reassemble, and all information is lost. "There seems to be a time horizon of 100,000 
years  or  so  under  most  preservation  conditions  during  which  intact  DNA  survives,"  Green  says.  Stephan 
20  Schuster  at  Pennsylvania  State  University,  who  led  the  woolly  mammoth  genome  project,  thinks  ancient 
genomics  is  already  plateauing.  Large  chunks  of  Inuk's  genome  couldn't  be  filled  in  because  his  DNA  had 
crumbled into small pieces. "We will face an uphill battle in trying to apply this to a large number of human 
remains," he says. 
 
31. With  reference  to  the  whole  text,  the  writer  (E) requiring  a  longer  chain  of  complete 
mainly deals with the topic on …  DNA sequences.  
(A) DNA research on the Neanderthals.   
(B) the past life of the Neanderthals.  33. The physical look of hominid species can even 
(C) the use of ancient DNA in anthropology.  be  reconstructed  using  the  DNA  technology 
(D) the role of DNA in fossil studies.  under the condition that … 
(E) DNA  engineering  in  the  Max  Planck  (A) the  DNA  forms  a  solid  fossil  in  a  good 
Institute.   shape.  
  (B) the  genetic  information  in  the  DNA  is 
32. The  writer  is  mainly  of  the  opinion  that  high‐quality.  
tracing ancient humans’ life using their DNA …  (C) the DNA had not been smaller in chunks.  
(A) is technologically possible through it has  (D) the  sequence  of  the  DNA  pieces  is 
natural challenges.  systematic.  
(B) promises  a  new  horizon  of  (E) the  DNA  sequences  can  be  easily 
understanding past illnesses.  reassembled.  
(C) is  a  new  breakthrough  in  modern   
anthropological studies.  34. Based on the text, the following would be the 

an_ay
(D) provides a better picture of old peoples’  kind  of  information  that  could  be  revealed 
DNA structures. 

Halaman 1 
www.onlinelesson.org     336 
about  ancient  people  through  modern  empirically  validated,  if  contemporary  DNA 
genome analyses, EXCEPT …  analyses  are  supported  by  the  following 
(A) typical illnesses.  factors, EXCEPT … 
(B) thinking potentials.  (A) more discovery of early human remains.  
(C) evolutionary phases.   (B) modern genomic laboratories.  
(D) migratory directions.   (C) expertise associated with ancient DNA.  
(E) nutritional patterns.   (D) better  management    in    fossil 
  excavation.  
  (E) availability  of  complete  ancient  human 
  genomes.  
35. Based  on  the  text,  soon  Darwin’s  human   
evolution  theory  will  be  most  likely 
Text 2 
 
John Apollos is losing weight the old‐fashioned way − by eating less. A whole lot less. As a volunteer in the two‐
year  Comprehensive  Assessment  of  Long‐Term  Effects  of  Reducing  Intake  of  Energy  (CALERIE)  study  at  Tufts 
University  in  Boston,  Apollos  has  lowered  his  daily  calorie  intake  25%  over  the  past  eight  months.  The  fat,  not 
surprisingly,  has  melted  away;  the  52‐year‐old  physical  trainer  has  lost  more  than  11  kg  since  the  study  began  
5  and is down to his high school weight. 
Yet,  that's  not  the  real  reason  Apollos  and  the  other  participants  in  the  program  are  eating  only  three‐ 
quarters  of  what  they  used  to.  The  researchers  running  the  multicenter  CALERIE  study  are  trying  to  determine 
whether restricting food intake can slow the aging process and extend our life span. "I feel better and lighter and 
healthier,"  says  Apollos.  "But  if  it  could  help  you  live  longer,  that  would  be  pretty  amazing."  The  idea  is 
10  counterintuitive:  If  we  eat  to  live,  how  can  starving  ourselves  add  years  to  our  lives?  Yet.  decades  of  calorie‐ 
restriction  studies  involving  organisms  ranging  from  microscopic  yeast  to  rats  have  shown  just  that,  extending   
the  life  spans  of  the  semi  starved  as  much  as  50%.  Last  July  a  long‐term  study  led  by  researchers  at  the    
University of Wisconsin nudged the implications of this a bit closer to our species, finding that calorie restriction 
seemed  to  extend  the  lives  of  humanlike  rhesus  monkeys  as  well.  The  hungry  primates  fell  victim  to  diabetes, 
15  heart and brain disease and cancer much less frequently than their well‐fed counterparts did. 
However,  there  may  be  more  than  just  the  absence  of  disease  operating  here.  Anytime  you  go  on  a  diet, 
after  all,  you  stand  a  good  chance  of  lowering  your  blood  pressure,  cholesterol  level  and  risk  of  diabetes  and  
other health woes. All that can translate into extra years. With calorie restriction, usually defined as a diet with 
25% to 30% fewer calories than normal but still containing essential nutrients, something else appears to be at 
20  work to extend longevity. 
 
36. Which  of  the  following  ideas  from  the  text  (C) reducing  the  amount  of  extreme  fat 
above contains an opinion?  accumulation.  
(A) Restricting  calorie  consumption  for  (D) controlling  blood  pressure  and 
longevity.   cholesterol levels.  
(B) Absence  of  disease  due  to  controlling  (E) aging‐process  slowing  and  life  span 
calorie intake.   extension.  
(C) Apollo’s  feeling  better  and  lighter  and   
healthier.   38. If the information in the text is true, the risks 
(D) The  melting  of  body  fat  by  having  less  that  someone  whose  calorie  consumption  is 
foods.   controlled up to the portion suggested in the 
(E) Apollos’  restricting  his  daily  calorie  study suffers from bone cancer are … 
consumption.  (A) substantial.  
  (B) negligible.  
37. The  study  aims  at  evaluating  the  impact  of  (C) unpredictable.  
calorie restriction on …  (D) serious.  
(A) minimizing  the  risks  of  getting  serious  (E) indefinite.  
illnesses.    

an_ay
(B) enhancing  the  feeling  of  happiness  and   
health.    

Halaman 2 
www.onlinelesson.org     336 
39. The  following  is  among  other  things  the  40. As  mentioned  in  the  text,  the  study  held  at 
empirical  impacts  of  the  study  mentioned  in  the  multicenter  CALERIE,  Tufts  University  in 
the text, EXCEPT …  Boston has employed a research method that 
(A) slender body shape.   seems to be … 
(B) long life expectancy.   (A) conventional.  
(C) good healthiness.   (B) complicated.  
(D) emotion stability.   (C) temporary.  
(E) good feelings.   (D) ultramodern.  
  (E) methodological.  
   
 
Text 3 
 
Further research, conducted by Dean Mobbs, then at Stanford University in California, uncovered a second 
point  of  activity  in  the  brain's  limbic  system  ‐  associated  with  dopamine  release  and  reward  processing  ‐ 
which  may  explain  the  pleasure  felt  once  you  "get"  the  joke.  Examining  one  particular  part  of  the  limbic 
system‐the ventral striatum‐was especially revealing, as its level of activity corresponded with the perceived 
funniness of a joke. "It's the same region that is involved in many different types of reward, from drugs, to 
sex and our favourite music," says Mobbs, now at the MRC Cognition and Brain Sciences Unit in Cambridge, 
UK. "Humour thus taps into basic rewards systems that are important to our survival." 
Yet humour is a far more multifaceted process than primeval pleasures like food. In addition to the two 
core processes of getting  the joke and  feeling good  about it, jokes also activate regions of the frontal and 
cingulate cortex, which are linked with association formation, learning and decision‐making. The team also 
found heightened activity in the anterior cingulate cortex and the frontoinsular cortex ‐ regions that are only 
present in humans and, in a less developed form, great apes. Indeed, the fact that these regions are involved 
suggests  that  humour  is  an  advanced  ability  which  may  have  only  evolved  in  early  humans,  says  Watson, 
who conducted the research. 
No two brains are the same, however, and how these differences are reflected in our sense of humour 
is  the  subject  of  much  research.  Men  and  women,  for  example,  seem  to  process  jokes  slightly  differently. 
Although  both  sexes  laugh  at  roughly  the  same  number  of  jokes,  women  show  greater  activity  in  the  left 
prefrontal  cortex  than  men.  "This  suggests  a  greater  degree  of  executive  processing  and  language‐based 
decoding," says Mobbs. As a result, women take significantly much longer than men to decide whether they 
find something funny, though that does not seem to spoil their enjoyment of the joke. Indeed, women show 
a greater response in the limbic system than men, suggesting they feel a greater sense of reward. 
 
41. Which of the following statements in the text  42. The  part  that  comes  before  the  text  would 
above contains an opinion?  most likely describe … 
(A) Basic  rewards  systems  associated  with  (A) Mobbs’  same  study  conducted  on 
humor  are  vital  to  human  continued  humans’  brain  limbic  system 
existence.   sensitiveness.  
(B) The  anterior  cingulate  cortex  and  the  (B) another  activity  found  in  the  brain’s 
frontoinsular  cortex  regions  belong  to  limbic system of Mobbs’ different study.  
humans.   (C) a  researcher’s  study  other  than  Mobbs’ 
(C) Women’s  limbic  system  reacts  more  on  another  activity  found  in  the  brain 
responsively  than  men’s  does  to  a  limbic system.  
humor stimulus.   (D) the importance of conducting a study on 
(D) Different types of reward like drugs and  responses  by  humans’  brain  limbic 
our  favorite  music  are  linked  to  the  system.  
ventral striatum.   (E) the  timely  reaction  of  humans’  brain 
(E) Joke funiness relates with the amount of  limbic system to perception on humor.  
dopamine  release  and  reward   

an_ay
processing.    
   
   

Halaman 3 
www.onlinelesson.org     336 
43. The  scientists  see  that  the  internal  (D)  
mechanisms  in  a  human’s  brain  related  with   
enjoyment in having good meals are …   
(A) more  explainable  than  those  in   
perceiving a joke.    
(B) less observable than pleasures in getting   
a prize.    
(C) as quick as those in seeing funniness of a   
joke.   (E)  
(D) more complex than those in perceiving a   
joke.    
(E) less  clear  than  those  of  enjoyment  in   
winning a prize.    
   
44. Humor is worth in these four facets, EXCEPT …   
(A) experiencing  the  sense  of  the  humor   
itself.    
(B) having good feelings about the humor.    
(C) activating  parts  of  the  frontal  and 
cungulate cortex.  
(D) enhancing  activities  in  two  humans’ 
cortex areas.  
(E) triggering more dynamic responses in all 
cells.  
 
45. Which  chart  below  most  likely  indicates  the 
response time needed by women and men to 
laugh on a joke? 
(A)  
 
 
 
 
 
 
 
 
(B)  
 
 
 
 
 
 
 
 
(C)  
 
 
 
 

an_ay
 
 
 

Halaman 4 
www.onlinelesson.org     734 
Simulasi Soal Snmptn 2011 
 
Mata Pelajaran  : Matematika Dasar 
Kode Soal    : 734 
 
 
Gunakan  PETUNJUK  A  untuk  menjawab  soal  4. Fungsi  f (x) = x 2 + ax   mempunyai  grafik 
nomor 1 sampai dengan nomor 15!  berikut. 
 
1. Pernyataan  yang  mempunyai  nilai 
kebenaran  sama  dengan  pernyataan:  “Jika 
bilangan  ganjil  sama  dengan  bilangan 
genap, maka 1 + 2 bilangan ganjil” adalah ... 
(A) “Bilangan ganjil sama dengan bilangan   
genap dan 1 + 2 bilangan ganjil”   Grafik fungsi  g(x) = x 2 − ax + 5  adalah ... 
(B) “Jika  1  +  2  bilangan  ganjil,  maka  (A)  
bilangan  ganjil  sama  dengan  bilangan   
genap”    
(C) “Jika  bilangan  ganjil  sama  dengan   
bilangan  genap,  maka  1  +  2  bilangan   
genap”   
(D) “Bilangan  ganjil  sama  dengan  bilangan   
genap dan 1 + 2 bilangan genap”    
(E) “Jika  bilangan  ganjil  tidak  sama  (B)  
dengan  bilangan  genap,  maka  1  +  2   
bilangan genap”   
   
2. Jika n memenuhi   
25
144
0 , 25
×4 0 , 25
254 4 ×2
4254 4×4
0 , 25
L4×4
2543 = 125  
0 , 25
 
n faktor  
Maka (n − 3)(n + 2) = ...  (C)  
(A) 36   
(B) 32   
(C) 28   
(D) 26   
(E) 24   
   
3. Persamaan  x 2 − ax − (a + 1) = 0   mempunyai  (D)  
akar‐akar  x1 > 1  dan  x 2 < 1  untuk …   
 
(A) a > 0  
 
(B) a < 0  
 
(C) a ≠ −2    
(D) a > −2    
(E) − 2 < a < 0   (E)  
   
   
   
   
   
   

an_ay
 
 

Halaman 1 
www.onlinelesson.org     734 
5. Nilai  x  yang  memenuhi  pertidaksamaan  10. Jika  fungsi  f ( x, y ) = 5000 − x − y   dengan 
x +1 x syarat  x ≥ 0 ,  y ≥ 0 ,  x − 2 y + 2 ≥ 0 ,  dan 
>  adalah .... 
x +1 x −1 2 x + y − 6 ≥ 0 , maka .... 
(A) x < 1  (A) fungsi  f   mempunyai  nilai  minimum 
(B) x > −1 
dan nilai maksimum 
(C) −1 ≤ x < 1 
(B) fungsi  f   tidak  mempunyai  nilai 
(D) x < −1 atau −1 < x < 1 
minimum maupun nilai maksimum 
(E) x < −1 atau x > 1 
(C) fungsi  f   mempunyai  nilai  minimum 
 
6. Jika M adalah matriks sehingga   dan tidak mempunyai nilai maksimum 
(D) fungsi  f  mempunyai nilai maksimum 
⎛a b⎞ ⎛ a b ⎞
M⎜⎜ ⎟⎟ = ⎜⎜ ⎟⎟   dan tidak mempunyai nilai minimum 
⎝c d⎠ ⎝−a + c − b + d⎠ (E) nilai  minimum  dan  nilai  maksimum 
maka determinan matriks M adalah ....  fungsi  f  tidak dapat ditentukan 
(A) 1 
 
(B) −1 
11. Balok  ABCD.EFGH   mempunyai  panjang 
(C) 0 
rusuk  AB = 4cm ,  BC = 3cm , dan  AE = 3cm . 
(D) −2 
Bidang  AFH  memotong  balok  menjadi  2 
(E) 2 
bagian  dengan  perbandingan  volumenya 
 
adalah .... 
7. Jika penyelesaian sistem persamaan 
(A) 1 : 3 
⎧(a − 2) x + y = 0 (B) 2 : 3 
⎨  
⎩ x + (a − 2) y = 0 (C) 3 : 5 
Tidak  hanya  ( x, y ) = (0,0)   saja,  maka  nilai  (D) 1 : 5 
(E) 1 : 6 
a 2 − 4a + 3 = ....   
(A)   0 
12. Jika  0 ≤ x ≤ 2π  dan  0 ≤ y ≤ 2π  memenuhi 
(B)   1 
(C)   4  persamaan  sin(x + y) = sin y cos x ,  maka 
(D)   9  cos y sin x = ... 
(E) 16  (A) −1 
 
(B) −1 
8. Jika  g(x − 2) = 2 x − 3   dan  2
( fog)(x − 2) = 4 x − 8 x + 3 , maka  f (−3) = .... 
2 (C) 0 
(A) −3  (D) 1 
(B) 0  2
(C) 3  (E) 1 
(D) 12   
(E) 15   
   
9. Jika  −6, a , b, c , d , e , f , g ,18   merupakan   
 
barisan aritmatika, maka  a + d + g = .... 
 
(A) 12   
(B) 18   
(C) 24   
(D) 30   
(E) 36   
   
   
   

an_ay
   
   
 

Halaman 2 
www.onlinelesson.org     734 
13. Distribusi  frekuensi  usia  pekerja  pada 
perusahaan  A  dan  perusahaan  B  diberikan 
pada tabel berikut. 
 

Usia  Banyak Pekerja 
(tahun)  Perusahaan A  Perusahaan B
20 – 29  7  1 
30 – 39  26  8 
40 – 49  15  1 
50 – 59  2  32 
60 – 69  0  8 
Total  50  50 
 

Berdasarkan  data  pada  tabel  tersebut, 


kesimpulan yang tidak benar adalah 
(A) rata‐rata,  median  dan  modus  usia 
pekerja  perusahaan  A  masing‐masing 
lebih  rendah  daripada  rata‐rata, 
median,  dan  modus  usia  pekerja 
perusahaan B. 
(B) rata‐rata  usia  pekerja  perusahaan  A 
lebih  kecil  daripada  median  usia 
pekerja perusahaan B. 
(C) modus  usia  pekerja  perusahaan  A 
lebih  kecil  daripada  median  usia 
pekerja perusahaan B. 
(D) median  usia  pekerja  perusahaan  A 
lebih  kecil  daripada  rata‐rata  usia 
pekerja perusahaan B. 
(E) rata‐rata,  median,  dan  modus  usia 
pekerja  kedua  perusahaan  terletak 
pada kelas interval yang sama. 
 
14. Jika  p < −3  dan  q > 5  , maka nilai  q − p  .... 
(A) lebih besar daripada 9 
(B) lebih besar daripada 7 
(C) lebih kecil daripada 8 
(D) lebih kecil daripada 2 
(E) lebih kecil daripada −2 
 
15. Andri  pergi  ke  tempat  kerja  pukul  7.00 
setiap  pagi.  Jika  menggunakan  mobil 
dengan  kecepatan  40  km/jam,  maka  dia 
tiba  di  tempat  kerja  terlambat  10  menit. 
Jika menggunakan mobil dengan kecepatan 
60  km/jam,  maka  dia  tiba  di  tempat  kerja 
20  menit  sebelum  jam  kerja  dimulai.  Jadi, 
jarak antara rumah Andri  dan tempat  kerja 
adalah .... 
(A) 120 km 
(B)   90 km 
(C)   80 km 

an_ay
(D)   70 km 
(E)   60 km 

Halaman 3 
www.onlinelesson.org    734 
Simulasi Soal Snmptn 2011 
 
Mata Pelajaran  : Bahasa Indonesia 
Kode Soal    : 734 
 
 
Gunakan  PETUNJUK  A  untuk  menjawab  soal  17. Pengakuan  dunia  internasional  terhadap 
nomor 16 sampai dengan nomor 30!  warisan  budaya  nasional  semakin  mantap. 
  Pemerintah  secara  simbolis,  telah  menerima 
16. Komite  Darurat  Organisasi  Kesehatan  Dunia  tiga  sertifikat  dari  United  Nations 
akan meninjau kembali status pandemi H1N1  Educational,  Scientific,  and  Cultural 
untuk  memutuskan  apakah  dunia  telah  Organization  (UNESCO).  Sertifikat  itu 
berpindah  ke  fase  pascapuncak  atau  belum.  merupakan  simbol  pengakuan  warisan 
Hilangnya H1N1 di Kanada disebabkan sekitar  budaya  Indonesia  dalam  daftar  The 
45%  warga  Kanada  telah  divaksinasi,  30%  Representative  List  of  the  Intangible  Culture 
didiagnosis  kebal,  dan  hanya  sedikit  orang  Heritage  of  Humanity.  “Sertifikat  itu 
kemungkinan  tertulari.  Hal  tersebut  mengakui wayang Indonesia, batik Indonesia, 
membuat  penyebaran  virus  semakin  sulit  dan  keris  Indonesia  sebagai  warisan  budaya 
terjadi. Akan tetapi, virus masih menyebar di  dunia dan kita harus bangga karenanya,” ujar 
bebera[a  bagian  Eropa  Timur  dan  Asia  Menko  Kesra  Agung  Laksono  dalam  serah 
Tengah.  WHO  mengatakan  bahwa  ada  terima sertifikat yang dilakukan di kantornya, 
aktivitas virus di negara‐negara Afrika, seperti  Jalan  Merdeka  Barat  Jakarta,  tanggal  5 
Senegal  dan  Mauritania.  Untuk  membantu  Februari 2010.  
badan  internasional  yang  berbasis  di  Jenewa   
itu,  Kanada  menyumbangkan  lima  juta  dosis  Pernyataan  berikut  yang  sesuai  dengan  isi 
vaksin untuk didistribusikan ke negara‐negara  teks di atas adalah ... 
berkembang  yang  tidak  mampu  (A) Wayang,  batik,  dan  keris  yang  telah 
menyediakannya.   diakui  UNESCO  sebagai  warisan 
  kebudayaan  dunia  hanya  ada  di 
Pernyataan berikut yang tidak sesuai dengan  Indonesia.  
isi paragraf di atas adalah ...  (B) Sertifikat  UNESCO  atas  warisan 
(A) Penyebaran  virus  H1N1  menjadi  sulit  kebudayaan  Indonesia  merupakan 
terjadi di Kanada antara lain disebabkan  pengakuan  atas  peradaban  bangsa 
30%  penduduk  diduga  kebar  virus  Indonesia  dalam  kancah  kebudayaan 
tersebut.   dunia.  
(B) Hilangnya  virus  H1N1  di  Kanada  salah  (C) Sertifikat  UNESCO  tentang  pengakuan 
satunya  disebabkan  vaksinasi  yang  warisan  budaya  Indonesia  sebagai 
dilakukan kepada sekitar 45% warga.   warisan  kebudayaan  dunia  diserahkan 
(C) Kanada  menyumbang  lima  juta  dosis  oleh Menko Kesra.  
vaksin  kepada  WHO  untuk  membantu  (D) Masyarakat  Indonesia  bangga  pada 
negara‐negara berkembang.   wayang,  batik,  dan  keris  sebagai 
(D) Penyebaran  virus  H1N1  saat  ini  masih  warisan kebudayaannya.  
terjadi  di  seluruh  negara  bagian  Eropa  (E) Sertifikat  UNESCO  diharapkan  menjadi 
Timur dan Asia Tengah.   simbol  pengakuan  warisan  budaya 
(E) Di negara‐negara Afrika, seperti Senegal  Indonesia  sebagai  bagian  warisan 
dan  Mauritania,  masih  ada  aktivitas  kebudayaan Indonesia.  
virus H1N1.    
   
   
   
   

an_ay
   

Halaman 1 
www.onlinelesson.org    734 
18. Fisika  adalah  sains  atau  ilmu  tentang  alam  (D) Para  pengguna  situs  jejaring  sosial 
dalam  arti  luas.  Fisika  ...  gejala  alam  yang  dapat  terasing  dari  kehidupan  nyata 
tidak  hidup  atau  materi  dalam  lingkup  ruang  kesehariannya.  
dan  waktu.  Fisikawan  mempelajari  ...  dan  (E) Situs  jejaring  sosial  berdampak  negatif 
sifat  materi  dalam  bidang  yang  sangat  sehingga  diperlukan  kontrol  diri  yang 
beragam,  mulai  dari  partikel  submikroskopis  baik.  
yang  ...  segala  materi  hingga  perilaku  materi   
alam semesta sebagai satu kesatuan kosmos.  20. (1) Salah satu akibat aturan pasar bebas yang 
Beberapa  sifat  yang  ...  dalam  fisika  paling  mencolok  adalah  dibatasinya  jumlah 
merupakan  sifat  yang  ada  dalam  semua  order (pesanan) oleh grosir dan pelanggan di 
sistem  materi  yang  ada,  seperti  hukum  beberapa  daerah,  seperti  Surabaya,  Malang, 
kekekalan energi. Sifat semacam ini ... hukum  Bali,  Kalimantan,  Jawa  Tengah,  dan  Jakarta. 
fisika.   (2)  “Pembatasan  order  itu  kini  mencapai  50 
  persen.  (3)  Mereka  (pelanggan  dan  grosir) 
Urutan  kata  yang  tepat  untuk  melengkapi  menyatakan  tidak  ingin  gegabah  memesan 
teks di atas adalah ...  barang  dalam  jumlah  besar  seperti  dulu,” 
(A) mempelajari,  perilaku,  membentuk,  ungkap  Emru,  perajin  sepatu  fashion  dan 
dipelajari, disebut.   mayoret.  (4)  Menurut  Amru,  pembatasan 
(B) menguraikan,  kegiatan,  membentuk,  jumlah  order  tersebut,  sebelumnya,  tidak 
dinyatakan, diulas.   diketahui alasan pastinya. (5) Sebab, sebagian 
(C) mempelajari,  aktivitas,  menjadikan,  perajin  banyak  yang  belum  mengetahui 
diulas, dipelajari.   adanya  perdagangan  bebas  itu.  (6)  Jadi, 
(D) membahas,  sikap,  membuat,  dibahas,  produksi perajin sepatu menurun drastis, dari 
disebut.   50 kodi menjadi hanya 25 kodi.  
(E) menjelaskan,  perilaku,  mengubah,   
dikatakan, dipelajari.   Kalimat utama paragraf di atas adalah ... 
  (A) kalimat 1.  
19. Belakangan  ini  berbagai  situs  jejaring  sosial,  (B) kalimat 2.  
misalnya  facebook,  twitter,  my  space,  (C) kalimat 3.  
multiply,  plurk,  Hi5,  dan  tagged  mewabah  di  (D) kalimat 4.  
Indonesia.  Keberadaan  situs‐situs  jejaring  (E) kalimat 6.  
sosial  itu  sempat  menuai  kritik.  Situs‐situs   
jejaring  sosial  tersebut  dianggap  kurang   
mendidik  dan  menyebabkan  kecanduan   
penggunanya. Seringkali karena berbagai hal,   
orang menjadi lupa waktu setelah mengakses   
situs jejaring sosial. Hal negatif lainnya adalah   
pengguna situs jejaring sosial itu dinilai dapat   
terasing dari kehidupan nyata kesehariannya.   
Oleh  sebab  itu,  diperlukan  kesadaran  dan   
kontrol  diri  yang  baik  sehingga  pengguna   
dapat  terhindar  dari  berbagai  efek  negatif   
situs‐situs jejaring sosial tersebut.    
   
Ide pokok paragraf di atas adalah ...   
(A) Situs  jejaring  sosial  yang  sedang   
mewabah  di  Indonesia  sempat  menuai   
banyak kritik.    
(B) Situs jejaring sosial dapat menyebabkan   
kecanduan bagi para penggunanya.    
(C) Facebook,  Twitter,  My  Space,  Multiply,   

an_ay
Plurk,  dan  lainnya  mewabah  di   
Indonesia.    
 

Halaman 2 
www.onlinelesson.org    734 
21. Sebagai  negara  berkembang,  Indonesia  22. Bacalah tabel berikut dengan cermat! 
 
sangat  rentan  terhadap  impor  limbah  B‐3  Tahun  Daerah A  Daerah B  Daerah C  Daerah D 
(bahan  berbahaya  dan  beracun),  khususnya  2006  275.000  275.000  285.000  240.000 
e‐waste  (e;ectronic  waste),  seperti  telepon  2007  280.000  275.000  295.000  249.000 
seluler  dan  komputer  dari  negara‐negara  2008  285.000  286.000  305.000  260.000 
 
maju. Minimnya pengawasan di pintu  masuk  Pernyataan  berikut  yang  sesuai  dengan  isi 
dan  belum  adanya  regulasi  khusus  tentang  tabel di atas adalah ... 
penangan  e‐waste  ini  menjadi  kendala  bagi  (A) poduksi  tebu  setiap  daerah  selalu 
pemerintah  dalam  menanggulangi  masalah  meningkat setiap tahun.  
ini.  Selama  ini,  negara‐negara  berkembang,  (B) persentase  kenaikan  produksi  tebu 
termasuk  Indonesia,  menjadi  salah  satu  daerah B paling rendah.  
importir  terbesar  e‐waste  karena  tidak  (C) persentase  kenaikan  produksi  tebu 
mampu  mengatur  regulasi  tersebut.  Deputi  daerah D paling tinggi.  
Kementrian  Lingkungan  Hidup  Bidang  (D) persentase  kenaikan  produksi  tebu 
Pengelolaan  Bahan  Berbahaya  Beracun,  daerah C paling tinggi.  
Imam  Hendargo  Abu  Ismoyo  mengatakan  (E) persentase  kenaikan  produksi  tebu 
bahwa  permasalahan  e‐waste  semakin  lama  daerah D paling rendah.  
semakin  rumit.  Oleh  sebab  itu,  kita  hrus   
memiliki regulasi khusus soal itu.   23. Bacalah diagram berikut dengan teliti! 
   

Rangkuman yang tepat untuk paragraf di atas  Diagram  Perkembangan  Jumlah  Produksi 


adalah ...  Buah‐buahan  di  Kecamatan  Panca  Indah 
(A) Permasalahan  e‐waste  ini  mendapat  (dalam Ton) 
perhatian serius dari Deputi Kementrian 
Lingkungan  Hidup  Bidang  Pengelolaan 
Bahan  Berbahaya  Beracun  dan  Limbah 
Bahan Berbahaya dan Beracun.  
(B) Indonesia  sebagai  negara  berkembang 
sangat rentan terhadap impor limbah B‐
3  (bahan  berbahaya  dan  beracun), 
khususnya  e‐waste  (electronic  waste) 
dari negara‐negara maju.    
Pernyataan  berikut  yang  sesuai  dengan  isi 
(C) Negara‐negara  berkembang,  termasuk 
diagram di atas adalah dari tahun ke tahun ... 
Indonesia  menjadi  salah  satu  importir 
(A) perlu  peningkatan  penyuluhan 
terbesar  e‐waste  karena  tidak  mampu 
pertanian  untuk  petani  semangka 
mengatur  regulasi  penanganan 
karena  jumlah  produksi  semangka 
pengelolaan limbah B‐3.  
merosot.  
(D) Indonesia  sangat  rentan  terhadap 
(B) perlu  pembinaan  kepada  para  petani 
limbah  B‐3  karena  masih  minimnya 
mangga  karena  jumlah  produksi 
pengawasan  serta  belum  adanya 
mangga semakin merosot.  
regulasi  khusus  terhadap  penanganan 
(C) petani  anggur  hendaknya  dibina  lebih 
e‐waste sehingga diperlukan regulasi.  
intensif  karena  jumlah  produksi  anggur 
(E) Permasalahan  pengelolaan  bahan 
paling rendah.  
berbahaya  beracun  dan  limbah  bahan 
(D) petani  anggur  hendaknya  dibina  lebih 
berbahaya  dan  beracun  di  Indonesia 
intensif  karena  jumlah  produksi  anggur 
semakin  berkembang  sehingga  harus 
cenderung menurun.  
ada regulasi khusus soal itu.  
(E) perlu  pengintensifan  kembali  program 
 
pertanian karena jumlah produksi buah‐
 
buahan  terbanyak  hanya  terjadi  pada 
 
2006. 
 
 

an_ay
 
 
 
 

Halaman 3 
www.onlinelesson.org    734 
24. Banyak orang setuju motifasi itu bagai misteri  (B) Oleh  karena  itu,  pemerintah  juga 
karena  motifasi  itu  tidak  tampak  tetapi  berharap  agar  ujian  nasional  tahun  ini 
berpengaruh  pada  performance  kerja,  sikap,  jauh  lebih  berkualitas  dibandingkan 
dan  perilaku  seseorang  dalam  kehidupan  dengan  ujian  nasional  tahun  lalu  yang 
sehari‐hari.   penuh kecurangan.  
  (C) Oleh  pemerintah  diharapkan  agar  ujian 
Perbaikan  ejaan  kalimat  di  atas  yang  tepat  nasional tahun ini jauh lebih berkualitas 
adalah ...  dibandingkan  dengan  ujian  nasional 
(A) Banyak  orang  setuju  motifasi  itu  bagai  tahun lalu yang penuh kecurangan.  
misteri,  karena  motifasi  itu  tidak  (D) Pemerintah  berharap  agar  ujian 
tampak  tetapi  berpengaruh  pad  nasional tahun ini jauh lebih berkualitas 
performance  kerja,  sikap  dan  perilaku  dibandingkan  dengan  ujian  nasional 
seseorang dalam kehidupan sehari‐hari.   tahun lalu karena penuh kecurangan.  
(B) Banyak  orang  setuju  motifasi  itu  bagai  (E) Pemerintah  juga  berharap  agar  ujian 
misteri karena motifasi itu tidak tampak  nasional tahun ini jauh lebih berkualitas 
tetapi  berpengaruh  pad  performance  dibandingkan  dengan  ujian  nasional 
kerja,  sikap  dan  perilaku  seseorang  tahun lalu yang penuh kecurangan.  
dalam kehidupan sehari‐hari.    
(C) Banyak  orang  setuju  motivasi  itu  bagai  26. Untuk  mengembangkan  topik  pembinaan 
misteri karena motivasi itu tidak tampak  keamanan  lingkungan  RT  menjadi  tulisan 
tetapi  berpengaruh  pada  performance  yang  runtut,  kerangka  karangan  yang  baik 
kerja,  sikap,  dan  perilaku  seseorang  adalah ... 
dalam kehidupan sehari‐hari.   (A) sasaran  pembinaan  keamanan 
(D) Banyak  orang  setuju  motifasi  itu  bagai  lingkungan  RT,  pentingnya  pembinaan 
misteri,  karena  motifasi  itu  tidak  keamanan  lingkungan  RT,  langkah 
tampak  tetapi  berpengaruh  pad  pembinaan  keamanan  lingkungan  RT, 
performance  kerja,  sikap  dan  perilaku  bentuk  pembinaan  keamanan 
seseorang dalam kehidupan sehari‐hari.   lingkungan RT.  
(E) Banyak  orang  setuju  motivasi  itu  bagai  (B) pentingnya  pembinaan  keamanan 
misteri  karena  motivasi  itu  tidak  lingkungan  RT,  sasaran  pembinaan 
tampak,  tetapi  berpengaruh  pad  keamanan  lingkungan  RT,  bentuk 
performance  kerja,  sikap,  dan  perilaku  pembinaan  keamanan  lingkungan  RT, 
seseorang dalam kehidupan sehari‐hari.   langkah  pembinaan  keamanan 
  lingkungan RT.  
25. Ujian  nasional  menjadi  perhatian  bnayak  (C) pentingnya  pembinaan  keamanan 
pihak,  baik  pemerintah,  orang  tua,  maupun  lingkungan  RT,  langkah  pembinaan 
masyarakat.  Tingginya  perhatian  tersebut  keamanan  lingkungan    RT,  bentuk 
menunjukkan  bahwa  semua  pihak  berharap  pembinaan  keamanan  lingkungan  RT, 
agar  pelaksanaan  ujian  nasional  tahun  ini  sasaran  pembinaan  keamanan 
berkualitas.  Orang  tua  berharap  agar  lingkungan RT.  
pelaksanaan ujian nasional tahun ini berjalan  (D) bentuk  pembinaan  keamanan 
lancar  sehingga  anak‐anak  mereka  dapat  lingkungan  RT,  sasaran  pembinaan 
lulus.  Pemerintah  menghendaki  agar  keamanan  lingkungan  RT,  pentingnya 
pelaksanaan  ujian  nasional  tahun  ini  pembinaan  keamanan  lingkungan  RT, 
didukung penuh oleh semua pihak.   langkah  pembinaan  keamanan 
  lingkungan RT.  
Agar  menjadi  paragraf  uang  baik,  kalimat  (E) Pentingnya  pembinaan  keamanan 
penutup yang sesuai adalah ...  lingkungan  RT,  langkah  pembinaan 
(A) Dengan demikian, pemerintah berharap  keamanan  lingkungan  RT,  sasaran 
agar  ujian  nasional  tahun  ini  jauh  lebih  pembinaan  keamanan  lingkungan  RT, 
berkualitas  dibandingkan  dengan  ujian  bentuk  pembinaan  keamanan 

an_ay
nasional  tahun  lalu  yang  penuh  lingkungan RT.  
kecurangan.    

Halaman 4 
www.onlinelesson.org    734 
27. Baru‐baru  ini,  tim  peneliti  dari  University  (B) Nurudin  (Imron  Zamani,  2008:11) 
School  of  Medicine,  Washington,  berhasil  menyatakan  bahwa  sebagai  pemegang 
mengidentifikasi  protein  plasmodium  sp,  titipan,  tidak  selayaknya  orang  tua 
yaitu  parasit  nyamuk  yang  menyebabkan  menitipkan  lagi  sang  anak  kepada 
penyakit  malaria.  Penemuan  ini  penting  pembantu.  
karena  malaria  kini  sebagai  pembunuh  (C) Nurudin  (2007)  menyatakan  bahwa 
nomor  tiga  di  dunia.  Penyakit  ini  tercatat  sebagai  pemegang  titipan,  tidak 
membunuh satu hingga tiga juta orang setiap  selayaknya  orang  tua  menitipkan  lagi 
tahun.   sang  anak  kepada  pembantu  (dalam 
Paragraf di atas dapat diperbaiki dengan cara  Zamani, 2008) 
berikut, kecuali ...  (D) Sebagai  pemegang  titipan,  tidak 
(A) kata ini diganti itu.  selayaknya  orang  tua  menitipkan  lagi 
(B) kata tim ditulis Tim.   sang  anak  kepada  pembantu  (Nurudin, 
(C) kata dari dihilangkan.   2007). 
(D) kata plasmodium sp dicetak miring.  (E) Menurut  Nurudin  dalam  bukunya 
(E) tanda  koma  setelah  Washington  Zamani  (2008)  menyatakan  bahwa 
dihilangkan.  sebagai  pemegang  titipan,  tidak 
  selayaknya  orang  tua  menitipkan  lagi 
28. Menurut  para  ahli  mengatakan  pengambilan  sang anak kepada pembantu.  
data  penelitian  yang  dilakukan  berulang‐  
ulang  kali  dapat  mempermudah  peneliti  30. Berikut  adalah  identitas  pustaka  yang  dapat 
dalam  melakukan  analisis  data  pasca  digunakan  sebagai  sumber  untuk  menulis 
pengumpulan data.   karangan.  
 
 
Judul  Penulis  Kota  Penerbit  Tahun 
Kalimat  tidak  efektif  di  atas  dapat  diperbaiki  Pendidikan  Rahmini P.  Surabaya   Bintang   2008 
dengan cara berikut, kecuali ...  Anak Jalanan 
Memanusiakan  P. Puspitasari  Medan   Cahaya   2007 
(A) kata menurut dihilangkan.   Anak Manusia 
(B) kata  berulang  kali  diganti  dengan  Perdagangan  Ahmad  Ambon  Cemerlang   2008 
berulang‐ulang.  Anak   Sugentar 
Bimbing  Anak  Dr.  Siana  Surabaya   Gemerlap   2009 
(C) setelah  kata  ahli  ditambahkan  tanda  Berakhlak Mulia  Rohe 
koma.    
(D) kata  bahwa  ditambahkan  setelah  kata  Azhari  menulis  daftar  pustaka  untuk 
mengatakan.   karangannya  yang  berjudul  Pembinaan  Anak 
(E) pasca  pengumpulan  dituliskan  Jalanan.  Berdasarkan  data  pustaka  di  atas, 
serangkai.   penulisan  daftar  pustaka  yang  relevan  dan 
  tepat untuk karangan Azhari tersebut adalah 
29. Dalam  buku  Jangan  Serahkan  anak  kepada  ... 
Pembantu  halaman  11  karya  Imron  Zamani  (A) Ahmad  Sugentar.  2008.  Perdagangan 
yang  terbit  tahun  2008  terdapat  kutipan  Anak. Ambon: Cemerlang. 
berikut.   P.Puspitasari.  2007.  Memanusiakan 
 
Anak Manusia. Medan: Cahaya.  
“Anak,  pada  hakikatnya,  titipan  Tuhan. 
Rahmini  P.  2008.  Pendidikan  Anak 
Sebagai  pemegang  titipan,  tidak  selayaknya 
Jalanan. Surabaya: Bintang.  
orang tua menitipkan lagi sang anak keapda 
Siana  Rohe.  2009.  Bimbing  Anak 
pembantu” 
Berakhlak  Mulia.  Surabaya: 
(Nurudin, 2007) 
Gemerlap. 
Jika  Nunung  mengutip  pendapat  Nurudin 
(B) Puspitasari,  P.  2007.  Memanusiakan 
yang  dikutip  dalam  buku  Imron  Zamani, 
Anak Manusia. Medan: Cahaya.  
penulisan kutipan yang benar adalah ... 
Rahmini  P.  2008.  Pendidikan  Anak 
(A) Nurudin  (dalam  Zamani,  2008:11) 
Jalanan. Surabaya: Bintang.  
menyatakan  bahwa  sebagai  pemegang 
Rohe, S. 2009. Bimbing Anak Berakhlak 
titipan,  tidak  selayaknya  orang  tua 

an_ay
Mulia. Surabaya: Gemerlap. 
menitipkan  lagi  sang  anak  kepada 
Sugentar,  Ahmad.  2008.  Perdagangan 
pembantu.  
Anak. Ambon: Cemerlang. 

Halaman 5 
www.onlinelesson.org    734 
(C) Puspitasari,  P.  2007.  Memanusiakan   
Anak Manusia. Medan: Cahaya.    
Rahmini  P.  2008.  Pendidikan  Anak   
Jalanan. Surabaya: Bintang.    
Rohe,  Siana.  2009.  Bimbing  Anak   
Berakhlak  Mulia.  Surabaya:   
Gemerlap.   
(D) Puspitasari,  P.  2007.  Memanusiakan   
Anak Manusia. Medan: Cahaya.    
Rahmini  P.  2008.  Pendidikan  Anak   
Jalanan. Surabaya: Bintang. 
Rohe, S. 2009. Bimbing Anak Berakhlak 
Mulia. Surabaya: Gemerlap. 
(E) Rahmini  P.  2008.  Pendidikan  Anak 
Jalanan. Surabaya: Bintang. 
Puspitasari,  P.  2007.  Memanusiakan 
Anak Manusia. Medan: Cahaya. 
Sugentar,  Ahmad.  2008.  Perdagangan 
Anak. Ambon: Cemerlang. 
Rohe,  Siana.  2009.  Bimbing  Anak 
Berakhlak  Mulia.  Surabaya: 
Gemerlap.  
 
 
 
 
 
 
 
 
 
 
 
 
 
 
 
 
 
 
 
 
 
 
 
 
 
 
 
 
 

an_ay
 
 
 

Halaman 6 
www.onlinelesson.org    734 
Simulasi Soal Snmptn 2011 
 
Mata Pelajaran  : Bahasa Inggris 
Kode Soal    : 734 
 
 
Gunakan PETUNJUK A untuk menjawab soal nomor 31 sampai dengan nomor 45! 
   
Text 1 
 
The first ancient DNA sequences to be gathered ‐ 3400 base pairs from a 2400‐year‐old Egyptian mummy    − 
were a proof of principle. A full genome sequence would be far more informative  − perhaps explaining what 
killed King Tut, for instance. At present, Inuk's is the only published ancient human genome. However, 
a  team  led  by  Svante  Paabo  and  Ed  Green  at  the  Max  Planck  Institute  for  Evolutionary  Anthropology  in 
5  Leipzig,  Germany,  will  soon  publish  the  complete  genome  sequence  combined  together  from  several 
Neanderthals, from between 38,000 and 70,000 years ago. 
Neanderthals  are  not  the  only  hominids  whose  genomes  could  be  sequenced,  says  Willerslev.  Homo 
erectus,  a  species  that  emerged  in  Africa  about  2  million  years  ago,  survived  in  east  Asia  until  less  than 
100,000  years  ago.  If  well‐preserved  bones  can  be  found,  a  genome  might  be  possible,  Willerslev  says. 
10 Willerslev's  laboratory  has  just  received  bones  from  Spain  belonging  to  Homo  heidelbergensis,  the 
predecessor  to  Neanderthals.  "We  are  basically  starting  on  it  right  now,"  he  says.  If  these  genomes  ever 
materialize ‐ and that's a big if ‐ they could lead to a better understanding of how different hominid species  
are related, and when and where they branched off. If the genetic information is good enough, it may tell us 
something about the nature of past peoples − possibly even what they looked like. Ancient human genomes 
15  could give us insights into the evolution of our own species, explaining when genes involved in disease and 
higher cognitive skills emerged. 
But  DNA  is  not  forever.  As  it  ages,  its  long  strands  shred  into  ever  smaller  pieces.  Eventually  they 
become too small to reassemble, and all information is lost. "There seems to be a time horizon of 100,000 
years  or  so  under  most  preservation  conditions  during  which  intact  DNA  survives,"  Green  says.  Stephan 
20  Schuster  at  Pennsylvania  State  University,  who  led  the  woolly  mammoth  genome  project,  thinks  ancient 
genomics  is  already  plateauing.  Large  chunks  of  Inuk's  genome  couldn't  be  filled  in  because  his  DNA  had 
crumbled into small pieces. "We will face an uphill battle in trying to apply this to a large number of human 
remains," he says. 
 
31. With  reference  to  the  whole  text,  the  writer  (E) requiring  a  longer  chain  of  complete 
mainly deals with the topic on …  DNA sequences.  
(A) the use of ancient DNA in anthropology.   
(B) the past life of the Neanderthals.  33. The physical look of hominid species can even 
(C) the role of DNA in fossil studies.  be  reconstructed  using  the  DNA  technology 
(D) DNA research on the Neanderthals.  under the condition that … 
(E) DNA  engineering  in  the  Max  Planck  (A) the  DNA  forms  a  solid  fossil  in  a  good 
Institute.   shape.  
  (B) the  genetic  information  in  the  DNA  is 
32. The  writer  is  mainly  of  the  opinion  that  high‐quality.  
tracing ancient humans’ life using their DNA …  (C) the DNA had not been smaller in chunks.  
(A) is technologically possible through it has  (D) the  sequence  of  the  DNA  pieces  is 
natural challenges.  systematic.  
(B) promises  a  new  horizon  of  (E) the  DNA  sequences  can  be  easily 
understanding past illnesses.  reassembled.  
(C) is  a  new  breakthrough  in  modern   
anthropological studies.  34. Based on the text, the following would be the 

an_ay
(D) provides a better picture of old peoples’  kind  of  information  that  could  be  revealed 
DNA structures. 

Halaman 1 
www.onlinelesson.org    734 
about  ancient  people  through  modern  empirically  validated,  if  contemporary  DNA 
genome analyses, EXCEPT …  analyses  are  supported  by  the  following 
(A) typical illnesses.  factors, EXCEPT … 
(B) thinking potentials.  (A) more discovery of early human remains.  
(C) evolutionary phases.   (B) modern genomic laboratories.  
(D) migratory directions.   (C) expertise associated with ancient DNA.  
(E) nutritional patterns.   (D) better management  in fossil excavation.  
  (E) availability  of  complete  ancient  human 
  genomes.  
   
35. Based  on  the  text,  soon  Darwin’s  human 
evolution  theory  will  be  most  likely 
Text 2 
 
Further research, conducted by Dean Mobbs, then at Stanford University in California, uncovered a second 
point  of  activity  in  the  brain's  limbic  system  ‐  associated  with  dopamine  release  and  reward  processing  ‐ 
which  may  explain  the  pleasure  felt  once  you  "get"  the  joke.  Examining  one  particular  part  of  the  limbic 
system‐the ventral striatum‐was especially revealing, as its level of activity corresponded with the perceived 
funniness of a joke. "It's the same region that is involved in many different types of reward, from drugs, to 
sex and our favourite music," says Mobbs, now at the MRC Cognition and Brain Sciences Unit in Cambridge, 
UK. "Humour thus taps into basic rewards systems that are important to our survival." 
Yet humour is a far more multifaceted process than primeval pleasures like food. In addition to the two 
core processes of getting  the joke and  feeling good  about it, jokes also activate regions of the frontal and 
cingulate cortex, which are linked with association formation, learning and decision‐making. The team also 
found heightened activity in the anterior cingulate cortex and the frontoinsular cortex ‐ regions that are only 
present in humans and, in a less developed form, great apes. Indeed, the fact that these regions are involved 
suggests  that  humour  is  an  advanced  ability  which  may  have  only  evolved  in  early  humans,  says  Watson, 
who conducted the research. 
No two brains are the same, however, and how these differences are reflected in our sense of humour 
is  the  subject  of  much  research.  Men  and  women,  for  example,  seem  to  process  jokes  slightly  differently. 
Although  both  sexes  laugh  at  roughly  the  same  number  of  jokes,  women  show  greater  activity  in  the  left 
prefrontal  cortex  than  men.  "This  suggests  a  greater  degree  of  executive  processing  and  language‐based 
decoding," says Mobbs. As a result, women take significantly much longer than men to decide whether they 
find something funny, though that does not seem to spoil their enjoyment of the joke. Indeed, women show 
a greater response in the limbic system than men, suggesting they feel a greater sense of reward. 
 
36. Which of the following statements in the text  37. The  part  that  comes  before  the  text  would 
above contains an opinion?  most likely describe … 
(A) Basic  rewards  systems  associated  with  (A) Mobbs’  same  study  conducted  on 
humor  are  vital  to  human  continued  humans’  brain  limbic  system 
existence.   sensitiveness.  
(B) The  anterior  cingulate  cortex  and  the  (B) another  activity  found  in  the  brain’s 
frontoinsular  cortex  regions  belong  to  limbic system of Mobbs’ different study.  
humans.   (C) a  researcher’s  study  other  than  Mobbs’ 
(C) Women’s  limbic  system  reacts  more  on  another  activity  found  in  the  brain 
responsively  than  men’s  does  to  a  limbic system.  
humor stimulus.   (D) the importance of conducting a study on 
(D) Different types of reward like drugs and  responses  by  humans’  brain  limbic 
our  favorite  music  are  linked  to  the  system.  
ventral striatum.   (E) the  timely  reaction  of  humans’  brain 
(E) Joke funiness relates with the amount of  limbic system to perception on humor.  

an_ay
dopamine  release  and  reward   
processing.    
   

Halaman 2 
www.onlinelesson.org    734 
38. The  scientists  see  that  the  internal  (B)  
mechanisms  in  a  human’s  brain  related  with   
enjoyment in having good meals are …   
(A) more  explainable  than  those  in   
perceiving a joke.    
(B) less observable than pleasures in getting   
a prize.    
(C) as quick as those in seeing funniness of a   
joke.    
(D) more complex than those in perceiving a  (C)  
joke.    
(E) less  clear  than  those  of  enjoyment  in   
winning a prize.    
   
39. Humor is worth in these four facets, EXCEPT …   
(A) experiencing  the  sense  of  the  humor   
itself.    
(B) having good feelings about the humor.   (D)  
(C) activating  parts  of  the  frontal  and   
cungulate cortex.    
(D) enhancing  activities  in  two  humans’   
cortex areas.    
(E) triggering more dynamic responses in all   
cells.    
   
40. Which  chart  below  most  likely  indicates  the  (E)  
response time needed by women and men to   
laugh on a joke?   
(A)    
   
   
   
   
   
   
 
Text 3 
 
John Apollos is losing weight the old‐fashioned way − by eating less. A whole lot less. As a volunteer in the two‐
year  Comprehensive  Assessment  of  Long‐Term  Effects  of  Reducing  Intake  of  Energy  (CALERIE)  study  at  Tufts 
University  in  Boston,  Apollos  has  lowered  his  daily  calorie  intake  25%  over  the  past  eight  months.  The  fat,  not 
surprisingly,  has  melted  away;  the  52‐year‐old  physical  trainer  has  lost  more  than  11  kg  since  the  study  began  
5  and is down to his high school weight. 
Yet,  that's  not  the  real  reason  Apollos  and  the  other  participants  in  the  program  are  eating  only  three‐ 
quarters  of  what  they  used  to.  The  researchers  running  the  multicenter  CALERIE  study  are  trying  to  determine 
whether restricting food intake can slow the aging process and extend our life span. "I feel better and lighter and 
healthier,"  says  Apollos.  "But  if  it  could  help  you  live  longer,  that  would  be  pretty  amazing."  The  idea  is 
10  counterintuitive:  If  we  eat  to  live,  how  can  starving  ourselves  add  years  to  our  lives?  Yet.  decades  of  calorie‐ 
restriction  studies  involving  organisms  ranging  from  microscopic  yeast  to  rats  have  shown  just  that,  extending   
the  life  spans  of  the  semi  starved  as  much  as  50%.  Last  July  a  long‐term  study  led  by  researchers  at  the    
University of Wisconsin nudged the implications of this a bit closer to our species, finding that calorie restriction 
seemed  to  extend  the  lives  of  humanlike  rhesus  monkeys  as  well.  The  hungry  primates  fell  victim  to  diabetes, 

an_ay
15  heart and brain disease and cancer much less frequently than their well‐fed counterparts did. 

Halaman 3 
www.onlinelesson.org    734 
However,  there  may  be  more  than  just  the  absence  of  disease  operating  here.  Anytime  you  go  on  a  diet, 
after  all,  you  stand  a  good  chance  of  lowering  your  blood  pressure,  cholesterol  level  and  risk  of  diabetes  and  
other health woes. All that can translate into extra years. With calorie restriction, usually defined as a diet with 
25% to 30% fewer calories than normal but still containing essential nutrients, something else appears to be at 
20  work to extend longevity. 
 
 
41. Which  of  the  following  ideas  from  the  text  45. As  mentioned  in  the  text,  the  study  held  at 
above contains an opinion?  the  multicenter  CALERIE,  Tufts  University  in 
(A) Restricting  calorie  consumption  for  Boston has employed a research method that 
longevity.   seems to be … 
(B) Absence  of  disease  due  to  controlling  (A) complicated.  
calorie intake.   (B) temporary.  
(C) Apollo’s  feeling  better  and  lighter  and  (C) ultramodern.  
healthier.   (D) conventional.  
(D) The  melting  of  body  fat  by  having  less  (E) methodological.  
foods.  
(E) Apollos’  restricting  his  daily  calorie 
consumption. 
 
42. The  study  aims  at  evaluating  the  impact  of 
calorie restriction on … 
(A) minimizing  the  risks  of  getting  serious 
illnesses.  
(B) enhancing  the  feeling  of  happiness  and 
health.  
(C) reducing  the  amount  of  extreme  fat 
accumulation.  
(D) controlling  blood  pressure  and 
cholesterol levels.  
(E) aging‐process  slowing  and  life  span 
extension.  
 
43. If the information in the text is true, the risks 
that  someone  whose  calorie  consumption  is 
controlled up to the portion suggested in the 
study suffers from bone cancer are … 
(A) substantial.  
(B) negligible.  
(C) unpredictable.  
(D) serious.  
(E) indefinite.  
 
44. The  following  is  among  other  things  the 
empirical  impacts  of  the  study  mentioned  in 
the text, EXCEPT … 
(A) slender body shape.  
(B) long life expectancy.  
(C) good healthiness.  
(D) emotion stability.  
(E) good feelings.  
 

an_ay
 
 

Halaman 4 
www.onlinelesson.org    328 
Simulasi Soal Snmptn 2011 
 
Mata Pelajaran  : Sejarah 
Kode Soal    : 328 
 
 
Gunakan  PETUNJUK  A  untuk  menjawab  soal  4. Latar  belakang  pemindahan  pusat 
nomor 1 sampai dengan nomor 8!  kekuasaan  kerajaan  Mataram  Kuno  dari 
  Jawa Tengah ke Jawa Timur adalah ... 
1. Alasan sejarah sebagai suatu peristiwa tidak  (A) perang saudara antara dinasti Sanjaya 
dapat berulang adalah ...  dan  Sailendra  yang  melibatkan 
(A) setiap peristiwa selalu terjadi di dalam  kerajaan Nalanda di India.  
ruang  dan  waktu  yang  berbeda  (B) adanya  bencana  alam  yang 
dengan peristiwa yang lain.   menyebabkan  banyak  korban  di 
(B) setiap  kisah  dari  suatu  peristiwa  wilayah Jawa Tengah. 
sejarah  tidak  dapat  mengulang  kisah  (C) serangan  kerjaan  Tarumanegara  dari 
sejarah yang sama.   Jawa Barat.  
(C) peristiwa  di  masa  lampau  tidak  dapat  (D) munculnya  aliran  baru  dalam  agama 
diteliti secara berulang‐ulang.   Hindu‐Buddha di Jawa Timur.  
(D) pelaku  dan  saksi  sejarah  tidak  dapat  (E) ancaman  bangsa  Cina  yang  selalu 
menceritakan  peristiwa  yang  sama  menuntut penyerahan upeti.  
dengan tafsir yang sama.    
(E) sulit  menemukan  bukti‐bukti  yang   
dapat  digunakan  untuk  mengulang   
peristiwa di masa lampau.    
   
2. Kehidupan  masyarakat  masa  neolithik  di   
Indonesia berbeda dengan kehidupan masa   
sebelumnya.  Hal  tersebut  ditandai  oleh   
kebiasaan ...   
(A) berburu  dan  membuat  alat‐alat  dari   
batu.    
(B) berburu  dan  bertempat  tinggal  di   
dalam gua‐gua.    
(C) bercocok  tanam  dan  membuat  alat‐  
alat dari logam.    
(D) bercocok  tanam  dan  membuat   
sarcopagus.    
(E) bercocok  tanam  dan  bertempat   
tinggal menetap.    
   
3. Langkah  yang  dilakukan  Raja  Kertanegara   
dari  Singasari  dalam  mengembangkan   
wilayahnya adalah ...   
(A) menyerahkan  kekuasaannya  kepada   
putra mahkota.    
(B) bekerja sama dengan Kediri.    
(C) melakukan ekspedisi Pamalayu.    
(D) bekerja sama dengan Sriwijaya.    
(E) membagi kerajaannya menjadi Panjalu   
dan Kediri.    

an_ay
 
Jawablah soal nomor 5 dan nomor 6 soal berdasarkan teks di bawah ini! 

Halaman 1 
www.onlinelesson.org    328 
 
Pembentukan Boedi Oetomo 
 
Pada  awal  abad  ke‐20  pemerintah  kolonial  mulai  mengusahakan  kemajuan  rakyat  bumiputera,  yang 
telah  berpuluh  tahun  dieksploitasi  melalui  cultuur  stelsel  dan  kemudian  sistem  liberal,  yang  telah 
menempatkan  orang  bumiputera  sebagai  tenaga  kerja  rendahan  dengan  gaji  atau  upah  yang  sangat 
rendah  jika  dibandingkan  gaji  para  pekerja  Eropa.  Usaha  untuk  memajukan  kehidupan  rakyat 
bumiputera  itu  dilembagakan  dalam  politik  etis  yang  mencakup  terutama  usaha  untuk  memajukan 
pengajaran.  Namun  demikian,  masih  banyak  anak  bangsa  bumiputera  tidak  bisa  bersekolah,  karena 
orang tuanya miskin sehingga tidak mampu untuk membiayai pendidikan anak‐anak mereka. 
Melihat  kondisi  ini,  Wahidin  Soediro  Hoesodo,  seorang  dokter  Jawa,  berusaha  mendirikan  studie 
fonds (dana pendidikan) untuk anak‐anak bangsa yang cerdas tetapi tidak mampu untuk memikul biaya 
pendidikan.  Pada  tahun  1906‐1907  dr.  Wahidin  mengunjungi  berbagai  tempat  di  pulau  Jawa  untuk 
memprogandakan  gagasannya  itu.  Pada  tahun  1907,  Wahidin  mengunjungi  School  tot  Opleiding  van 
Inlandsche Artsen (STOVIA) di Weltevreden, Batavia. Idenya ini diterima secara antusias oleh siswa‐siswa 
STOVIA.  Soetomo  dan  rekannya  Goenawan  Mangoenkoesoemo,  yang  ketika  itu  masih  menjadi  siswa 
STOVIA,  memproklamasikan  pendirian  Boedi  Oetomo  pada  tanggal  20  Mei  1908.  Pembentukan  Boedi 
Oetomo  dihadiri  dan  didukung  oleh  siswa‐siswa  sekolah  lainnya,  yaitu  Sekolah  Pertanian  di  Bogor, 
Sekolah  Dokter  Hewan  di  Bogor,  Sekolah  Kepala  Negeri  di  Magelang  dan  Probolinggo,  Sekolah  Malam 
untuk  Penduduk  Kota  di  Surabaya,  Sekolah  Pendidikan  Guru  Bumiputera  di  Bandung,  Yogyakarta,  dan 
Probolinggo. Tujuan Boedi Oetomo adalah mengusahakan persatuan kaum Boemipoetera yang sedapat 
mungkin  bersifat  umum,  sehingga  akan  tercapai  Persatuan  orang  Jawa  pada  umumnya,  dengan  Boedi 
Oetomo  hanya  sebagai  pelopor,  yang  tugas  utamanya  adalah  untuk  merancang  cara‐cara  yang  tepat 
untuk  mencapai  terwujudnya  suatu  pendidikan  yang  serasi  bagi  negara  dan  rakyat  Hindia  Belanda 
(Pengumuman Afdeeling Bestuur Boedi Oetomo, 1908). 
 
5. Boedi  Oetomo  dibentuk  sebagai  reaksi  (E) berusaha menghapus liberalisme yang 
terhadap ...  menyengsarakan rakyat bumiputera.  
(A) cultuur  Stelsel  yang  mengeksploitasi   
tenaga kerja bumiputera.  7. Sikap pemerintah pendudukan Jepang pada 
(B) liberalisme  yang  mengeksploitasi  tahun  1942  –  1945  yang  membatasi 
tenaga kerja bumiputera dengan upah  gerakan  kemerdekaan  Indonesia  tampak 
yang sangat rendah.   pada upayanya dengan cara ... 
(C) keinginan  dr.  Wahidin  Soediro  (A) membentuk  organisasi  Putera  (Pusat 
Hoesodo  untuk  membentuk  studie  Tenaga Rakyat) 
fonds.   (B) merekrut  rakyat  bumiputera  untuk 
(D) usaha  pemerintah  kolonial  untuk  menjadi romusha.  
memajukan  pendidikan  rakyat  (C) melarang  penggunaan  bahasa 
bumiputera.   Indonesia.  
(E) kebijakan  pemerintah  kolonial  yang  (D) membubarkan  organisasi‐organisasi 
memberi  sedikit  kesempatan  bagi  politik yang telah ada sebelumnya.  
rakyat bumiputera untuk bersekolah.   (E) memberlakukan  undang‐undang 
  antimiliter.  
6. Berdasarkan teks di atas, dapat disimpulkan   
bahwa Boedi Oetomo ...   
(A) bertujuan  untuk  menghapuskan   
Cultuur  stelsel  yang  menyengsarakan   
rakyat Jawa.    
(B) adalah  perintis  kemajuan  pendidikan   
rakyat bumiputera di Indonesia.   8. Peran  besar  Jenderal  Soedirman  pada 
(C) dibentuk untuk kemajuan rakyat Jawa  peristiwa  yang  terjadi  pada  tanggal  20 

an_ay
dan Madura.   November 1945 adalah ... 
(D) dibentuk  untuk  persatuan  rakyat  (A) mengalahkan tentara Sekutu dan NICA 
Hindia Belanda.   di Yogyakarta melalui perang gerilya.  

Halaman 2 
www.onlinelesson.org    328 
(B) mempertahankan  Yogyakarta  dari  13. Penyebab kegagalan pemerintah Orde Baru 
serangan Sekutu dan NICA.   dalam  menerapkan  sistem  demokrasi  di 
(C) melawan  tentara  Sekutu  dan  NICA  Indonesia, adalah ... 
dalam pertempuran di Ambarawa.   (1) kebebasan  pers  dibelenggu  dengan 
(D) melawan  tentara  Sekutu  dan  Belanda  slogan “bebas & bertanggung jawab”. 
dalam pertempuran di Sumatera.   (2) tidak  ada  kaderisasi  pemimpin  di 
(E) memenangkan  perang  melawan  jajaran pemerintahan.  
Sekutu dan NICA di Semarang.   (3) tidak  ada  pergantian  presiden  selama 
  30 tahun.  
  (4) tidak  ada  sistem  pemilihan  umum 
Gunakan  PETUNJUK  B  untuk  menjawab  soal  yang jujur dan adil.  
nomor 9 sampai dengan nomor 11!   
  14. Tujuan  dibentuknya  Dana  Moneter 
9. Pembentukan  ASEAN  diresmikan  di  International  (International  Monetary 
Bangkok,  Thailand  pada  tanggal  8  Agustus  Fund/IMF) pada tahun 1945 adalah ... 
1967.   (1) memajukan  kerja  sama  internasional 
SEBAB  dalam bidang keuangan. 
Pada  waktu  itu  terjadi  krisis  ekonomi  yang  (2) memperlancar  perdagangan 
melanda  negara‐negara  di  kawasan  Asia  internasional.  
Tenggara.   (3) memperlancar  lalu‐lintas  pembayaran 
  bilateral.  
10. Sistem  demokrasi  langsung  dapat  (4) membuka  bank‐bank  swasta  di 
dilaksanakan pada masa Yunani kuno.   negara‐negara berkembang.  
SEBAB   
Rakyat  Yunani  kuno  tidak  mengenal  15. Akibat  berdirinya  negara  Republik  Rakyat 
pembagian golongan atau kasta.   Cina  yang  berhaluan  komunis  pada  tahun 
  1949 adalah ... 
11. Awal  revolusi  industri  ditandai  dengan  (1) organisasi  pertahanan  negara‐negara 
penemuan tambang batu bara di Inggris.   komunis, Pakta Warsawa makin kuat.  
SEBAB  (2) orang‐orang  Cina  di  Indonesia  yang 
Pada  awal  abad  ke‐19  batu  bara  mulai  berhaluan  komunis  mendirikan  partai 
dipergunakan  sebagai  bahan  bakar  untuk  komunis Indonesia.  
menjalankan kereta api.   (3) masyarakat  Cina  di  seluruh  dunia 
  mendukung  komunisme  demi 
  kejayaan  dan  kebesaran  bangsa  dan 
Gunakan  PETUNJUK  C  untuk  menjawab  soal  negara Cina.  
nomor 12 sampai dengan nomor 15!  (4) Masyarakat  Cina  yang  tidak  setuju 
  dengan  ideologi  komunisme 
12. Penyebab  Uni  Soviet  bergabung  dengan  mendirikan negara Taiwan.  
Sekutu  dalam  menghadapi  fasisme  selama   
berlangsungnya Perang Dunia II adalah ... 
(1) pengambil‐alihan  daerah  jajahan  Uni 
Soviet oleh Jepang.  
(2) janji  Amerika  untuk  memberikan 
bantuan ekonomi dan militer pada Uni 
Soviet  selama  berlangsungnya  Perang 
Dunia II.  
(3) penerapan  ideologi  marxisme  dan 
kapitalisme  secara  bersamaan  di  Uni 
Soviet.  
(4) kedekatan  baik  marxisme  maupun 

an_ay
kapitalisme  dengan  demokrasi 
dibanding dengan fasisme.  
 

Halaman 3 
www.onlinelesson.org    328 
Simulasi Soal Snmptn 2011 
 
Mata Pelajaran  : Sosiologi 
Kode Soal    : 328 
 
 
Gunakan  PETUNJUK  A  untuk  menjawab  soal  (D) budaya 
nomor 16 sampai dengan nomor 32!  (E) edukatif 
   
16. Sebagai ilmu pengetahuan, sifat yang harus  20. Sosialisasi  yang  bersifat  perintah  dengan 
melekat dalam kajian sosiologi adalah ...  komunikasi  satu  arah  disertai  hukuman 
(A) normatif, khusus, dan bebas nilai.  atau  imbalan  dengan  harapan  munculnya 
(B) interaktif, subjekstif, dan evaluatif.  sikap patuh merupakan sosialisasi ... 
(C) prediktif, aplikatif, dan ekonomis.  (A) partisipatoris 
(D) kritis, analitis dan empiris.  (B) informal  
(E) komunikatif,  sugestif,  dan  (C) formal  
menyenangkan.  (D) represif 
  (E) primer 
17. Menjelang  pertandingan  sepak  bola  di   
Gelora Bung Karno Jakarta terdapat banyak  21. Menurut  Talcott  Parsons,  perubahan  sosial 
orang  yang  antre  membeli  tiket  untuk  yang  mendasar  dalam  suatu  sistem  sosial 
menyaksikan  pertandingan.  Di  sana  disebut ... 
terdapat  kelompok  sosial  yang  tidak  (A) involusi 
terorganisasi.  Interaksi  antarindividu  (B) kultural  
bersifat  spontan  dan  tidak  terduga.  (C) teknologi  
Kelompok sosial semacam itu tergolong ...  (D) ideologi  
(A) publik  (E) struktural  
(B) simpatisan   
(C) kerumunan   22. Berikut  ini  adalah  contoh  perilaku 
(D) suporter  menyimpang, kecuali ... 
(E) massa  (A) mencontek waktu ujian  
  (B) merokok di dalam bis kota.  
18. Dalam  masyarakat  multietnik  dan  (C) mencukur rambut model jabrik.  
multikultur  seperti  Indonesia,  asimiliasi  (D) membolos sekolah.  
budaya sangat diperlukan. Salah satu faktor  (E) menerabas antrean di pompa bensin.  
yang  mempersulit  terjadinya  asimilasi   
adalah ...  23. Pernyataan yang benar tentang kontravensi 
(A) sikap etnosentris.   adalah bentuk proses sosial yang berada di 
(B) perkawinan campuran.   antara ... 
(C) adanya musuh bersama dari luar.   (A) kerja sama dan persaingan.  
(D) sikap  terbuka  dari  golongan  yang  (B) integrasi dan pertentangan.  
berkuasa.   (C) akomodasi dan pertentangan.  
(E) perasaan empati yang tinggi.   (D) akomodasi dan kerjasam.  
  (E) persaingan dan pertentangan.  
19. Munculnya  anggapan  masyarakat  bahwa   
HIV/AIDS  merupakan  penyakit  akibat   
pergaulan  bebas  yang  berdampak  pada   
penderita  dan  keluarganya  dikucilkan   
masyarakat.  Hal  ini  merupakan  salah  satu   
bentuk sanksi ....   
(A) ekonomis  Gunakan  PETUNJUK  C  untuk  menjawab  soal 

an_ay
(B) sosial   nomor 24 sampai dengan nomor 27! 
(C) fisik    

Halaman 1 
www.onlinelesson.org    328 
24. Komunitas facebookers, komunitas pencinta  28. Seiring  dengan  perubahan  masyarakat 
sepak  bola,  dan  sebagainya  sedang  tradisional  menjadi  modern  berkembang 
menggejala  di  berbagai  kalangan  pula solidaritas organik menjadi mekanik.  
masyarakat.  Komunitas  semacam  itu  SEBAB 
bersifat ...  Kehidupan  modern  ditandai  oleh  inovasi 
(1) memiliki  interaksi  secara  intensif  di  teknologi  yang  berdampak  melemahnya 
antara anggotanya.   interaksi secara langsung di antara anggota 
(2) memiliki tujuan bersama.   kelompok primer.  
(3) memiliki jejaring sosial yang luas.    
(4) berada  dalam  sautu  wilayah  yang  29. Dalam  stratifikasi  sosial  yang  bersifat 
sangat terbatas.   terbuka,  pendidikan  berperan  menentukan 
  terjadinya mobilitas sosial.  
25. Walapun “masyarakat” memiliki pengertian  SEBAB 
bermacam‐macam,  di  dalamnya  selalu  Tingkat pendidikan pasti berkorelasi dengan 
terdapat ciri berikut ...  tingkat  penghasilan  yang  diperoleh 
(1) mempunyai tujuan bersama.  seseorang.  
(2) memenuhi  kebutuhan  yang  sangat   
terbatas.  30. Korupsi  yang  membudaya  dan  penggunaan 
(3) mempunyai kepentingan yang sama.   NAPZA  yang  marak  merupakan  contoh 
(4) memiliki karakterisitik fisik yang sama.   perilaku  menyimpang  yang  dapat 
  dipandang  sebagai  instrumen  untuk 
26. Secara  sosiologis,  keluarga  dipelajari  menguji  moral  dan  hukum  dalam 
sebagai  suatu  sistem,  kelompok  sosial  dan  masyarakat.  
institusi  sosial.  Sebagai  institusi  sosial,  SEBAB 
keluarga memiliki fungsi ...  Struktur  sosial  tidak  hanya  menghasilkan 
(1) proteksi  perilaku  konformis,  tetapi  juga  non‐
(2) reproduksi  konformis,  seperti  pelanggaran  terhadap 
(3) sosialisai  aturan sosial.  
(4) monopoli   
 
27. Manusia  adalah  makhluk  sosial.  Sebagai 
ekspresi  “zoon  politicon”,  setiap  manusia 
selalu memiliki keinginan untuk ... 
(1) mendapatkan  kasih  sayang  dari 
sesama manusia. 
(2) menjadi  satu  dengan  manusia  lain  di 
sekelilingnya.  
(3) memperoleh  pengakuan  dari 
masyarakat.  
(4) menjadi  satu  dengan  suasana  alam  di 
sekelilingnya.  
 
 
 
 
 
 
 
 
 
Gunakan  PETUNJUK  B  untuk  menjawab  soal 

an_ay
nomor 28 sampai dengan nomor 30! 
 

Halaman 2 
www.onlinelesson.org    328 
Simulasi Soal Snmptn 2011 
 
Mata Pelajaran  : Geografi 
Kode Soal    : 328 
 
 
Gunakan  PETUNJUK  A  untuk  menjawab  soal  35. Unsur  interpretasi  untuk  mengidentifikasi 
nomor 31 sampai dengan nomor 38!  peralatan  militer  di  dalam  bunker  dengan 
  citra radar adalah ... 
31. Komponen  peta  topografi  yang  diletakkan  (A) tekstur 
di  pojok  kanan  bawah  dekat  garis  tepi  (B) bentuk 
adalah ...  (C) ukuran 
(A) arah mata angin.   (D) bayangan  
(B) legenda.   (E) rona 
(C) sumber peta.    
(D) skala.   36. Data  masukan  berikut  yang  dapat  diolah 
(E) inset peta.   secara  langsung  melalui  sistem  informasi 
  geografis adalah ... 
32. Penyebab  hutan  hujan  tropis  mempunyai  (A) tingkat pendidikan.  
peran  penting  sebagai  penghasil  oksigen  (B) jenis kelamin. 
dan pengendali iklim global adalah ...  (C) status penduduk.  
(A) berbatang besar.   (D) kepadatan penduduk.  
(B) tumbuhan berlapis.   (E) jenis pekerjaan. 
(C) berdaun lebat.    
(D) kerapatan tinggi.   37. Faktor  penyebab  terjadinya  pergeseran 
(E) berdaun hijau.   lempeng tektonik bumi adalah ... 
  (A) ketidakmerataan  persebaran  laut  dan 
33. Objek  pada  peta  yang  paling  tepat  benua. 
digambarkan  dengan  simbol  kualitatif  (B) variasi relief permukaan bumi. 
adalah ...  (C) perbedaan  substansi  penyusun  kerak 
(A) kepadatan penduduk.   bumi. 
(B) kerapatan hutan.   (D) arus konveksi di bawah kerak bumi. 
(C) produktivitas lahan.   (E) stratigrafi  batuan  penyusun  kerak 
(D) jenis tambang.   bumi. 
(E) kadar pencemaran.    
  38. Gempa  bumi  dengan  kekuatan  6,2  skala 
34. Pada peta topografi dengan interval kontur  richter  di  Yogyakarta  tahun  2006 
25  meter,  tergambar  dua  bukit  dengan  mengakibatkan  kerusakan  hebat  pada 
jarak antara puncak satu dengan lainnya 20  berbagai  bangunan  terutama  permukiman. 
cm.  Jarak  lurus  di  lapang  antara  kedua  Kerusakan  terbanyak  terjadi  di  wilayah 
puncak bukit tersebut adalah ...  kabupaten  Bantul  Yogyakarta  dan 
(A) 1 km  kabupaten  Klaten  Jawa  Tengah.  Gempa 
(B) 5 km  tersebut  juga  menimbulkan  korban  jiwa 
(C) 10 km  sekitar 5000 orang. Kajian geografi tersebut 
(D) 15 km  menggunakan prinsip ... 
(E) 20 km  (A) korologi.  
  (B) deskripsi.  
  (C) korelasi.  
  (D) kronologi.  
  (E) interelasi.  
   

an_ay
Gunakan  PETUNJUK  B  untuk  menjawab  soal 
nomor 39 sampai dengan nomor 41! 

Halaman 1 
www.onlinelesson.org    328 
  (1) harapan hidup tinggi.  
39. Di  daerah  tropis  proses  pelapukan  tanah  (2) beban ketergantungan tinggi.  
berjalan  cepat  karena  memiliki  suhu  dan  (3) piramida penduduk stationer.  
curah hujan yang tinggi.   (4) tingkat mortalitas tinggi.  
SEBAB   
Suhu  dan  curah  hujan  menentukan  proses  45. Prinsip  yang  harus  dianut  dalam 
pembentukan  tanah,  khususnya  dalam  pelaksanaan  pembangunan  berkelanjutan 
mendukung berlangsungnya pelapukan fisik  berwawasan lingkungan adalah ... 
dan kimia.   (1) diversifikasi 
  (2) konservasi 
40. Indonesia  beriklim  tropis  sehingga  (3) harmoni  
berdasarkan  klasifikasi  Koppen  tidak  (4) interdependensi 
terdapat variasi tipe iklim.    
SEBAB 
Koppen menyatakan bahwa setiap kenaikan 
permukaan  bumi  setinggi  100  meter  suhu 
akan turun 0,6 °C. 
 
41. Sungai  anteseden  dapat  mempertahankan 
arah  alirannya  walaupun  wilayah  yang 
dilalui mengalami pengangkatan.  
SEBAB 
Pengangkatan kulit bumi yang relatif lambat 
dapat  diimbangi  oleh  pengikisan  dasar 
sungai.  
 
Gunakan  PETUNJUK  C  untuk  menjawab  soal 
nomor 42 sampai dengan nomor 45! 
 
42. Faktor  pendorong  menurunnya  proporsi 
penduduk  yang  tinggal  di  perdesaan 
Indonesia adalah ... 
(1) perkembangan pesat sektor industri.  
(2) keterbatasan  daya  dukung  lahan 
pertanian.  
(3) perubahan penggunaan lahan di desa.  
(4) keterbatasan lapangan kerja di desa.  
 
43. Karakteristik sektor informal yang dilakukan 
oleh  kaum  urban  dengan  modal  terbatas, 
pendidikan rendah, dan tanpa keterampilan 
adalah ... 
(1) bebas birokrasi.  
(2) pendapatan rendah.  
(3) waktu bebas.  
(4) tempat mapan.  
 
 
 
 
 

an_ay
 
44. Ciri‐ciri kependudukan negara‐negara Eropa 
Barat dan Amerika Serikat adalah ... 

Halaman 2 
www.onlinelesson.org    328 
Simulasi Soal Snmptn 2011 
 
Mata Pelajaran  : Ekonomi 
Kode Soal    : 328 
 
 
Gunakan  PETUNJUK  A  untuk  menjawab  soal  49. Sebuah  kartel  yang  tidak  diorganisasikan 
nomor 46 sampai dengan nomor 58!  dengan  baik  dan  ilegal  cenderung  berumur 
  pendek karena ... 
46. Masuknya  daging  sapi  impor  yang  lebih  (A) tidak  adanya  dorongan  yang  sama 
murah  akan  mengancam  pasar  daging  sapi  akan  kepastian  ekonomi  yang  kuat 
di  Indonesia.  Akibat  masuknya  daging  sapi  dari para anggotanya sendiri.  
impor  terhadap  kurva  permintaan  dan  (B) adanya  dorongan  bagi  para  anggota 
penawaran daging sapi lokal adalah ...  untuk melanggar kuota output.  
(A) kurva  permintaan  bergeser  ke  kiri  (C) lemahnya  kepastian  hukum  bagi  para 
bawah dan kurva penawaran tetap.   anggota  yang  tertarik  untuk 
(B) kurva  permintaan  bergeser  ke  kanan  mengembangkan usaha.  
atas dan kurva penawaran tetap.   (D) kartel  menjadi  tidak  dapat 
(C) kurva  permintaan  bergeser  ke  kiri  menetapkan harga.  
bawah dan kurva penawaran bergeser  (E) kartel  menjadi  tidak  dapat 
ke kanan bawah.   menetapkan outputnya.  
(D) kurva  permintaan  bergeser  ke  kanan   
atas dan kurva penawaran bergeser ke  50. Seorang  pedagang  buah‐buahan  menjual  2 
kiri atas.   produk:  durian  dan  apel.  Elastisitas 
(E) kurva  permintaan  bergeser  ke  kiri  permintaan durian 0,62 sedangkan apel 2,3. 
bawah dan kurva penawaran bergeser  pedagang  mengalami  kesulitan  keuangan 
ke kiri atas.   yang  serius  dan  harus  meningkatkan 
  penerimaan.  Strategi  yang  dapat  dilakukan 
47. Kurva  yang  menjelaskan  tingkat  produksi  pedagang  dalam  penetapan  harga  durian 
yang  sama  di  antara  berbagai  kombinasi  (Pd) dan harga apel (Pa) adalah ... 
dua  jenis  faktor  produksi  yang  digunakan  (A) Pd dinaikkan dan Pa diturunkan.  
adalah ...  (B) Pd dinaikkan dan Pa dinaikkan.  
(A) production possibility curve.   (C) Pd diturunkan dan Pa dinaikkan.  
(B) isoquant curve.  (D) Pd diturunkan dan Pa diturunkan.  
(C) indifference curve.  (E) Pd tetap dan Pa dinaikkan.  
(D) isocost curve.   
(E) budget line curve.   51. Pengaruh  penerapan  kebijakan  moneter 
  yang  kontraktif  oleh  Bank  Indonesia 
48. Jika dalam suatu pasar terdapat kemudahan  terhadap  kurva  permintaan  agregat  adalah 
bagi  perusahaan  untuk  keluar‐masuk  pasar  ... 
dan  terdapat  diferensiasi  produk,  maka  (A) bergeser ke kanan.  
pasar ini disebut ...  (B) bergeser ke kiri.  
(A) persaingan sempurna.   (C) tetap.  
(B) persaingan monopolistik.   (D) bergeser kekanan kemudian ke kiri.  
(C) persaingan oligopolis.   (E) bergeser ke kiri kemudian ke kanan.  
(D) persaingan duopoli.    
(E) persaingan murni.    
   
   
   
   

an_ay
 

Halaman 1 
www.onlinelesson.org    328 
52. Perdagangan  internasional  Indonesia  57. Diselesaikan  pekerjaan  jasa  untuk 
ditunjukkan  oleh  peningkatan  volume  langganan  senilai  Rp100.000,00  dengan 
ekspor  dan  impor  namun  peningkatan  pembayaran  seminggu  kemudian,  maka 
impor  lebih  kecil  daripada  ekspor,  kondisi  jurnalnya adalah ... 
tersebut  akan  berpengaruh  terhadap  nilai  (A) Kas   Rp100.000,00 
tukar rupiah, yaitu memicu terjadinya ...    Pendapatan usaha  Rp100.000,00 
(A) devaluasi  (B) Kas   Rp100.000,00 
(B) revaluasi    Piutang usaha  Rp100.000,00 
(C) depresiasi  (C) Pendapatan service   Rp100.000,00 
(D) apresiasi    Piutang usaha  Rp100.000,00 
(E) stagflasi  (D) Piutang usaha   Rp100.000,00 
  Pendapatan service   Rp100.000,00 
 
(E) Piutang usaha   Rp100.000,00 
53. Pak  Amir  memiliki  tagihan  di  luar  negeri 
  Kas   Rp100.000,00 
sebesar  US$2.000.00.  Jika  terjadi  kenaikan 
 
nilai  tukar  dari  Rp9.000,00  menjadi 
58. Pada  tanggal  8  Januari  2010,  seorang 
Rp9.030,00  untuk  setiap  US$1.00,  maka 
pedagang  eceran  menarik  pinjaman  dari 
perubahan  dalam  nilai  rupiah  jumlah 
Bank  Pelita  Jaya  sebesar  Rp50.000.000,00 
tagihan pak Amir ... 
dengan  bunga  12%  per  tahun.  Uang 
(A) bertambah Rp30.000,00 
tersebut  sepenuhnya  digunakan  sebagai 
(B) bertambah Rp40.000,00 
modal usah sehingga berhasil mendapatkan 
(C) bertambah Rp60.000,00 
penerimaan  penjualan  sebesar 
(D) berkurang Rp30.000,00 
Rp70.000.000,00, dan segera setelah itu dia 
(E) berkurang Rp60.000,00 
melunasi  pinjamannya  ke  bank  pada 
 
tanggal  8  Juni  2010.  Pedagang  eceran 
54. Pengusaha  yang  kurang  memiliki  jiwa 
tersebut memperoleh penghasilan ... 
entrepreneur dapat terlihat pada ... 
(A) Rp14.700.000,00 
(A) kurangnya  keberanian  mengambil 
(B) Rp15.000.000,00 
risiko usaha.  
(C) Rp15.300.000,00 
(B) kesulitan  mendapatkan  tambahan 
(D) Rp17.500.000,00 
modal.  
(E) Rp20.000.000,00 
(C) produk yang dihasilkan inovatif.  
 
(D) kurangnya bantuan dari pemerintah.  
Gunakan  PETUNJUK  B  untuk  menjawab  soal 
(E) kurang memadainya perlatan produksi 
nomor 59! 
yang dimiliki.  
 
 
59. Perusahaan  rental  komputer  menerima 
55. Bauran  pemasaran  barang  meliputi  empat 
uang  sewa  dari  langganannya.  Dalam 
hal, yaitu ... 
laporan  labar  rugi  uang  sewa  tersebut 
(A) product, price, income, distribution 
masuk dalam penghasilan lain‐lain.  
(B) product, price, place, promotion 
SEBAB 
(C) product, income, taste, distribution 
Yang  termasuk  dalam  penghasilan  lain‐lain 
(D) product, price, income, utility 
untuk  perusahaan  tersebut  antara  lain 
(E) product, promotion, price, distribution 
adalah bunga dan sewa.  
 
 
56. Kegiatan  yang  terkait  dengan  efisiensi 
 
usaha adalah melakukan .... 
 
(A) ekstensifikasi usaha.  
 
(B) diversifikasi produk.  
 
(C) intensifikasi usaha.  
 
(D) pemutusan hubungan kerja.  
 
(E) peningkatan produksi.  
 
 
 

an_ay
 
 
 
 
 

Halaman 2 
www.onlinelesson.org    328 
Gunakan  PETUNJUK  C  untuk  menjawab  soal 
nomor 60! 
 
60. Transaksi  yang  dicatat  dalam  buku  jurnal 
penyesuaian adalah ... 
(1) kerugian  piutang  yang 
diperhitungkan/dikeluarkan  dari 
cadangan kerugian piutang.  
(2) retur  dan  potongan  pembelian  yang 
dikeluarkan dari pembelian.  
(3) gaji  karyawan  yang  dikeluarkan  dari 
persekot gaji.  
(4) biaya sewa yang dikeluarkan dari kas.  
 
 

an_ay Halaman 3 
www.onlinelesson.org    720 
Simulasi Soal Snmptn 2011 
 
Mata Pelajaran  : Sejarah 
Kode Soal    : 720 
 
 
Gunakan  PETUNJUK  A  untuk  menjawab  soal  4. Latar  belakang  pemindahan  pusat 
nomor 1 sampai dengan nomor 8!  kekuasaan  kerajaan  Mataram  Kuno  dari 
  Jawa Tengah ke Jawa Timur adalah ... 
1. Alasan sejarah sebagai suatu peristiwa tidak  (A) perang saudara antara dinasti Sanjaya 
dapat berulang adalah ...  dan  Sailendra  yang  melibatkan 
(A) setiap peristiwa selalu terjadi di dalam  kerajaan Nalanda di India.  
ruang  dan  waktu  yang  berbeda  (B) adanya  bencana  alam  yang 
dengan peristiwa yang lain.   menyebabkan  banyak  korban  di 
(B) setiap  kisah  dari  suatu  peristiwa  wilayah Jawa Tengah. 
sejarah  tidak  dapat  mengulang  kisah  (C) serangan  kerjaan  Tarumanegara  dari 
sejarah yang sama.   Jawa Barat.  
(C) peristiwa  di  masa  lampau  tidak  dapat  (D) munculnya  aliran  baru  dalam  agama 
diteliti secara berulang‐ulang.   Hindu‐Buddha di Jawa Timur.  
(D) pelaku  dan  saksi  sejarah  tidak  dapat  (E) ancaman  bangsa  Cina  yang  selalu 
menceritakan  peristiwa  yang  sama  menuntut penyerahan upeti.  
dengan tafsir yang sama.    
(E) sulit  menemukan  bukti‐bukti  yang   
dapat  digunakan  untuk  mengulang   
peristiwa di masa lampau.    
   
2. Kehidupan  masyarakat  masa  neolithik  di   
Indonesia berbeda dengan kehidupan masa   
sebelumnya.  Hal  tersebut  ditandai  oleh   
kebiasaan ...   
(A) berburu  dan  membuat  alat‐alat  dari   
batu.    
(B) berburu  dan  bertempat  tinggal  di   
dalam gua‐gua.    
(C) bercocok  tanam  dan  membuat  alat‐  
alat dari logam.    
(D) bercocok  tanam  dan  membuat   
sarcopagus.    
(E) bercocok  tanam  dan  bertempat   
tinggal menetap.    
   
3. Langkah  yang  dilakukan  Raja  Kertanegara   
dari  Singasari  dalam  mengembangkan   
wilayahnya adalah ...   
(A) menyerahkan  kekuasaannya  kepada   
putra mahkota.    
(B) bekerja sama dengan Kediri.    
(C) melakukan ekspedisi Pamalayu.    
(D) bekerja sama dengan Sriwijaya.    
(E) membagi kerajaannya menjadi Panjalu   
dan Kediri.    

an_ay
 
Jawablah soal nomor 5 dan nomor 6 soal berdasarkan teks di bawah ini! 

Halaman 1 
www.onlinelesson.org    720 
 
Pembentukan Boedi Oetomo 
 
Pada  awal  abad  ke‐20  pemerintah  kolonial  mulai  mengusahakan  kemajuan  rakyat  bumiputera,  yang 
telah  berpuluh  tahun  dieksploitasi  melalui  cultuur  stelsel  dan  kemudian  sistem  liberal,  yang  telah 
menempatkan  orang  bumiputera  sebagai  tenaga  kerja  rendahan  dengan  gaji  atau  upah  yang  sangat 
rendah  jika  dibandingkan  gaji  para  pekerja  Eropa.  Usaha  untuk  memajukan  kehidupan  rakyat 
bumiputera  itu  dilembagakan  dalam  politik  etis  yang  mencakup  terutama  usaha  untuk  memajukan 
pengajaran.  Namun  demikian,  masih  banyak  anak  bangsa  bumiputera  tidak  bisa  bersekolah,  karena 
orang tuanya miskin sehingga tidak mampu untuk membiayai pendidikan anak‐anak mereka. 
Melihat  kondisi  ini,  Wahidin  Soediro  Hoesodo,  seorang  dokter  Jawa,  berusaha  mendirikan  studie 
fonds (dana pendidikan) untuk anak‐anak bangsa yang cerdas tetapi tidak mampu untuk memikul biaya 
pendidikan.  Pada  tahun  1906‐1907  dr.  Wahidin  mengunjungi  berbagai  tempat  di  pulau  Jawa  untuk 
memprogandakan  gagasannya  itu.  Pada  tahun  1907,  Wahidin  mengunjungi  School  tot  Opleiding  van 
Inlandsche Artsen (STOVIA) di Weltevreden, Batavia. Idenya ini diterima secara antusias oleh siswa‐siswa 
STOVIA.  Soetomo  dan  rekannya  Goenawan  Mangoenkoesoemo,  yang  ketika  itu  masih  menjadi  siswa 
STOVIA,  memproklamasikan  pendirian  Boedi  Oetomo  pada  tanggal  20  Mei  1908.  Pembentukan  Boedi 
Oetomo  dihadiri  dan  didukung  oleh  siswa‐siswa  sekolah  lainnya,  yaitu  Sekolah  Pertanian  di  Bogor, 
Sekolah  Dokter  Hewan  di  Bogor,  Sekolah  Kepala  Negeri  di  Magelang  dan  Probolinggo,  Sekolah  Malam 
untuk  Penduduk  Kota  di  Surabaya,  Sekolah  Pendidikan  Guru  Bumiputera  di  Bandung,  Yogyakarta,  dan 
Probolinggo. Tujuan Boedi Oetomo adalah mengusahakan persatuan kaum Boemipoetera yang sedapat 
mungkin  bersifat  umum,  sehingga  akan  tercapai  Persatuan  orang  Jawa  pada  umumnya,  dengan  Boedi 
Oetomo  hanya  sebagai  pelopor,  yang  tugas  utamanya  adalah  untuk  merancang  cara‐cara  yang  tepat 
untuk  mencapai  terwujudnya  suatu  pendidikan  yang  serasi  bagi  negara  dan  rakyat  Hindia  Belanda 
(Pengumuman Afdeeling Bestuur Boedi Oetomo, 1908). 
 
5. Berdasarkan teks di atas, dapat disimpulkan  (E) kebijakan  pemerintah  kolonial  yang 
bahwa Boedi Oetomo ...  memberi  sedikit  kesempatan  bagi 
(A) bertujuan  untuk  menghapuskan  rakyat bumiputera untuk bersekolah.  
Cultuur  stelsel  yang  menyengsarakan   
rakyat Jawa.   7. Peran  besar  Jenderal  Soedirman  pada 
(B) adalah  perintis  kemajuan  pendidikan  peristiwa  yang  terjadi  pada  tanggal  20 
rakyat bumiputera di Indonesia.   November 1945 adalah ... 
(C) dibentuk untuk kemajuan rakyat Jawa  (A) mengalahkan tentara Sekutu dan NICA 
dan Madura.   di Yogyakarta melalui perang gerilya.  
(D) dibentuk  untuk  persatuan  rakyat  (B) mempertahankan  Yogyakarta  dari 
Hindia Belanda.   serangan Sekutu dan NICA.  
(E) berusaha menghapus liberalisme yang  (C) melawan  tentara  Sekutu  dan  NICA 
menyengsarakan rakyat bumiputera.   dalam pertempuran di Ambarawa.  
  (D) melawan  tentara  Sekutu  dan  Belanda 
6. Boedi  Oetomo  dibentuk  sebagai  reaksi  dalam pertempuran di Sumatera.  
terhadap ...  (E) memenangkan  perang  melawan 
(A) cultuur  Stelsel  yang  mengeksploitasi  Sekutu dan NICA di Semarang.  
tenaga kerja bumiputera.   
(B) liberalisme  yang  mengeksploitasi   
tenaga kerja bumiputera dengan upah   
yang sangat rendah.    
(C) keinginan  dr.  Wahidin  Soediro   
Hoesodo  untuk  membentuk  studie   
fonds.    
(D) usaha  pemerintah  kolonial  untuk   

an_ay
memajukan  pendidikan  rakyat   
bumiputera.   8. Sikap pemerintah pendudukan Jepang pada 
tahun  1942  –  1945  yang  membatasi 

Halaman 2 
www.onlinelesson.org    720 
gerakan  kemerdekaan  Indonesia  tampak  13. Penyebab  Uni  Soviet  bergabung  dengan 
pada upayanya dengan cara ...  Sekutu  dalam  menghadapi  fasisme  selama 
(A) membentuk  organisasi  Putera  (Pusat  berlangsungnya Perang Dunia II adalah ... 
Tenaga Rakyat)  (1) pengambil‐alihan  daerah  jajahan  Uni 
(B) merekrut  rakyat  bumiputera  untuk  Soviet oleh Jepang.  
menjadi romusha.   (2) janji  Amerika  untuk  memberikan 
(C) melarang  penggunaan  bahasa  bantuan ekonomi dan militer pada Uni 
Indonesia.   Soviet  selama  berlangsungnya  Perang 
(D) membubarkan  organisasi‐organisasi  Dunia II.  
politik yang telah ada sebelumnya.   (3) penerapan  ideologi  marxisme  dan 
(E) memberlakukan  undang‐undang  kapitalisme  secara  bersamaan  di  Uni 
antimiliter.   Soviet.  
  (4) kedekatan  baik  marxisme  maupun 
Gunakan  PETUNJUK  B  untuk  menjawab  soal  kapitalisme  dengan  demokrasi 
nomor 9 sampai dengan nomor 11!  dibanding dengan fasisme.  
   
9. Sistem  demokrasi  langsung  dapat  14. Tujuan  dibentuknya  Dana  Moneter 
dilaksanakan pada masa Yunani kuno.   International  (International  Monetary 
SEBAB  Fund/IMF) pada tahun 1945 adalah ... 
Rakyat  Yunani  kuno  tidak  mengenal  (1) memajukan  kerja  sama  internasional 
pembagian golongan atau kasta.   dalam bidang keuangan. 
  (2) memperlancar  perdagangan 
10. Awal  revolusi  industri  ditandai  dengan  internasional.  
penemuan tambang batu bara di Inggris.   (3) memperlancar  lalu‐lintas  pembayaran 
SEBAB  bilateral.  
Pada  awal  abad  ke‐19  batu  bara  mulai  (4) membuka  bank‐bank  swasta  di 
dipergunakan  sebagai  bahan  bakar  untuk  negara‐negara berkembang.  
menjalankan kereta api.    
  15. Akibat  berdirinya  negara  Republik  Rakyat 
11. Pembentukan  ASEAN  diresmikan  di  Cina  yang  berhaluan  komunis  pada  tahun 
Bangkok,  Thailand  pada  tanggal  8  Agustus  1949 adalah ... 
1967.   (1) organisasi  pertahanan  negara‐negara 
SEBAB  komunis, Pakta Warsawa makin kuat.  
Pada  waktu  itu  terjadi  krisis  ekonomi  yang  (2) orang‐orang  Cina  di  Indonesia  yang 
melanda  negara‐negara  di  kawasan  Asia  berhaluan  komunis  mendirikan  partai 
Tenggara.   komunis Indonesia.  
  (3) masyarakat  Cina  di  seluruh  dunia 
Gunakan  PETUNJUK  C  untuk  menjawab  soal  mendukung  komunisme  demi 
nomor 12 sampai dengan nomor 15!  kejayaan  dan  kebesaran  bangsa  dan 
  negara Cina.  
12. Penyebab kegagalan pemerintah Orde Baru  (4) masyarakat  Cina  yang  tidak  setuju 
dalam  menerapkan  sistem  demokrasi  di  dengan  ideologi  komunisme 
Indonesia, adalah ...  mendirikan negara Taiwan.  
(1) kebebasan  pers  dibelenggu  dengan   
slogan “bebas & bertanggung jawab”. 
(2) tidak  ada  kaderisasi  pemimpin  di 
jajaran pemerintahan.  
(3) tidak  ada  pergantian  presiden  selama 
30 tahun.  
(4) tidak  ada  sistem  pemilihan  umum 
yang jujur dan adil.  

an_ay
 

Halaman 3 
www.onlinelesson.org    720 
Simulasi Soal Snmptn 2011 
 
Mata Pelajaran  : Sosiologi 
Kode Soal    : 720 
 
 
Gunakan  PETUNJUK  A  untuk  menjawab  soal  (D) suporter 
nomor 16 sampai dengan nomor 32!  (E) massa 
   
16. Pernyataan yang benar tentang kontravensi  20. Menurut  Talcott  Parsons,  perubahan  sosial 
adalah bentuk proses sosial yang berada di  yang  mendasar  dalam  suatu  sistem  sosial 
antara ...  disebut ... 
(A) kerja sama dan persaingan.   (A) involusi 
(B) integrasi dan pertentangan.   (B) kultural  
(C) akomodasi dan pertentangan.   (C) teknologi  
(D) akomodasi dan kerjasam.   (D) ideologi  
(E) persaingan dan pertentangan.   (E) struktural  
   
17. Munculnya  anggapan  masyarakat  bahwa  21. Sosialisasi  yang  bersifat  perintah  dengan 
HIV/AIDS  merupakan  penyakit  akibat  komunikasi  satu  arah  disertai  hukuman 
pergaulan  bebas  yang  berdampak  pada  atau  imbalan  dengan  harapan  munculnya 
penderita  dan  keluarganya  dikucilkan  sikap patuh merupakan sosialisasi ... 
masyarakat.  Hal  ini  merupakan  salah  satu  (A) partisipatoris 
bentuk sanksi ....  (B) informal  
(A) ekonomis  (C) formal  
(B) sosial   (D) represif 
(C) fisik   (E) primer 
(D) budaya   
(E) edukatif  22. Berikut  ini  adalah  contoh  perilaku 
  menyimpang, kecuali ... 
18. Dalam  masyarakat  multietnik  dan  (A) mencontek waktu ujian  
multikultur  seperti  Indonesia,  asimiliasi  (B) merokok di dalam bis kota.  
budaya sangat diperlukan. Salah satu faktor  (C) mencukur rambut model jabrik.  
yang  mempersulit  terjadinya  asimilasi  (D) membolos sekolah.  
adalah ...  (E) menerabas antrean di pompa bensin.  
(A) sikap etnosentris.    
(B) perkawinan campuran.   23. Sebagai ilmu pengetahuan, sifat yang harus 
(C) adanya musuh bersama dari luar.   melekat dalam kajian sosiologi adalah ... 
(D) sikap  terbuka  dari  golongan  yang  (A) normatif, khusus, dan bebas nilai. 
berkuasa.   (B) interaktif, subjekstif, dan evaluatif. 
(E) perasaan empati yang tinggi.   (C) prediktif, aplikatif, dan ekonomis. 
  (D) kritis, analitis dan empiris. 
19. Menjelang  pertandingan  sepak  bola  di  (E) komunikatif,  sugestif,  dan 
Gelora Bung Karno Jakarta terdapat banyak  menyenangkan. 
orang  yang  antre  membeli  tiket  untuk   
menyaksikan  pertandingan.  Di  sana   
terdapat  kelompok  sosial  yang  tidak   
terorganisasi.  Interaksi  antarindividu   
bersifat  spontan  dan  tidak  terduga.   
Kelompok sosial semacam itu tergolong ...   
(A) publik   

an_ay
(B) simpatisan  Gunakan  PETUNJUK  C  untuk  menjawab  soal 
(C) kerumunan   nomor 24 sampai dengan nomor 27! 

Halaman 1 
www.onlinelesson.org    720 
   
24. Komunitas facebookers, komunitas pencinta  28. Seiring  dengan  perubahan  masyarakat 
sepak  bola,  dan  sebagainya  sedang  tradisional  menjadi  modern  berkembang 
menggejala  di  berbagai  kalangan  pula solidaritas organik menjadi mekanik.  
masyarakat.  Komunitas  semacam  itu  SEBAB 
bersifat ...  Kehidupan  modern  ditandai  oleh  inovasi 
(1) memiliki  interaksi  secara  intensif  di  teknologi  yang  berdampak  melemahnya 
antara anggotanya.   interaksi secara langsung di antara anggota 
(2) memiliki tujuan bersama.   kelompok primer.  
(3) memiliki jejaring sosial yang luas.    
(4) berada  dalam  sautu  wilayah  yang  29. Korupsi  yang  membudaya  dan  penggunaan 
sangat terbatas.   NAPZA  yang  marak  merupakan  contoh 
  perilaku  menyimpang  yang  dapat 
25. Manusia  adalah  makhluk  sosial.  Sebagai  dipandang  sebagai  instrumen  untuk 
ekspresi  “zoon  politicon”,  setiap  manusia  menguji  moral  dan  hukum  dalam 
selalu memiliki keinginan untuk ...  masyarakat.  
(1) mendapatkan  kasih  sayang  dari  SEBAB 
sesama manusia.  Struktur  sosial  tidak  hanya  menghasilkan 
(2) menjadi  satu  dengan  manusia  lain  di  perilaku  konformis,  tetapi  juga  non‐
sekelilingnya.   konformis,  seperti  pelanggaran  terhadap 
(3) memperoleh  pengakuan  dari  aturan sosial.  
masyarakat.    
(4) menjadi  satu  dengan  suasana  alam  di  30. Dalam  stratifikasi  sosial  yang  bersifat 
sekelilingnya.   terbuka,  pendidikan  berperan  menentukan 
  terjadinya mobilitas sosial.  
26. Secara  sosiologis,  keluarga  dipelajari  SEBAB 
sebagai  suatu  sistem,  kelompok  sosial  dan  Tingkat pendidikan pasti berkorelasi dengan 
institusi  sosial.  Sebagai  institusi  sosial,  tingkat  penghasilan  yang  diperoleh 
keluarga memiliki fungsi ...  seseorang.  
(1) proteksi   
(2) reproduksi   
(3) sosialisai 
(4) monopoli 
 
27. Walapun “masyarakat” memiliki pengertian 
bermacam‐macam,  di  dalamnya  selalu 
terdapat ciri berikut ... 
(1) mempunyai tujuan bersama. 
(2) memenuhi  kebutuhan  yang  sangat 
terbatas. 
(3) mempunyai kepentingan yang sama.  
(4) memiliki karakterisitik fisik yang sama.  
 
 
 
 
 
 
 
 
 

an_ay
 
Gunakan  PETUNJUK  B  untuk  menjawab  soal 
nomor 28 sampai dengan nomor 30! 

Halaman 2 
www.onlinelesson.org    720 
Simulasi Soal Snmptn 2011 
 
Mata Pelajaran  : Geografi 
Kode Soal    : 720 
 
 
Gunakan  PETUNJUK  A  untuk  menjawab  soal  35. Data  masukan  berikut  yang  dapat  diolah 
nomor 31 sampai dengan nomor 38!  secara  langsung  melalui  sistem  informasi 
  geografis adalah ... 
31. Objek  pada  peta  yang  paling  tepat  (A) tingkat pendidikan.  
digambarkan  dengan  simbol  kualitatif  (B) jenis kelamin. 
adalah ...  (C) status penduduk.  
(A) kepadatan penduduk.   (D) kepadatan penduduk.  
(B) kerapatan hutan.   (E) jenis pekerjaan. 
(C) produktivitas lahan.    
(D) jenis tambang.   36. Faktor  penyebab  terjadinya  pergeseran 
(E) kadar pencemaran.   lempeng tektonik bumi adalah ... 
  (A) ketidakmerataan  persebaran  laut  dan 
32. Penyebab  hutan  hujan  tropis  mempunyai  benua. 
peran  penting  sebagai  penghasil  oksigen  (B) variasi relief permukaan bumi. 
dan pengendali iklim global adalah ...  (C) perbedaan  substansi  penyusun  kerak 
(A) berbatang besar.   bumi. 
(B) tumbuhan berlapis.   (D) arus konveksi di bawah kerak bumi. 
(C) berdaun lebat.   (E) stratigrafi  batuan  penyusun  kerak 
(D) kerapatan tinggi.   bumi. 
(E) berdaun hijau.    
  37. Unsur  interpretasi  untuk  mengidentifikasi 
33. Komponen  peta  topografi  yang  diletakkan  peralatan  militer  di  dalam  bunker  dengan 
di  pojok  kanan  bawah  dekat  garis  tepi  citra radar adalah ... 
adalah ...  (A) tekstur 
(A) arah mata angin.   (B) bentuk 
(B) legenda.   (C) ukuran 
(C) sumber peta.   (D) bayangan  
(D) skala.   (E) rona 
(E) inset peta.    
  38. Pada peta topografi dengan interval kontur 
34. Gempa  bumi  dengan  kekuatan  6,2  skala  25  meter,  tergambar  dua  bukit  dengan 
richter  di  Yogyakarta  tahun  2006  jarak antara puncak satu dengan lainnya 20 
mengakibatkan  kerusakan  hebat  pada  cm.  Jarak  lurus  di  lapang  antara  kedua 
berbagai  bangunan  terutama  permukiman.  puncak bukit tersebut adalah ... 
Kerusakan  terbanyak  terjadi  di  wilayah  (A) 1 km 
kabupaten  Bantul  Yogyakarta  dan  (B) 5 km 
kabupaten  Klaten  Jawa  Tengah.  Gempa  (C) 10 km 
tersebut  juga  menimbulkan  korban  jiwa  (D) 15 km 
sekitar 5000 orang. Kajian geografi tersebut  (E) 20 km 
menggunakan prinsip ...   
(A) korologi.    
(B) deskripsi.    
(C) korelasi.    
(D) kronologi.    
(E) interelasi.    

an_ay
   

Halaman 1 
www.onlinelesson.org    720 
Gunakan  PETUNJUK  B  untuk  menjawab  soal  (3) perubahan penggunaan lahan di desa.  
nomor 39 sampai dengan nomor 41!  (4) keterbatasan lapangan kerja di desa.  
   
39. Sungai  anteseden  dapat  mempertahankan  45. Prinsip  yang  harus  dianut  dalam 
arah  alirannya  walaupun  wilayah  yang  pelaksanaan  pembangunan  berkelanjutan 
dilalui mengalami pengangkatan.   berwawasan lingkungan adalah ... 
SEBAB  (1) diversifikasi 
Pengangkatan kulit bumi yang relatif lambat  (2) konservasi 
dapat  diimbangi  oleh  pengikisan  dasar  (3) harmoni  
sungai.   (4) interdependensi 
   
40. Indonesia  beriklim  tropis  sehingga 
berdasarkan  klasifikasi  Koppen  tidak 
terdapat variasi tipe iklim.  
SEBAB 
Koppen menyatakan bahwa setiap kenaikan 
permukaan  bumi  setinggi  100  meter  suhu 
akan turun 0,6 °C. 
 
41. Di  daerah  tropis  proses  pelapukan  tanah 
berjalan  cepat  karena  memiliki  suhu  dan 
curah hujan yang tinggi.  
SEBAB 
Suhu  dan  curah  hujan  menentukan  proses 
pembentukan  tanah,  khususnya  dalam 
mendukung berlangsungnya pelapukan fisik 
dan kimia.  
 
 
Gunakan  PETUNJUK  C  untuk  menjawab  soal 
nomor 42 sampai dengan nomor 45! 
 
42. Ciri‐ciri kependudukan negara‐negara Eropa 
Barat dan Amerika Serikat adalah ... 
(1) harapan hidup tinggi.  
(2) beban ketergantungan tinggi.  
(3) piramida penduduk stationer.  
(4) tingkat mortalitas tinggi.  
 
43. Karakteristik sektor informal yang dilakukan 
oleh  kaum  urban  dengan  modal  terbatas, 
pendidikan rendah, dan tanpa keterampilan 
adalah ... 
(1) bebas birokrasi.  
(2) pendapatan rendah.  
(3) waktu bebas.  
(4) tempat mapan.  
 
44. Faktor  pendorong  menurunnya  proporsi 
penduduk  yang  tinggal  di  perdesaan 
Indonesia adalah ... 

an_ay
(1) perkembangan pesat sektor industri.  
(2) keterbatasan  daya  dukung  lahan 
pertanian.  

Halaman 2 
www.onlinelesson.org    720 
Simulasi Soal Snmptn 2011 
 
Mata Pelajaran  : Ekonomi 
Kode Soal    : 720 
 
 
Gunakan  PETUNJUK  A  untuk  menjawab  soal  50. Sebuah  kartel  yang  tidak  diorganisasikan 
nomor 46 sampai dengan nomor 58!  dengan  baik  dan  ilegal  cenderung  berumur 
  pendek karena ... 
46. Pengusaha  yang  kurang  memiliki  jiwa  (A) tidak  adanya  dorongan  yang  sama 
entrepreneur dapat terlihat pada ...  akan  kepastian  ekonomi  yang  kuat 
(A) kurangnya  keberanian  mengambil  dari para anggotanya sendiri.  
risiko usaha.   (B) adanya  dorongan  bagi  para  anggota 
(B) kesulitan  mendapatkan  tambahan  untuk melanggar kuota output.  
modal.   (C) lemahnya  kepastian  hukum  bagi  para 
(C) produk yang dihasilkan inovatif.   anggota  yang  tertarik  untuk 
(D) kurangnya bantuan dari pemerintah.   mengembangkan usaha.  
(E) kurang memadainya perlatan produksi  (D) kartel  menjadi  tidak  dapat 
yang dimiliki.   menetapkan harga.  
  (E) kartel  menjadi  tidak  dapat 
47. Kurva  yang  menjelaskan  tingkat  produksi  menetapkan outputnya.  
yang  sama  di  antara  berbagai  kombinasi   
dua  jenis  faktor  produksi  yang  digunakan  51. Seorang  pedagang  buah‐buahan  menjual  2 
adalah ...  produk:  durian  dan  apel.  Elastisitas 
(A) production possibility curve.   permintaan durian 0,62 sedangkan apel 2,3. 
(B) isoquant curve.  pedagang  mengalami  kesulitan  keuangan 
(C) indifference curve.  yang  serius  dan  harus  meningkatkan 
(D) isocost curve.  penerimaan.  Strategi  yang  dapat  dilakukan 
(E) budget line curve.   pedagang  dalam  penetapan  harga  durian 
  (Pd) dan harga apel (Pa) adalah ... 
48. Perdagangan  internasional  Indonesia  (A) Pd dinaikkan dan Pa diturunkan.  
ditunjukkan  oleh  peningkatan  volume  (B) Pd dinaikkan dan Pa dinaikkan.  
ekspor  dan  impor  namun  peningkatan  (C) Pd diturunkan dan Pa dinaikkan.  
impor  lebih  kecil  daripada  ekspor,  kondisi  (D) Pd diturunkan dan Pa diturunkan.  
tersebut  akan  berpengaruh  terhadap  nilai  (E) Pd tetap dan Pa dinaikkan.  
tukar rupiah, yaitu memicu terjadinya ...   
(A) devaluasi  52. Jika dalam suatu pasar terdapat kemudahan 
(B) revaluasi  bagi  perusahaan  untuk  keluar‐masuk  pasar 
(C) depresiasi  dan  terdapat  diferensiasi  produk,  maka 
(D) apresiasi  pasar ini disebut ... 
(E) stagflasi  (A) persaingan sempurna.  
  (B) persaingan monopolistik.  
49. Pengaruh  penerapan  kebijakan  moneter  (C) persaingan oligopolis.  
yang  kontraktif  oleh  Bank  Indonesia  (D) persaingan duopoli.  
terhadap  kurva  permintaan  agregat  adalah  (E) persaingan murni.  
...   
(A) bergeser ke kanan.    
(B) bergeser ke kiri.    
(C) tetap.    
(D) bergeser kekanan kemudian ke kiri.    
(E) bergeser ke kiri kemudian ke kanan.    

an_ay
   

Halaman 1 
www.onlinelesson.org    720 
53. Pak  Amir  memiliki  tagihan  di  luar  negeri  Rp70.000.000,00, dan segera setelah itu dia 
sebesar  US$2.000.00.  Jika  terjadi  kenaikan  melunasi  pinjamannya  ke  bank  pada 
nilai  tukar  dari  Rp9.000,00  menjadi  tanggal  8  Juni  2010.  Pedagang  eceran 
Rp9.030,00  untuk  setiap  US$1.00,  maka  tersebut memperoleh penghasilan ... 
perubahan  dalam  nilai  rupiah  jumlah  (A) Rp14.700.000,00 
tagihan pak Amir ...  (B) Rp15.000.000,00 
(A) bertambah Rp30.000,00  (C) Rp15.300.000,00 
(B) bertambah Rp40.000,00  (D) Rp17.500.000,00 
(C) bertambah Rp60.000,00  (E) Rp20.000.000,00 
(D) berkurang Rp30.000,00   
(E) berkurang Rp60.000,00  58. Diselesaikan  pekerjaan  jasa  untuk 
  langganan  senilai  Rp100.000,00  dengan 
54. Masuknya  daging  sapi  impor  yang  lebih  pembayaran  seminggu  kemudian,  maka 
murah  akan  mengancam  pasar  daging  sapi  jurnalnya adalah ... 
di  Indonesia.  Akibat  masuknya  daging  sapi  (A) Kas   Rp100.000,00 
impor  terhadap  kurva  permintaan  dan    Pendapatan usaha  Rp100.000,00 
penawaran daging sapi lokal adalah ...  (B) Kas   Rp100.000,00 
(A) kurva  permintaan  bergeser  ke  kiri    Piutang usaha  Rp100.000,00 
bawah dan kurva penawaran tetap.   (C) Pendapatan service   Rp100.000,00 
(B) kurva  permintaan  bergeser  ke  kanan    Piutang usaha  Rp100.000,00 
atas dan kurva penawaran tetap.   (D) Piutang usaha   Rp100.000,00 
  Pendapatan service   Rp100.000,00 
(C) kurva  permintaan  bergeser  ke  kiri 
(E) Piutang usaha   Rp100.000,00 
bawah dan kurva penawaran bergeser 
  Kas   Rp100.000,00 
ke kanan bawah.  
 
(D) kurva  permintaan  bergeser  ke  kanan 
Gunakan  PETUNJUK  B  untuk  menjawab  soal 
atas dan kurva penawaran bergeser ke 
nomor 59! 
kiri atas.  
 
(E) kurva  permintaan  bergeser  ke  kiri 
59. Perusahaan  rental  komputer  menerima 
bawah dan kurva penawaran bergeser 
uang  sewa  dari  langganannya.  Dalam 
ke kiri atas.  
laporan  labar  rugi  uang  sewa  tersebut 
 
masuk dalam penghasilan lain‐lain.  
55. Bauran  pemasaran  barang  meliputi  empat 
SEBAB 
hal, yaitu ... 
Yang  termasuk  dalam  penghasilan  lain‐lain 
(A) product, price, income, distribution 
untuk  perusahaan  tersebut  antara  lain 
(B) product, price, place, promotion 
adalah bunga dan sewa.  
(C) product, income, taste, distribution 
 
(D) product, price, income, utility 
Gunakan  PETUNJUK  C  untuk  menjawab  soal 
(E) product, promotion, price, distribution 
nomor 60! 
 
 
56. Kegiatan  yang  terkait  dengan  efisiensi 
60. Transaksi  yang  dicatat  dalam  buku  jurnal 
usaha adalah melakukan .... 
penyesuaian adalah ... 
(A) ekstensifikasi usaha.  
(1) kerugian  piutang  yang 
(B) diversifikasi produk.  
diperhitungkan/dikeluarkan  dari 
(C) intensifikasi usaha.  
cadangan kerugian piutang.  
(D) pemutusan hubungan kerja.  
(2) retur  dan  potongan  pembelian  yang 
(E) peningkatan produksi.  
dikeluarkan dari pembelian.  
 
(3) gaji  karyawan  yang  dikeluarkan  dari 
57. Pada  tanggal  8  Januari  2010,  seorang 
persekot gaji.  
pedagang  eceran  menarik  pinjaman  dari 
(4) biaya sewa yang dikeluarkan dari kas.  
Bank  Pelita  Jaya  sebesar  Rp50.000.000,00 
 
dengan  bunga  12%  per  tahun.  Uang 
 

an_ay
tersebut  sepenuhnya  digunakan  sebagai 
modal usah sehingga berhasil mendapatkan 
penerimaan  penjualan  sebesar 

Halaman 2 
www.onlinelesson.org    546 
Simulasi Soal Snmptn 2011 
 
Mata Pelajaran  : Matematika  
Tanggal    : 17 Juni 2010 
Kode Soal    : 546 
 
 
Gunakan  PETUNJUK  A  untuk  menjawab  soal  5. Jumlah  50  suku  pertama  deret 
nomor 1 sampai dengan nomor 15!  log 5 + log 55 + log 605 + log 6665 + ...   adalah 
  .... 
1. Diketahui a dan b adalah dua bilangan bulat  (A) log(2525111225 )  
1 1 13
positif  yang  memenuhi  + = .  Nilai  (B) log(525111225 )  
a b 36
ab(a + b)  adalah ….  (C) log(2751150 )  
(A) 468  (D) 1150 log(5)  
(B) 448  (E) log(551150 )  
(C) 368 
 
(D) 49 
6. Diketahui  barisan  dengan  suku  pertama 
(E) 36 
u1 = 15   dan  memenuhi  un − un−1 = 2n + 3 , 
 
2. Diketahui  x < −3 .  Bentuk  yang  setara  n ≥ 2 . Nilai  U50 + U2   adalah …. 
dengan  1 − 1 + 3x  adalah ….  (A) 2688 
(B) 2710 
(A) −2 − 3x  (C) 2732 
(B) 3x  (D) 2755 
(C) −2 + 3x  (E) 2762 
(D) −3x   
(E) 2 − 3x  7. Kubus  ABCD.EFGH   panjang  sisinya  1 dm . 
  Titik P pada BC  dengan  |PC|  =  t dm . Titik 
3. Suku banyak  yang akarnya  2 − 5  adalah  Q  adalah  proyeksi  A  pada  DP  dan  R  adalah 
….  proyeksi Q pada bidang EFGH. Luas segitiga 
(A) x 4 + 14 x 2 + 9   AQR adalah … dm2 
(B) x 4 − 14 x 2 + 9   (A)
1
 
(C) x 4 − 14 x 2 − 9   2 t2 + 1
(D) x 4 + 14 x 2 + 89   1
(B)  
(E) x 4 − 14 x 2 + 89   t +1
2

  (C) 2 t2 + 1  
→ → →
4. Diketahui  a ,  b ,  dan  c   vektor  dalam  t2 − 1
(D)  
→ → → → → 2
dimensi‐3.  Jika  a ⊥ b   dan  a ⊥ ( b + 2 c ),   (E) 1 + t2  
→ → →
maka  a ⋅ (2 b − c ) = ...    
8. Manakah pernyataan berikut yang benar? 
(A) 4 
(A) Jika  cos x = cos y , maka  x = y  
(B) 2 
(C) 1  (B) Jika  sin x = sin y , maka  x = y  
(D) 0  (C) x 2 = x  untuk semua x 
(E) −1  (D) Jika  log x = log y , maka  x = y  
  (E) log x 2 = 2log x  untuk semua  x ≠ 0  
   
 

an_ay
 
 
 
 
 

Halaman 1 
www.onlinelesson.org    546 
4x 13. Diketahui selembar seng dengan panjang 80 
9. Nilai  Lim  adalah ….  cm  dan  lebar  30  cm.  Jika  panjang  dan 
x →0
sin2 x
lebarnya  dipotong  dengan  ukuran  sama 
(A) 2  sehingga  luas  seng  menjadi  275  cm2,  maka 
(B) 1  panjang dan lebarnya harus dipotong … cm 
1 (A) 30 
(C)  
2 (B) 25 
1 (C) 24 
(D)  
4 (D) 20 
(E) 0  (E) 15 
   
10. Luas  daerah  persegipanjang  terbesar  yang  14. Sejumlah  siswa  terdiri  atas  5  putra  dan  5 
dapat  dibuat  dalam  daerah  yang  dibatasi  putri membentuk panitia yang terdiri atas 4 
1 orang  siswa.  Peluang  panitia  tersebut 
kurva  y = x 2  dan  y = 5  adalah .... 
3 memuat paling banyak 2 siswa putri adalah 
16 …. 
(A) 5 
3 16
(A)  
17 21
(B) 5 
3 11
(B)  
(C) 6 5   37
19 23
(D) 5  (C)  
3 42
20 31
(E) 5  (D)  
3 42
  35
(E)  
11. Perhatikan  gambar  berikut!  Persegi  ABCD  42
dengan  panjang  sisi  10  cm.  Lingkaran   
melalui  titik  A  dan  D  dan  menyinggung  sisi  15. Integral yang menyatakan luas daerah yang 
BC. Luas lingkaran tersebut adalah … cm2  dibatasi  oleh  kurva  y = x ,  x + y − 6 = 0  
(A) 10π  dan sumbu X adalah .... 
(B) 20π  A B 6 9

(C)
625
π 
(A) ∫ x dx + ∫ ( x − 6)dx  
16 0 6
4 9
325
(D)
8
π 
C
(B) ∫ x dx − ∫ ( x − 6)dx  
D 0 4
85
(E) π  4 9

 
2 (C) ∫
0
x dx + ∫ ( x − 6)dx  
4
12. Jika  nilai  maksimum  f (x) = x + 2p − 3x   4 6

adalah 
5
, maka nilai p adalah .... 
(D) ∫
0
x dx − ∫ ( x − 6)dx  
4
4 4 6
(A) 1 
2
(E) ∫
0
x dx + ∫ ( x − 6)dx  
4
(B)  
3  
3  
(C)  
4  
3
(D)  
2
(E) 2 

an_ay
 
 

Halaman 2 
www.onlinelesson.org    546 
Simulasi Soal Snmptn 2011 
 
Mata Pelajaran  : Fisika 
Kode Soal    : 546 
 
 
Gunakan  PETUNJUK  A  untuk  menjawab  soal  19. Jika  sebuah  mesin  Carnot  yang  memiliki 
nomor 16 sampai dengan nomor 25!  efisiensi  terbesar  25%  dalam  operasinya 
  membuang sebagian kalor ke tandon dingin 
16. Sebuah benda yang diam di atas lantai licin  bertemperatur 0 °C, maka tandon panasnya 
didorong  dengan  gaya  konstan  selama  bertemperatur .... 
selang waktu  Δt , sehingga benda mencapai  (A)   76 °C 
kelajuan  v.  Bila  percobaan  diulang,  tetapi  (B)   91 °C 
dengan  besar  gaya  dua  kali  semula,  (C) 170 °C 
berapakahselang  waktu  yang  diperlukan  (D) 100 °C 
untuk mencapai kelajuan yang sama?   (E) 364 °C 
(A) 4 Δt    
(B) 2 Δt   20. Suatu  benda  titik  melakukan  osilasi 
(C) Δt   harmonik  sederhana  dengan  amplitudo  0,2 
Δt m.  Titik  tersebut  melakukan  20  getaran 
(D)  
2 penuh  dalam  satu  detiknya.  Jika  pada  saat 
Δt awal (t = 0) simpangan titik tersebut adalah 
(E)  
4 0,1  m,  maka  persamaan  gerak  osilasi 
  harmoniknya adalah .... 
17. Sebuah  batu  kecil  dilempar  ke  atas  dan  ⎛ π⎞
mendarat  di  sebuah  papan  yang  terletak      (A) x(t ) = 0,2m sin⎜ 40πt − ⎟  
⎝ 6⎠
2 m di atas titik pelemparan. Jika kecepatan 
⎛ π⎞
awal  batu  dilempar  ke  atas  adalah  7  m/s,  (B) x(t ) = 0,2m sin⎜ 40πt − ⎟  
maka  kecepatan  batu  ketika  mengenai  ⎝ 3⎠
⎛ π⎞
papan adalah ...  (C) x(t ) = 0,1m sin⎜ 40πt − ⎟  
(Petunjuk: arah ke atas adalah positif)  ⎝ 6⎠
(A) 0 m/s  ⎛ π⎞
(D) x(t ) = 0,1m sin⎜ 40πt − ⎟  
(B) –3 m/s  ⎝ 4⎠
(C) 3 m/s  π⎞

(D) 3,5 m/s  (E) x(t ) = 0,2m sin⎜ 40πt − ⎟  
⎝ 4⎠
(E) –2 m/s 
   
18. Tekanan  suatu  gas  ideal  di  dalam  suatu  21. Dawai piano yang memiliki panjang 1 m dan 
tabung  dilipatduakan  dengan  volume  bermassa  10  g  diberi  tegangan  900  N. 
dipertahankan  tetap.  Jika  gas  dianggap  Berapakah  frekuensi  nada  atas  pertama 
bersifat  ideal,  maka  perbandingan  kelajuan  yang dihasilkannya? 
rms  (vrms)  keadaan  awal  dan  keadaan  akhir  (A) 133 Hz 
adalah ....  (B) 150 Hz 
(A) 4  (C) 300 Hz 
(B) 2  (D) 450 Hz 
1 (E) 550 Hz 
(C)    
2
 
(D) 2   
1  
(E)  
2  

an_ay
   
   

Halaman 1 
www.onlinelesson.org    546 
22. Seorang  yang  berpenglihatan  dekat  tidak  Gunakan  PETUNJUK  B  untuk  menjawab  soal 
dapat  melihat  dengan  jelas  benda  yang  nomor 26 dan nomor 27!! 
berjarak lebih jauh dari 60 cm dari matanya.   
Berapakah  kuat  lensa  kacamata  yang  26. Periode putar suatu satelit dalam mengitari 
dibutuhkan  agar  ia  dapat  melihat  dengan  bumi  pada  jarak  orbit  r  bergantung  pada 
jelas?   massa bumi.  
3 SEBAB 
(A) − 1  dioptri 
2 Menurut  hukum  Keppler  periode  satelit 
2 bergantung pada jari‐jari orbitnya.  
(B) − 1  dioptri 
3  
2 27. Konduktor  adalah  bahan  yang  sangat 
(C) + 2  dioptri  mudah menghantarkan arus listrik.  
3
3 SEBAB 
(D) +  dioptri  Konduktor  memiliki  konduktivitas  listrik 
2
2 yang besar.  
(E) −  dioptri   
3
Gunakan  PETUNJUK  C  untuk  menjawab  soal 
 
nomor 28 dan nomor 30! 
23. Jika dalam suatu medium gaya yang bekerja 
 
pada  dua  buah  muatan  masing‐masing 
28. Pernyataan  berikut  yang  benar  tentang 
sebesar  10  μC  dan  4  μC,  yang  terpisah 
gaya tak konservatif adalah .... 
sejauh 10 cm adalah 12 N, maka konstanta 
(1) energi  mekanik  benda  yang 
permitivitas relatif medium tersebut adalah 
dipengaruhi  oleh  gaya  tak  konservatif 
.... 
besarnya tidak konstan 
(A) 1 
(2) kerja  yang  dilakukan  oleh  gaya  tak 
(B) 2 
konservatif  pada  benda  sama  dengan 
(C) 3 
nilai  negatif  perubahan  energi 
(D) 4 
potensial benda 
(E) 5 
(3) kerja  yang  dilakukan  oleh  gaya  tak 
 
konservatif  pada  benda  bergantung 
24. Potensial listrik di suatu titik yang berjarak r 
pada lintasan gerak benda 
dari  muatan  Q  adalah  600  V.  Intensitas 
(4) contoh  gaya  tak  konservatif  adalah 
medan  listrik  di  titik  tersebut  adalah  400 
gaya Coulomb 
N/C. 
 
Tentukan  besar  muatan  Q,  jika  k  =  9  x  109 
29. Tiga  kapasitor  dan  sumber  tegangan 
Nm2/C2! 
tersusun  seperti  gambar  di  bawah  ini. 
(A) 1,5   x 10‐7 C 
Manakah  pernyataan  di  bawah  ini  yang  
(B) 1,0   x 10‐7 C 
benar?  
(C) 2,5   x 10‐7 C 
(D) 0,67 x 10‐7 C 
(E) 9,0   x 10‐7 C 
 
25. Jumlah foton per detik yang dihasilkan oleh 
suatu  sumber  laser  yang  berpanjang   
gelombang  6,926  nm  dan  berdaya  (1) Kapasitansi  pengganti  rangkaian 
keluaran1 mW mendekati .... foton.  58
kapasitor di atas adalah  μF  
(A) 300 x 109  9
(B) 350 x 1010  (2) Muatan  pada  kapasitor  C3  adalah 
(C) 3,33 x 1010  3,6 × 10 −4 C . 
(D) 420 x 1013  (3) Energi  listrik  yang  tersimpan  dalam 
(E) 500 x 1013  kapasitor C2 adalah  4 × 10 −2 J .  
  (4) Beda  potensial  pada  kutub‐kutub 

an_ay
  kapasitor C1 dan C3 sama besar.  
 

Halaman 2 
www.onlinelesson.org    546 
30. Menurut  model  atom  Bohr  pernyataan 
berikut yang benar adalah .... 
(1) tingkatan  energi  elektron  dalam 
sebuah  atom  tidak  tergantung  pada 
nomor atom 
(2) atom  akan  memancarkan  foton  bila 
elektron  dalam  atom  tersebut 
berpindah ke tingkat energi yang lebih 
rendah 
(3) spektrum  atom  hidrogen  bersifat 
kontinyu 
(4) besarnya  energi  foton  yang 
dipancarkan  oleh  atom  sama  dengan 
selisih  tingkat  energi  elektron  dalam 
atom tersebut 
 
 
 
 
 
 

an_ay Halaman 3 
www.onlinelesson.org    548 
Simulasi Soal Snmptn 2011 
 
Mata Pelajaran  : Kimia 
Kode Soal    : 546 
 
 
Gunakan  PETUNJUK  A  untuk  menjawab  soal  (D) 2,82 K 
nomor 31 sampai dengan nomor 40!  (E) 3,86 K 
   
31. Konfigurasi ion besi (III),  26Fe3+, mempunyai  35. Data  berikut  merupakan  data  laju  reduksi 
elektron tidak berpasangan sebanyak ....  nitrogenmonoksida (NO) oleh gas hidrogen: 
(A) dua  2NO(g) + 2H2(g) → N2(g) + 2H2O(g) 
 

(B) tiga   

(C) empat  [NO]0  [H2]0  Laju awal (vo) 


(D) lima  (mol.L‐1)  (mol.L‐1)  (mol.L‐1s‐1) 
(E) enam  0,1  0,1  1,23 x 10‐3 
  0,1  0,2  2,46 x 10‐3 
32. Dalam  15,9  gram  senyawa  tembaga  (1)  0,1  0,2  4,92 x 10‐3 
 

sulfida  (Ar:  Cu  =  63,5  dan  S  =  32)  terdapat  Orde reaksi total dari reaksi tersebut adalah 


Cu sebanyak ....  .... 
(A)   6,35 g  (A) 0 
(B) 12,70 g  (B) 1 
(C) 15,90 g  (C) 2 
(D) 25,40 g  (D) 3 
(E) 31,80 g  (E) 4 
   
33. Perhatikan reaksi berikut!  36. Jika  70  ml  CH3COOH  0,1  M  (Ka  =  10‐5) 
3TiO2(s) + 4BrF3(l ) → 3TiF4(s) + 2Br2 (l ) + 3O2(g)   direaksikan  dengan  100  mL  NaOH  0,05  M, 
Bila  1,6  gram  cuplikan  yang  mengandung  maka pH larutan akhir adalah .... 
TiO2 (Ar : Ti = 48, O = 16) menghasilkan 0,16  (A) 2 – log 3 
g  O2,  maka  persentase  (%)  massa  TiO2  (B) 3 – log 4 
dalam cuplikan tersebut adalah ....  (C) 3 – log 2 
(A)   4  (D) 4 – log 6 
(B)   8  (E) 6 – log 4 
(C) 16   
(D) 20  37. Perhatikan tabel berikut! 
 

(E) 25  Asam  Nilai Ka 


 
H3PO4  7,2 x 10‐3 
34. Pembakaran glukosa (C6H12O6) dalam tubuh 
H2PO4‐  6,3 x 10‐8 
manusia  mengikuti  persamaan  reaksi 
HPO42‐  4,2 X 10‐13 
berikut :    

C6H12O6 + 6O2 → 6H2O + 6CO2    ΔH = –2820 kJ  Berdasarkan data di atas, jika perbandingan 
Dengan menganggap semua glukosa terurai  konsentrasi asam dan basa konjugasinya 1:1 
menjadi  air  dan  karbondioksida,  serta  pasangan  yang  paling  cocok  untuk 
semua  kalor  yang  dihasilkan  digunakan  membuat  larutan  penyangga  dengan  pH 
menaikkan  suhu  badan,  seseorang  dengan  sekitar 7 adalah .... 
berat badan 75 kg (kapasitas kalor spesifik =  (A) K3PO4 + K2HPO4 
4  J  K‐1  g‐1),  yang  mengkonsumsi  18  gram  (B) K3PO4 + KH2PO4 
glukosa  (Ar:  C  =  12,  O  =  16,  H  =  1),  akan  (C) H3PO4 + KH2PO4 
mengalami kenaikan suhu badan sebesar ....   (D) K2HPO4 + KH2PO4 
(A) 0,4 K  (E) H3PO4 + K2HPO4 

an_ay
(B) 0,94 K  38. Perubahan  bilangan  oksidasi  unsur  N  pada 
(C) 1,88 K  reaksi:  

Halaman 1 
www.onlinelesson.org    548 
Sn + 4HNO3 → SnO2 + 4NO2 + 2H2O  dibandingkan  dengan  kalor  yang 
adalah ....  dilepaskan  pada  pembentukan  1  mol 
(A) 1  H2O(l) 
(B) 2  (3) perubahan  entalpi  pembentukan 
(C) 3  standar gas CO adalah –110,5 kJ/mol 
(D) 4  (4) pada T dan P yagn sama, pembakaran 
(E) 5  1  mol  C(s),  1  mol  H2(g)  dan  1  mol 
  CO(g)  masing‐masing  memerlukan 
39. Hasil  reaksi  antara  2‐klorobutana  dengan  volume gas oksigen yang sama.  
NaOH  direaksikan  kembali  dengan  asam   
sulfat  pada  suhu  100  °C.  Hasil  akhir  yang  43. Sebanyak  25  mL  CH3COOH  0,1  M  tepat 
diperoleh adalah ....  dititrasi  dengan  25  mL  NaOH  0,1  M.  Jika 
(A) butana  diketahui  Ka  CH3COOH  =  10‐5,  pernyataan 
(B) butanol   yang  benar  tentang  reaksi  titrasi  tersebut 
(C) butena  adalah .... 
(D) butenol   (1) pH  larutan  asam  sebelum  titrasi 
(E) butanal   adalah 3 
  (2) pH larutan setela titrasi > 7 
40. Senyawa  yang  dihasilkan  dari  reaksi  antara  (3) CH3COONa  hasil  reaksi  mengalami 
benzena  dengan  asam  nitrat  dan  asam  hidrolisis 
sulfat pekat pada suhu 50 °C adalah ....  (4) konsentrasi Na+ dalam campuran 0,05 
(A) nitrobenzena  M 
(B) asam benzena sulfonat   
(C) asam benzoat  Gunakan  PETUNJUK  B  untuk  menjawab  soal 
(D) amino benzena  nomor 44 dan nomor 45.! 
(E) nitro benzena sulfonat   
  44.  Pada kesetimbangan: 
  N2(g) + O2(g) ' 2NO(g)  ΔHo = +180 kJ,  
Gunakan  PETUNJUK  C  untuk  menjawab  soal  Jumlah  NO(g)  yang  terbentuk  akan  lebih 
nomor 41 sampai dengan nomor 43!  besar pada temperatur yang lebih tinggi.  
  SEBAB 
41. senyawa‐senyawa  berikut  ini  dibentuk  dari  Dalam  reaksi  endotermis,  keadaan 
atom‐atom  1H,  5B,  6C,  17N,  9F  dan  16S.  kesetimbangan  bergeser  ke  kanan  jika 
Molekul‐molekul yang menggunakan orbital  temperatur dinaikkan.  
hibrida sp3 pada atom pusatnya adalah ....   
(1) BF3  45. Apabila  sepotong  logam  alumunium 
(2) NH3  dimasukkan  ke  dalam  larutan  MgCl2  1  M, 
(3) SF4  unsur magnesium akan mengendap.  
(4) CH4  (Eo Mg2+/Mg = –2,356 V; Eo Al3+/Al = –1,676 V). 
  SEBAB 
42. Perhatikan data berikut!  Oksigen  lebih  mudah  mengoksidasi 
(I) C(s) + O2(g)  →  CO2(g)   ΔH = –394 kJ  magnesium  dibandingkan  dengan  oksigen 
(II) 2H2(g) + O2(g) → 2H2O(l)   ΔH = –572 kJ  mengoksidasi  alumunium  (Eo  O2/H2O  = 
(III) 2CO(g) + O2(g) → 2CO2(g)   ΔH = –567 kJ  +1,23 V). 
 
 
Pernyataan  yang  benar  tentang  reaksi‐
 
reaksi di atas adalah ... 
 
(1) kalor  yang  dilepaskan  pada 
pembakaran  1  mol  C(s)  lebih  besar 
dibandingkan  dengan  kalor  yang 
dilepaskan  pada  pembakaran  1  mol 
CO(g) 

an_ay
(2) kalor  yang  dilepaskan  pada 
pembentukan 1 mol CO2(g) lebih kecil 

Halaman 2 
www.onlinelesson.org    546 
Simulasi Soal Snmptn 2011 
 
Mata Pelajaran  : Biologi 
Kode Soal    : 546 
 
 
Gunakan  PETUNJUK  A  untuk  menjawab  soal  (C) lingkungan  mempengaruhi  perubahan 
nomor 46 sampai dengan nomor 55!  gen pada makhluk hidup  
  (D) pertambahan  populasi  tidak  berjalan 
46. Pada  mitosis  terjadi  peristiwa  berikut,  terus‐menerus 
kecuali ....  (E) perkembangbiakan  memerlukan 
(A) penduplikasian kromosom   makanan dan ruang yang cukup 
(B) pembelahan awal induk sel gamet   
(C) perbanyakan sel‐sel somatik   50. Perhatikan tabel di bawah ini! 
 

(D) pengembangan  sistem  informasi  Peran bagi 


Jenis Jamur  Produksi 
seluler  manusia 
(E) pemisahan  kromosom  berpasangan  1. Aspergillus  A. alkohol  P. racun 
menjadi tunggal   2. Saccharomyces  B. aflatoksin  Q. antibodi 
3. Rhizopus  C. sake  R. minuman 
   

47. Keanekaragaman  genetik  pada  suatu  Dari  tabel  di  atas  yang  menunjukkan 
populasi organisme akan meningkat jika ....  hubungan  yang  benar  antara  jenis  jamur, 
(A) habitat yang ditempati makin luas  produksi,  dan  peranannya  bagi  manusia 
(B) terdapat dimorfisme seksual   adalah .... 
(C) ukuran populasi meningkat  (A) 1 – B – P 
(D) terjadi migrasi gen   (B) 1 – B – R 
(E) terjadi evolusi   (C) 2 – A – P  
  (D) 2 – B – P  
48. Pernyataan  berikut  yang  benar  tentang  (E) 3 – C – P  
daur  ulang  limbah  nontoksik  suatu   
organisme adalah ....  51. Perhatikan ciri‐ciri tumbuhan di bawah ini:  
(A) ammonia  hasil  metabolisme  protein  a. hiasan bunga spiral  
digunakan  cacing  tanah  untuk  b. bakal buah tenggelam  
mensintesis asam amino  c. biji bersayap  
(B) molekul  air  hasil  respirasi  digunakan  d. memiliki sulur 
oleh  hewan  menggantikan  air  yang  Di  antara  ciri‐ciri  tumbuhan  di  atas,  yang 
hilang melalui kulit  termasuk ciri suku Cucurbitaceae adalah .... 
(C) karbondioksida  hasil  respirasi  (A) b dan c 
digunakan  oleh  belalang  untuk  (B) b dan d 
menghasilkan oksigen   (C) a dan b 
(D) asam  organik  hasil  fotosintesis  (D) a dan c 
digunakan  tumbuhan  hijau  untuk  (E) a dan d 
mensintesis glikogen    
(E) oksidasi  hasil  fotosintesis  fitoplankton  52. Energi  yang  dihasilkan  dari  makanan  dapat 
digunakan  hewan  laut  untuk  disimpan  dalam  waktu  yang  lama  di  dalam 
menyusun energi   tubuh dalam bentuk .... 
  (A) protein yang terserap sel  
49. Pernyataan  berikut  merupakan  pokok‐ (B) panas yang dilepaskan tubuh  
pokok  pemikiran  yang  mendasari  hipotesis  (C) ATP yang tersimpan dalam sel 
Darwin tentang seleksi alam, kecuali ....  (D) lemak yang tersimpan di bawah kulit 
(A) tidak ada dua individu yang identik  (E)  karbohidrat yang terserap sel  
(B) setiap  ukuran  populasi  cenderung   

an_ay
bertambah    
53. Berikut  ini  yang  merupakan  hasil  reaksi 
gelap fotosintesis adalah .... 
Halaman 1 
www.onlinelesson.org    546 
(A) NADPH, Pi, dan ATP   
(B) CO2, NADPH, dan ATP  58. Dalam suatu siklus hidup organisme, mitosis 
(C) CO2, ATP, dan glukosa  terlibat dalam proses .... 
(D) H2O, CO2, dan cahaya  (1) pertumbuhan dan perkembangan 
(E) NADP+, ADP, dan glukosa  (2) perbaikan jaringan yang rusak  
  (3) gametogenesis 
54. Seorang  laki‐laki  dengan  sifat  hemofili  (4) diferensiasi 
menikah  dengan  perempuan  yang  tidak   
membawa  sifat  hemofili.  Kemungkinan  59. Lebah tidak digolongkan dalam takson yang 
anak  pertama  mereka  laki‐laki  hemofili  sama dengan kutu buku sebab lebah .... 
sebesar ....  (1) metamorfosisnya tidak sempurna 
(A) 0%  (2) tidak mengalami metamorfosis 
(B) 25%  (3) tidak mempunyai kelenjar bau  
(C) 50%  (4) metamorfosisnya sempurna 
(D) 75%   
(E) 100%  60. Pernyataan  berikut  yang  benar  terkait 
  dengan dinamika populasi adalah .... 
55. Tabel  berikut  menunjukkan  organel  dan  (1) ukuran  populasi  bergantung 
fungsi organel.   kemampuan adaptasi anggotanya 
  Organel     Fungsi   (2) kestabilan  populasi  dalam  kurun 
I  Ribosom   a  menghancurkan  waktu tertentu 
organel yang rusak  (3) pola  penyebaran  dan  kepadatan 
II  Mitokondria   b  menghasilkan  energi  mempengaruhi dinamika populasi  
melalui  metabolisme 
(4) kematian  dan  emigrasi  memperbesar 
aerobik 
III  Lisosom   c  sintesis protein 
ukuran populasi  
IV  Aparatus  d  Mengumpulkan  bahan   
Golgi  dan  mensekresikan  ke   
luar sel   
 

Manakah  dari  pasangan  organel  dan   


fungsinya yang benar? 
(A) I‐d, II‐a, III‐b, dan IV‐c 
(B) I‐a, II‐b, III‐c, dan IV‐d 
(C) I‐c, II‐b, III‐a, dan IV‐d 
(D) I‐c, II‐b, III‐d, dan IV‐a 
(E) I‐b, II‐c, III‐d, dan IV‐a 
 
Gunakan  PETUNJUK  B  untuk  menjawab  soal 
nomor 56 sampai dengan nomor 57! 
 
56. Kekurangan  unsur  tembaga  (Cu) 
menyebabkan  anemia.  
SEBAB 
Tembaga  diperlukan  sel  untuk 
mengakitfkan rantai transport elektron.  
 
57. Polinasi  pada  tanaman  berumah  dua  dapat 
terjadi secara autogami.  
SEBAB 
Tanaman  berumah  dua  mempunyai  bunga 
jantan dan betina yang terpisah dalam satu 
pohon.  

an_ay
Gunakan  PETUNJUK  B  untuk  menjawab  soal 
nomor 58 sampai dengan  nomor 60.  

Halaman 2 
www.onlinelesson.org    548 
Simulasi Soal Snmptn 2011 
 
Mata Pelajaran  : Matematika  
Kode Soal    : 548 
 
 
Gunakan  PETUNJUK  A  untuk  menjawab  soal  (A) 368 
nomor 1 sampai dengan nomor 15!  (B) 448 
  (C) 468 
1. Jumlah  50  suku  pertama  deret  (D) 49 
log 5 + log 55 + log 605 + log 6665 + ...   adalah  (E) 36 
....   
(A) log(2525111225 )   6. Integral yang menyatakan luas daerah yang 
dibatasi  oleh  kurva  y = x ,  x + y − 6 = 0  
(B) log(525111225 )  
dan sumbu X adalah .... 
(C) log(2751150 )   6 9
(D) 1150 log(5)   (A) ∫ x dx + ∫ ( x − 6)dx  
(E) log(551150 )   0 6
4 9
 
2. Diketahui  x < −3 .  Bentuk  yang  setara 
(B) ∫
0
x dx − ∫ ( x − 6)dx  
4
dengan  1 − 1 + 3x  adalah ….  4 9

(A) 3x  (C) ∫ x dx + ∫ ( x − 6)dx  


(B) −3x 
0 4
4 6
(C) 2 − 3x 
(D) −2 + 3x 
(D) ∫
0
x dx − ∫ ( x − 6)dx  
4
(E) −2 − 3x  4 6
  (E) ∫ x dx + ∫ ( x − 6)dx  
3. Suku banyak  yang akarnya  2 − 5  adalah  0 4

….   
(A) x 4 − 14 x 2 + 9   7. Kubus  ABCD.EFGH   panjang  sisinya  1 dm . 
Titik P pada BC  dengan  |PC|  =  t dm . Titik 
(B) x 4 − 14 x 2 − 9  
Q  adalah  proyeksi  A  pada  DP  dan  R  adalah 
(C) x 4 + 14 x 2 + 9  
proyeksi Q pada bidang EFGH. Luas segitiga 
(D) x 4 − 14 x 2 + 89   AQR adalah … dm2 
(E) x 4 + 14 x 2 + 89   1
  (A)  
→ → →
2 t2 + 1
4. Diketahui  a ,  b ,  dan  c   vektor  dalam  (B) 2 t2 + 1  
→ → → → →
1
dimensi‐3.  Jika  a ⊥ b   dan  a ⊥ ( b + 2 c ),   (C)  
→ → → t +1
2

maka  a ⋅ (2 b − c ) = ...   t2 − 1
(A) −1  (D)  
2
(B) 0  (E) 1 + t2  
(C) 1   
(D) 2   
(E) 4  8. Perhatikan  gambar  berikut!  Persegi  ABCD 
  dengan  panjang  sisi  10  cm.  Lingkaran 
5. Diketahui a dan b adalah dua bilangan bulat  melalui  titik  A  dan  D  dan  menyinggung  sisi 
1 1 13 BC. Luas lingkaran tersebut adalah … cm2 
positif  yang  memenuhi  + = .  Nilai 

an_ay
a b 36 (A) 10π 
ab(a + b)  adalah ….  (B) 20π  A B

Halaman 1 

D C
www.onlinelesson.org    548 
625 3
(C) π  (C)  
16 4
325 3
(D) π  (D)  
8 2
85 (E) 2 
(E) π 
2  
  13. Diketahui selembar seng dengan panjang 80 
4x cm  dan  lebar  30  cm.  Jika  panjang  dan 
9. Nilai  Lim  adalah ….  lebarnya  dipotong  dengan  ukuran  sama 
x →0
sin2 x
sehingga  luas  seng  menjadi  275  cm2,  maka 
(A) 2  panjang dan lebarnya harus dipotong … cm 
(B) 1  (A) 30 
1 (B) 25 
(C)  
2 (C) 24 
1 (D) 20 
(D)  
4 (E) 15 
(E) 0   
  14. Sejumlah  siswa  terdiri  atas  5  putra  dan  5 
10. Luas  daerah  persegipanjang  terbesar  yang  putri membentuk panitia yang terdiri atas 4 
dapat  dibuat  dalam  daerah  yang  dibatasi  orang  siswa.  Peluang  panitia  tersebut 
1 memuat paling banyak 2 siswa putri adalah 
kurva  y = x 2  dan  y = 5  adalah ....  …. 
3
16
(A) 6 5   (A)  
16 21
(B) 5  11
3 (B)  
17 37
(C) 5  23
3 (C)  
19 42
(D) 5  31
3 (D)  
20 42
(E) 5  35
3 (E)  
  42
11. Manakah pernyataan berikut yang benar?   
(A) Jika  cos x = cos y , maka  x = y   15. Diketahui  barisan  dengan  suku  pertama 
(B) Jika  sin x = sin y , maka  x = y   u1 = 15   dan  memenuhi  un − un−1 = 2n + 3 , 
n ≥ 2 . Nilai  U50 + U2   adalah …. 
(C) x 2 = x  untuk semua x 
(A) 2688 
(D) Jika  log x = log y , maka  x = y  
(B) 2710 
(E) log x 2 = 2log x  untuk semua  x ≠ 0   (C) 2732 
  (D) 2755 
  (E) 2762 
 
 
 
 
12. Jika  nilai  maksimum  f (x) = x + 2p − 3x  
5
adalah  , maka nilai p adalah .... 
4
(A) 1 

an_ay
2
(B)  
3

Halaman 2 
www.onlinelesson.org    548 
Simulasi Soal Snmptn 2011 
 
Mata Pelajaran  : Fisika 
Kode Soal    : 548 
 
 
Gunakan  PETUNJUK  A  untuk  menjawab  soal  19. Dawai piano yang memiliki panjang 1 m dan 
nomor 16 sampai dengan nomor 25!  bermassa  10  g  diberi  tegangan  900  N. 
  Berapakah  frekuensi  nada  atas  pertama 
16. Sebuah  batu  kecil  dilempar  ke  atas  dan  yang dihasilkannya? 
mendarat  di  sebuah  papan  yang  terletak      (A) 133 Hz 
2 m di atas titik pelemparan. Jika kecepatan  (B) 150 Hz 
awal  batu  dilempar  ke  atas  adalah  7  m/s,  (C) 300 Hz 
maka  kecepatan  batu  ketika  mengenai  (D) 450 Hz 
papan adalah ....  (E) 550 Hz 
(Petunjuk: arah ke atas adalah positif)   
(A) 0 m/s  20. Suatu  benda  titik  melakukan  osilasi 
(B) –3 m/s  harmonik  sederhana  dengan  amplitudo  0,2 
(C) 3 m/s  m.  Titik  tersebut  melakukan  20  getaran 
(D) 3,5 m/s  penuh  dalam  satu  detiknya.  Jika  pada  saat 
(E) –2 m/s  awal (t = 0) simpangan titik tersebut adalah 
  0,1  m,  maka  persamaan  gerak  osilasi 
17. Sebuah benda yang diam di atas lantai licin  harmoniknya adalah .... 
didorong  dengan  gaya  konstan  selama  ⎛ π⎞
(A) x(t ) = 0,2m sin⎜ 40πt − ⎟  
selang waktu  Δt , sehingga benda mencapai  ⎝ 6⎠
kelajuan  v.  Bila  percobaan  diulang,  tetapi  π⎞

dengan  besar  gaya  dua  kali  semula,  (B) x(t ) = 0,2m sin⎜ 40πt − ⎟  
⎝ 3⎠
berapakahselang  waktu  yang  diperlukan 
⎛ π⎞
untuk mencapai kelajuan yang sama?   (C) x(t ) = 0,1m sin⎜ 40πt − ⎟  
(A) 4 Δt   ⎝ 6⎠
(B) 2 Δt   ⎛ π⎞
(D) x(t ) = 0,1m sin⎜ 40πt − ⎟  
(C) Δt   ⎝ 4⎠
Δt ⎛ π⎞
(D)   (E) x(t ) = 0,2m sin⎜ 40πt − ⎟  
2 ⎝ 4⎠
Δt
(E)    
4 21. Jika  sebuah  mesin  Carnot  yang  memiliki 
  efisiensi  terbesar  25%  dalam  operasinya 
18. Tekanan  suatu  gas  ideal  di  dalam  suatu  membuang sebagian kalor ke tandon dingin 
tabung  dilipatduakan  dengan  volume  bertemperatur 0 °C, maka tandon panasnya 
dipertahankan  tetap.  Jika  gas  dianggap  bertemperatur .... 
bersifat  ideal,  maka  perbandingan  kelajuan 
(A)   76 °C 
rms  (vrms)  keadaan  awal  dan  keadaan  akhir 
(B)   91 °C 
adalah .... 
(C) 170 °C 
(A) 4 
(D) 100 °C 
(B) 2 
1 (E) 364 °C 
(C)    
2
 
(D) 2   
1  
(E)  
2  

an_ay
   
  22. Seorang  yang  berpenglihatan  dekat  tidak 
dapat  melihat  dengan  jelas  benda  yang 
Halaman 1 
www.onlinelesson.org    548 
berjarak lebih jauh dari 60 cm dari matanya.  26. Menurut  model  atom  Bohr  pernyataan 
Berapakah  kuat  lensa  kacamata  yang  berikut yang benar adalah .... 
dibutuhkan  agar  ia  dapat  melihat  dengan  (1) tingkatan  energi  elektron  dalam 
jelas?   sebuah  atom  tidak  tergantung  pada 
2 nomor atom 
(A) −  dioptri 
3 (2) atom  akan  memancarkan  foton  bila 
3 elektron  dalam  atom  tersebut 
(B) − 1  dioptri 
2 berpindah ke tingkat energi yang lebih 
2 rendah 
(C) − 1  dioptri  (3) spektrum  atom  hidrogen  bersifat 
3
2 kontinyu 
(D) + 2  dioptri  (4) besarnya  energi  foton  yang 
3
3 dipancarkan  oleh  atom  sama  dengan 
(E) +  dioptri  selisih  tingkat  energi  elektron  dalam 
2
atom tersebut 
 
 
23. Jika dalam suatu medium gaya yang bekerja 
27. Tiga  kapasitor  dan  sumber  tegangan 
pada  dua  buah  muatan  masing‐masing 
tersusun  seperti  gambar  di  bawah  ini. 
sebesar  10  μC  dan  4  μC,  yang  terpisah 
Manakah  pernyataan  di  bawah  ini  yang  
sejauh 10 cm adalah 12 N, maka konstanta 
benar?  
permitivitas relatif medium tersebut adalah 
.... 
(A) 5 
(B) 4 
(C) 3 
(D) 2   
(E) 1  (1) Kapasitansi  pengganti  rangkaian 
  58
kapasitor di atas adalah  μF  
24. Potensial listrik di suatu titik yang berjarak r  9
dari  muatan  Q  adalah  600  V.  Intensitas  (2) Muatan  pada  kapasitor  C3  adalah 
medan  listrik  di  titik  tersebut  adalah  400  3,6 × 10−4 C . 
N/C.  (3) Energi  listrik  yang  tersimpan  dalam 
Tentukan  besar  muatan  Q,  jika  k  =  9  x  109  kapasitor C2 adalah  4 × 10 −2 J .  
Nm2/C2!  (4) Beda  potensial  pada  kutub‐kutub 
(A) 1,0   x 10‐7 C  kapasitor C1 dan C3 sama besar.  
(B) 1,5   x 10‐7 C   
(C) 2,5   x 10‐7 C  28. Pernyataan  berikut  yang  benar  tentang 
(D) 9,0   x 10‐7 C  gaya tak konservatif adalah .... 
(E) 0,67 x 10‐7 C  (1) energi  mekanik  benda  yang 
  dipengaruhi  oleh  gaya  tak  konservatif 
25. Jumlah foton per detik yang dihasilkan oleh  besarnya tidak konstan 
suatu  sumber  laser  yang  berpanjang  (2) kerja  yang  dilakukan  oleh  gaya  tak 
gelombang  6,926  nm  dan  berdaya  konservatif  pada  benda  sama  dengan 
keluaran1 mW mendekati .... foton.  nilai  negatif  perubahan  energi 
(A) 3,10 x 1010  potensial benda 
(B) 3,33 x 1010  (3) kerja  yang  dilakukan  oleh  gaya  tak 
(C) 300 x 109  konservatif  pada  benda  bergantung 
(D) 420 x 1013  pada lintasan gerak benda 
(E) 500 x 1013  (4) contoh  gaya  tak  konservatif  adalah 
  gaya Coulomb 
   
Gunakan  PETUNJUK  C  untuk  menjawab  soal  Gunakan  PETUNJUK  B  untuk  menjawab  soal 

an_ay
nomor 26 sampai dengan nomor 28!  nomor 29 dan nomor 30! 
   

Halaman 2 
www.onlinelesson.org    548 
29. Konduktor  adalah  bahan  yang  sangat 
mudah menghantarkan arus listrik.  
SEBAB 
Konduktor  memiliki  konduktivitas  listrik 
yang besar.  
 
30. Periode putar suatu satelit dalam mengitari 
bumi  pada  jarak  orbit  r  bergantung  pada 
massa bumi.  
SEBAB 
Menurut  hukum  Keppler  periode  satelit 
bergantung pada jari‐jari orbitnya.  
 

an_ay Halaman 3 
www.onlinelesson.org    548 
Simulasi Soal Snmptn 2011 
 
Mata Pelajaran  : Kimia 
Kode Soal    : 548 
 
 
Gunakan  PETUNJUK  A  untuk  menjawab  soal  (D) 2,82 K 
nomor 31 sampai dengan nomor 40!  (E) 3,86 K 
   
31. Konfigurasi ion besi (III),  26Fe3+, mempunyai  35. Perhatikan tabel berikut! 
 

elektron tidak berpasangan sebanyak ....  Asam  Nilai Ka 


(A) enam  H3PO4  7,2 x 10‐3 
(B) lima  H2PO4‐  6,3 x 10‐8 
(C) empat 
HPO42‐  4,2 X 10‐13 
(D) tiga   

(E) dua  Berdasarkan data di atas, jika perbandingan 


  konsentrasi asam dan basa konjugasinya 1:1 
32. Perhatikan reaksi berikut!  pasangan  yang  paling  cocok  untuk 
3TiO2(s) + 4BrF3(l ) → 3TiF4(s) + 2Br2 (l ) + 3O2(g)   membuat  larutan  penyangga  dengan  pH 
Bila  1,6  gram  cuplikan  yang  mengandung  sekitar 7 adalah .... 
TiO2 (Ar : Ti = 48, O = 16) menghasilkan 0,16  (A) K3PO4 + K2HPO4 
g  O2,  maka  persentase  (%)  massa  TiO2  (B) K3PO4 + KH2PO4 
dalam cuplikan tersebut adalah ....  (C) H3PO4 + KH2PO4 
(A) 4  (D) K2HPO4 + KH2PO4 
(B) 8  (E) H3PO4 + K2HPO4 
(C) 16   
(D) 20  36. Jika  70  ml  CH3COOH  0,1  M  (Ka  =  10‐5) 
(E) 25  direaksikan  dengan  100  mL  NaOH  0,05  M, 
  maka pH larutan akhir adalah .... 
33. Dalam  15,9  gram  senyawa  tembaga  (1)  (A) 2 – log 3 
sulfida  (Ar:  Cu  =  63,5  dan  S  =  32)  terdapat  (B) 3 – log 4 
Cu sebanyak ....  (C) 3 – log 2 
(A) 6,35 g  (D) 4 – log 6 
(B) 12,70 g  (E) 6 – log 4 
(C) 15,90 g   
(D) 25,40 g  37. Data  berikut  merupakan  data  laju  reduksi 
(E) 31,80 g  nitrogenmonoksida (NO) oleh gas hidrogen: 
  2NO(g) + 2H2(g) → N2(g) + 2H2O(g) 
 

34. Pembakaran glukosa (C6H12O6) dalam tubuh   

[NO]0  [H2]0  Laju awal (vo) 


manusia  mengikuti  persamaan  reaksi 
(mol.L‐1)  (mol.L‐1)  (mol.L‐1s‐1) 
berikut :  
C6H12O6 + 6O2 → 6H2O + 6CO2    ΔH = –2820 kJ 
0,1  0,1  1,23 x 10‐3 
Dengan menganggap semua glukosa terurai  0,1  0,2  2,46 x 10‐3 
menjadi  air  dan  karbondioksida,  serta  0,1  0,2  4,92 x 10‐3 
 

semua  kalor  yang  dihasilkan  digunakan  Orde reaksi total dari reaksi tersebut adalah 


menaikkan  suhu  badan,  seseorang  dengan  .... 
berat badan 75 kg (kapasitas kalor spesifik =  (A) 4 
4  J  K‐1  g‐1),  yang  mengkonsumsi  18  gram  (B) 3 
glukosa  (Ar:  C  =  12,  O  =  16,  H  =  1),  akan  (C) 2 
mengalami kenaikan suhu badan sebesar ....   (D) 1 
(A) 0,4 K  (E) 0 

an_ay
(B) 0,94 K  38. Perubahan  bilangan  oksidasi  unsur  N  pada 
(C) 1,88 K  reaksi:  

Halaman 1 
www.onlinelesson.org    548 
Sn + 4HNO3 → SnO2 + 4NO2 + 2H2O  (II) 2H2(g) + O2(g) → 2H2O(l)   ΔH = –572 kJ 
adalah ....  (III) 2CO(g) + O2(g) → 2CO2(g)  ΔH = –567 kJ 
 

(A) 5  Pernyataan  yang  benar  tentang  reaksi‐


(B) 4  reaksi di atas adalah ... 
(C) 3  (1) kalor  yang  dilepaskan  pada 
(D) 2  pembakaran  1  mol  C(s)  lebih  besar 
(E) 1  dibandingkan  dengan  kalor  yang 
  dilepaskan  pada  pembakaran  1  mol 
39. Senyawa  yang  dihasilkan  dari  reaksi  antara  CO(g) 
benzena  dengan  asam  nitrat  dan  asam  (2) kalor  yang  dilepaskan  pada 
sulfat pekat pada suhu 50 °C adalah ....  pembentukan 1 mol CO2(g) lebih kecil 
(A) nitrobenzena  dibandingkan  dengan  kalor  yang 
(B) asam benzena sulfonat  dilepaskan  pada  pembentukan  1  mol 
(C) asam benzoat  H2O(l) 
(D) amino benzena  (3) perubahan  entalpi  pembentukan 
(E) nitro benzena sulfonat  standar gas CO adalah –110,5 kJ/mol 
  (4) pada T dan P yagn sama, pembakaran 
40. Hasil  reaksi  antara  2‐klorobutana  dengan  1  mol  C(s),  1  mol  H2(g)  dan  1  mol 
NaOH  direaksikan  kembali  dengan  asam  CO(g)  masing‐masing  memerlukan 
sulfat  pada  suhu  100  °C.  Hasil  akhir  yang  volume gas oksigen yang sama.  
diperoleh adalah ....   
(A) butana  Gunakan  PETUNJUK  B  untuk  menjawab  soal 
(B) butanol   nomor 44 dan nomor 45.! 
(C) butena   
(D) butenol   44.  Apabila  sepotong  logam  alumunium 
(E) butanal   dimasukkan  ke  dalam  larutan  MgCl2  1  M, 
  unsur magnesium akan mengendap.  
Gunakan  PETUNJUK  C  untuk  menjawab  soal  (Eo Mg2+/Mg = –2,356 V; Eo Al3+/Al = –1,676 V). 
nomor 41 sampai dengan nomor 43!  SEBAB 
  Oksigen  lebih  mudah  mengoksidasi 
41. senyawa‐senyawa  berikut  ini  dibentuk  dari  magnesium  dibandingkan  dengan  oksigen 
atom‐atom  1H,  5B,  6C,  17N,  9F  dan  16S.  mengoksidasi  alumunium  (Eo  O2/H2O  = 
Molekul‐molekul yang menggunakan orbital  +1,23 V). 
hibrida sp3 pada atom pusatnya adalah ....   
(1) BF3  45. Pada kesetimbangan: 
(2) NH3  N2(g) + O2(g) ' 2NO(g)  ΔHo = +180 kJ,  
(3) SF4  Jumlah  NO(g)  yang  terbentuk  akan  lebih 
(4) CH4  besar pada temperatur yang lebih tinggi.  
  SEBAB 
42. Sebanyak  25  mL  CH3COOH  0,1  M  tepat  Dalam  reaksi  endotermis,  keadaan 
dititrasi  dengan  25  mL  NaOH  0,1  M.  Jika  kesetimbangan  bergeser  ke  kanan  jika 
diketahui  Ka  CH3COOH  =  10‐5,  pernyataan  temperatur dinaikkan.  
yang  benar  tentang  reaksi  titrasi  tersebut   
adalah ....   
(1) pH  larutan  asam  sebelum  titrasi 
adalah 3 
(2) pH larutan setela titrasi > 7 
(3) CH3COONa  hasil  reaksi  mengalami 
hidrolisis 
(4) konsentrasi Na+ dalam campuran 0,05 

an_ay
 
43. Perhatikan data berikut! 
(I) C(s) + O2(g)  →  CO2(g)   ΔH = –394 kJ 
Halaman 2 
www.onlinelesson.org    548 
Simulasi Soal Snmptn 2011 
 
Mata Pelajaran  : Biologi 
Kode Soal    : 548 
 
 
Gunakan  PETUNJUK  A  untuk  menjawab  soal  (E) perkembangbiakan  memerlukan 
nomor 46 sampai dengan nomor 55!  makanan dan ruang yang cukup 
   
46. Seorang  laki‐laki  dengan  sifat  hemofili  50. Pernyataan  berikut  yang  benar  tentang 
menikah  dengan  perempuan  yang  tidak  daur  ulang  limbah  nontoksik  suatu 
membawa  sifat  hemofili.  Kemungkinan  organisme adalah .... 
anak  pertama  mereka  laki‐laki  hemofili  (A) ammonia  hasil  metabolisme  protein 
sebesar ....  digunakan  cacing  tanah  untuk 
(A) 0%  mensintesis asam amino 
(B) 25%  (B) molekul  air  hasil  respirasi  digunakan 
(C) 50%  oleh  hewan  menggantikan  air  yang 
(D) 75%  hilang melalui kulit 
(E) 100%  (C) karbondioksida  hasil  respirasi 
  digunakan  oleh  belalang  untuk 
47. Keanekaragaman  genetik  pada  suatu  menghasilkan oksigen  
populasi organisme akan meningkat jika ....  (D) asam  organik  hasil  fotosintesis 
(A) habitat yang ditempati makin luas  digunakan  tumbuhan  hijau  untuk 
(B) terdapat dimorfisme seksual   mensintesis glikogen  
(C) ukuran populasi meningkat  (E) oksidasi  hasil  fotosintesis  fitoplankton 
(D) terjadi migrasi gen   digunakan  hewan  laut  untuk 
(E) terjadi evolusi   menyusun energi  
   
48. Perhatikan tabel di bawah ini!  51. Perhatikan ciri‐ciri tumbuhan di bawah ini:  
 

Peran bagi  a. hiasan bunga spiral  
Jenis Jamur  Produksi 
manusia  b. bakal buah tenggelam  
1. Aspergillus  A. alkohol  P. racun  c. biji bersayap  
2. Saccharomyces  B. aflatoksin  Q. antibodi  d. memiliki sulur 
3. Rhizopus  C. sake  R. minuman 
  Di  antara  ciri‐ciri  tumbuhan  di  atas,  yang 
Dari  tabel  di  atas  yang  menunjukkan  termasuk ciri suku Cucurbitaceae adalah .... 
hubungan  yang  benar  antara  jenis  jamur,  (A) b dan c 
produksi,  dan  peranannya  bagi  manusia  (B) b dan d 
adalah ....  (C) a dan b 
(A) 1 – B – P  (D) a dan c 
(B) 1 – B – R  (E) a dan d 
(C) 2 – A – P    
(D) 2 – B – P   52. Energi  yang  dihasilkan  dari  makanan  dapat 
(E) 3 – C – P   disimpan  dalam  waktu  yang  lama  di  dalam 
  tubuh dalam bentuk .... 
49. Pernyataan  berikut  merupakan  pokok‐ (A) protein yang terserap sel  
pokok  pemikiran  yang  mendasari  hipotesis  (B) panas yang dilepaskan tubuh  
Darwin tentang seleksi alam, kecuali ....  (C) ATP yang tersimpan dalam sel 
(A) tidak ada dua individu yang identik  (D) lemak yang tersimpan di bawah kulit 
(B) setiap  ukuran  populasi  cenderung  (E)  karbohidrat yang terserap sel  
bertambah   53. Berikut  ini  yang  merupakan  hasil  reaksi 
(C) lingkungan  mempengaruhi  perubahan  gelap fotosintesis adalah .... 

an_ay
gen pada makhluk hidup   (A) NADPH, Pi, dan ATP 
(D) pertambahan  populasi  tidak  berjalan  (B) CO2, NADPH, dan ATP 
terus‐menerus 
Halaman 1 
www.onlinelesson.org    548 
(C) CO2, ATP, dan glukosa  (3) tidak mempunyai kelenjar bau  
(D) H2O, CO2, dan cahaya  (4) metamorfosisnya sempurna 
(E) NADP+, ADP, dan glukosa   
  58. Dalam suatu siklus hidup organisme, mitosis 
54. Pada  mitosis  terjadi  peristiwa  berikut,  terlibat dalam proses .... 
kecuali ....  (1) pertumbuhan dan perkembangan 
(A) penduplikasian kromosom   (2) perbaikan jaringan yang rusak  
(B) pembelahan awal induk sel gamet  (3) gametogenesis 
(C) perbanyakan sel‐sel somatik   (4) diferensiasi 
(D) pengembangan  sistem  informasi   
seluler  Gunakan  PETUNJUK  B  untuk  menjawab  soal 
(E) pemisahan  kromosom  berpasangan  nomor 59 dan nomor 60.  
menjadi tunggal    
  59. Kekurangan  unsur  tembaga  (Cu) 
55. Tabel  berikut  menunjukkan  organel  dan  menyebabkan anemia.  
fungsi organel.   SEBAB 
 

  Organel     Fungsi   Tembaga  diperlukan  sel  untuk 


I  Ribosom   a  menghancurkan  mengaktifkan rantai transpor elektron.  
organel yang rusak   
II  Mitokondria   b  menghasilkan  energi  60. Polinasi  pada  tanaman  berumah  dua  dapat 
melalui  metabolisme  terjadi secara autogami.  
aerobik  SEBAB 
III  Lisosom   c  sintesis protein  Tanaman  berumah  dua  mempunyai  bunga 
IV  Aparatus  d  Mengumpulkan  bahan 
jantan dan betina yang terpisah dalam satu 
Golgi  dan  mensekresikan  ke 
luar sel 
pohon.  
 
 
Manakah  dari  pasangan  organel  dan   
fungsinya yang benar? 
(A) I‐d, II‐a, III‐b, dan IV‐c 
(B) I‐a, II‐b, III‐c, dan IV‐d 
(C) I‐c, II‐b, III‐a, dan IV‐d 
(D) I‐c, II‐b, III‐d, dan IV‐a 
(E) I‐b, II‐c, III‐d, dan IV‐a 
 
Gunakan  PETUNJUK  C  untuk  menjawab  soal 
nomor 56 sampai dengan nomor 58! 
 
56. Pernyataan  berikut  yang  benar  terkait 
dengan dinamika populasi adalah .... 
(1) ukuran  populasi  bergantung 
kemampuan adaptasi anggotanya 
(2) kestabilan  populasi  dalam  kurun 
waktu tertentu 
(3) pola  penyebaran  dan  kepadatan 
mempengaruhi dinamika populasi  
(4) kematian  dan  emigrasi  memperbesar 
ukuran populasi  
 
 
 
57. Lebah tidak digolongkan dalam takson yang 
sama dengan kutu buku sebab lebah .... 

an_ay
(1) metamorfosisnya tidak sempurna 
(2) tidak mengalami metamorfosis 

Halaman 2 
Copyright©2010 www.onlinelesson.eu
copyright protected by the copyright laws

LATIHAN TES POTENSI AKADEMIK SNMPTN 2011


(Keterangan : option yang benar yang ditebalkan!)

Mata Ujian : Tes Potensi Akademik


Jumlah Soal : 75 soal

Tes Sinonim (Persamaan Kata)

1. DIKTUM
A. Ucapan D. Standar
B. Dikte E. Perkiraan
C. perbuatan

2. DAGI
A. Patuh D. Melawan
B. Taat E. Kontra
C. Acuh

3. DISTORSI
A. Keselarasan D. Kepatuhan
B. Kesinambungan E. Merusak
C. Penyimpangan

4. DIRGANTARA
A. Bumi D. Tanah
B. Angkasa E. Awan
C. Udara

5. ELEGI
A. Rindu D. Syair
B. Lagu E. Naskah
C. Lirik

6. RABAT
A. Potongan harga D. Harga jual
B. Nilai Jual E. Keuntungan
C. Pembayaran

7. CITRA
A. Dimensi D. Cinta
B. Bayangan E. Mutu
C. Gambaran

8. PROTEKSI
A. Penjagaan D. Bantuan
B. Pengawasan E. Perlindungan
C. Pengamanan

9. INSOMNIA
A. Cepat tidur D. Khawatir
B. Cemas E. Sedih
C. Sulit tidur

an_ay Copyright©2010 www.onlinelesson.eu


copyright protected by the copyright laws
Copyright©2010 www.onlinelesson.eu
copyright protected by the copyright laws

10. ASUMSI
A. Anggapan D. NIlai
B. Perkiraan E. Persamaan
C. Kesimpulan

11. RANCU
A. Semu D. Ranjau
B. Ragu-ragu E. Tidak wajar
C. Kacau

12. IMAJINASI
A. Harapan D. Inkarnasi
B. Bohong E. Khayalan
C. Ingatan

13. OVAL
A. Bulat telur D. Busur
B. Bundar E. Bulat
C. Lonjong

14. PRIMER
A. Utama D. Mutakhir
B. Terakhir E. Tambahan
C. Sampingan

15. DIKOTOMI
A. Kepala dua D. Dibagi rata
B. Biji dua E. Berkaki dua
C. Dibagi dua

Tes Antonim
1. DAIF
A. Kuat D. Malas
B. Berani E. Lemah
C. Setia

2. ADILUHUNG
A. Mulia D. Berani
B. Hina E. Tinggi
C. Kaya

3. AKUISISI
A. Pengeluaran D. Pendapat
B. Masukan E. Saran
C. Produk

4. ARISTOKRAT
A. Ningrat D. Hartawan
B. Bangsawan E. Terpandang
C. Jelata

5. NOMADIK
A. Berpindah-pindah D. Menetap
B. Perlahan-lahan E. Datang
C. Terus-menerus

an_ay Copyright©2010 www.onlinelesson.eu


copyright protected by the copyright laws
Copyright©2010 www.onlinelesson.eu
copyright protected by the copyright laws

6. AMAR
A. Larangan D. Patokan
B. Perintah E. Peraturan
C. Petunjuk

7. MANDIRI
A. Senang D. Memihak
B. Bergantung E. Antipati
C. Tenggang rasa

8. PROMINEN
A. Utama D. Tertinggi
B. Terkemuka E. Kelas Elit
C. Biasa

9. ALAM FANA
A. Alam baka D. Hidup
B. Alam dunia E. Alam kubur
C. Meninggal

10. PROLOG
A. Pembukaan D. Monoton
B. Akhiran E. Epilepsi
C. Epilog

11. INDEPENDEN
A. Tidak bergantung kepada orang lain
B. Dependen
C. Berdiri sendiri
D. Bekerja mandiri
E. Swadaya

12. MONOTON
A. Terus-menerus D. Diam
B. Berganti-ganti E. Jenuh
C. Tetap

13. KONTINYU
A. Terus-menerus D. Tidak tentu
B. Tak henti-henti E. Terputus-putus
C. Mengalir

14. PERINTIS
A. Pionir D. Penemu
B. Pendahulu E. Pewaris
C. Pemula

15. KENDALA
A. Halangan D. Pendukung
B. Rintangan E. Hambatan
C. Penghalang

Tes Kelompok Kata

an_ay Copyright©2010 www.onlinelesson.eu


copyright protected by the copyright laws
Copyright©2010 www.onlinelesson.eu
copyright protected by the copyright laws

1. A. Sahara D. Amazon
B. Kalahari E. Gobi
C. Sinai

2. A. Konser D. Sandiwara
B. Opera E. Drama
C. Sinetron

3. A. Gulung tikar D. Ambruk


B. Pailit E. Bangkrut
C. Colapse

4. A. Besi D. Tembaga
B. Perunggu E. Batubara
C. Perak

5. A. Jakarta D. Paris
B. London E. Roma
C. Sidney

6. A. Perolehan D. Laba
B. Keuntungan E. Bunga
C. Bahaya

7. A. Faktual D. Kenyataan
B. Otentik E. Konkret
C. Khayalan

8. A. Raden Wijaya
B. Mulawarman
C. Gajah Mada
D. Hayam Wuruk
E. Purnawarman

9. A. Merkurius D. Pluto
B. Uranium E. Uranus
C. Neptunus

10. A. Kilogram D. Gram


B. Ons E. Meter
C. Ton

11. A. Omega D. Romeo


B. Lamda E. Gamma
C. Epsilon

12. A. George Washington


B. Ronal Reagan
C. Abraham Lincoln
D. Arnold Schwarzeneger
E. Barack Obama

13. A. USM ITB D. SMUP UNPAD


B. SIMAKUI E. SNMPTN
C. UMB PTN

14. A. Fisika D. Biologi


B. Kimia E. Matematika
C. Ekonomi

an_ay Copyright©2010 www.onlinelesson.eu


copyright protected by the copyright laws
Copyright©2010 www.onlinelesson.eu
copyright protected by the copyright laws

15. A. UNPAD D. UPI


B. UI E. IPB
C. UIN Bandung

Padanan Hubungan Kata

1. DOKTER : RESEP
A. Apoteker : Obat D. Burung : Sangkar
B. Sopir : Mobil E. Haus : Minum
C. Lapar : Makan

2. POHON : BUAH
A. Telur : Ayam D. Napas : Udara
B. Sapi : Susu E. Daging : Kambing
C. Kambing : Kulit

3. PANAS : API
A. Dingin : Es D. Terang : Matahari
B. Listrik : Nyala E. Minum : Haus
C. Abu : Arang

4. BERAS : NASI GORENG


A. Kayu : Papan D. Kayu : Meja
B. Kayu : Pasak E. Kayu : Ganjal
C. Kayu : Tripleks

5. ANTISEPTIK : KUMAN
A. Gajah : Belalai D. Orang : Rumah
B. Macan : Kuku E. Hutan : Binatang
C. Harimau : Rusa

6. PEDAS : CABAI
A. Gula : Manis D. Kopi : Pahit
B. Manis : Gula E. Jeruk : Masam
C. Sirup : Manis

7. KEPALA : HELM
A. Kaki : Kaus D. Rumah : Atap
B. Album : Foto E. Atap : Genteng
C. Jari : Cincin

8. RAMBUT : GUNDUL
A. Pakaian : Bugil D. Kepala : Botak
B. Jari : Cincin E. Kotor : Sapu
C. Haus : AIr

9. DIAMETER : LINGKARAN
A. Radius : Jari-jari D. Panjang : Lebar
B. Sudut : Lancip E. Sudut : Busur
C. Diagonal : Persegi

10. RAMALAN : ASTROLOGI


A. JIwa : Fisioterapi
B. Kemasyarakatan : Sosiologi
C. Gunung : Demografi
D. Pendidikan : Psikologi
E. Pemerintahan : Antropologi

an_ay Copyright©2010 www.onlinelesson.eu


copyright protected by the copyright laws
Copyright©2010 www.onlinelesson.eu
copyright protected by the copyright laws

11. IKAN : INSANG : LAUT


A. Kereta api : Rel : Gerbong
B. Kesehatan : Vitamin : Kuat
C. Manusia : Paru-paru : Darat
D. Bulan : Bintang : Matahari
E. Ayam : Kandang : Beras

12. ULAT ; KEPOMPONG : KUPU-KUPU


A. Rajin : Belajar: Pandai
B. Makan : Kenyang : Sehat
C. Telur : Ayam : Itik
D. Besar : Sedang : Kecil
E. Telur : Jentik : Nyamuk

13. LAPAR : MAKAN : KENYANG


A. Minum : Haus : Nikmat
B. Sakit : Berobat : Sembuh
C. Sakit : Sembuh : Berobat
D. Haus : Minum : Nikmat
E. Kenyang : Lapar : Makan

14. BENANG : TENUN : KAIN


A. Kain : jahit : Baju
B. Benang : jahit : Celana
C. Kain : Baju : Celana
D. Baju : Celana : Kain
E. Celana : Baju : Kain

15. LAMPU : GELAP : TERANG


A. Makanan : Lapar : Kenyang
B. Makanan : Kenyang : Lapar
C. Minuman : Kenyang : Haus
D. Bulan : Bintang : Malam
E. Siang : Terang : Matahari

Tes Kemampuan Analitik

1. Vicky adalah anak Pak Dewan dan Bu Dewan. Nada adalah anak adiknya Bu Dewan. Apakah
hubungan antara Vicky dan Nada?
A. Saudara ipar
B. Saudara kandung
C. Saudara misan
D. Saudara sepupu
E. Saudara tiri

2. Produksi karet kita tahun ini merosot, tetapi kualitasnya lebih baik.
A. Kualitas karet tahun terdahulu kurang baik
B. Karet kita pasti laku di pasaran
C. Kita tidak menderita kerugian
D. Ada kemajuan yang besar dalam produksi karet
E. Semuanya salah

3. Semua anggota asosiasi profesi harus hadir dalam rapat.


Sementara dokter adalah anggota asosiasi.
A. Semua yang hadir dalam rapat adalah dokter

an_ay
B. Sementara peserta rapat bukan anggota asosiasi profesi

Copyright©2010 www.onlinelesson.eu
copyright protected by the copyright laws
Copyright©2010 www.onlinelesson.eu
copyright protected by the copyright laws

C. Sementara peserta rapat adalah dokter


D. Semua dokter hadir dalam rapat
E. Semua yang hadir bukan dokter

4. Bila A>B; Q>C; R>D, sedangkan A>Q>R dan B>C>D maka:


A. R>B; C>Q
B. D<A; C<R; B=C
C. R>B; Q>B
D. D<A; C>R; B>Q
E. Salah semua

5. Umur Badu sepuluh tahun, setengah dari umur Yanto dan tiga tahun lebih muda dari Ahmad.
Siapakah yang paling tua?
A. Badu
B. Yanto
C. Ahmad
D. Badu dan Yanto
E. Ahmad dan Badu

Tes Seri Angka dan Huruf

1. 2 4 6 8 …
A. 14 D. 11
B. 13 E. 10
C. 12

2. 2 5 10 17 26 …
A. 36 D. 50
B. 37 E. 31
C. 49

3. 1 3 7 15 … …
A. 31 dan 63 D. 30 dan 62
B. 31 dan 62 E. 30 dan 63
C. 31 dan 64

4. 100 5 90 10 80 … …
A. 70 dan 5 D. 70 dan 10
B. 70 dan 15 E. 15 dan 80
C. 15 dan 70

5. B A C A D A E A F A G A …
A. A D. J
B. H E. C
C. I

Tes Aritmatika

1. …/ 7 = 13
A. 81 D. 19
B. 13 E. 91
C. 61

2. 38 x 7 =

an_ay Copyright©2010 www.onlinelesson.eu


copyright protected by the copyright laws
Copyright©2010 www.onlinelesson.eu
copyright protected by the copyright laws

A. 196 D. 166
B. 266 E. 264
C. 164

3. $6,50 + … = $10
A. $2,50 D. $4,50
B. $1,50 E. $2,00
C. $3,50

4. $1,50 + $2,00+ … = $6,00


A. $2 D. $1,50
B. $1 E. $2,50
C. $0,50

5. Berapakah 75 % dari 90?


A. 67,5 D. 9
B. 8,5 E. 12
C. 9,2

an_ay Copyright©2010 www.onlinelesson.eu


copyright protected by the copyright laws

Anda mungkin juga menyukai